Wilkins’ Clinical Assessment in Respiratory Care 7th Edition Heuer Test Bank Chapter 1: Preparing for the Patient Encounter Test Bank MULTIPLE CHOICE 1. Which of the following activities is not part of the role of respiratory therapists (RTs) in
patient assessment? a. Assist the physician with diagnostic reasoning skills. b. Help the physician select appropriate pulmonary function tests. c. Interpret arterial blood gas values and suggest mechanical ventilation changes. d. Document the patient diagnosis in the patient’s chart. ANS: D
RTs are not qualified to make an official diagnosis. This is the role of the attending physician. REF: Table 1-1, pg. 4
OBJ: 9
2. In which of the following stages of patient–clinician interaction is the review of physician
orders carried out? a. Treatment stage b. Introductory stage c. Preinteraction stage d. Initial assessment stage ANS: C
Physician orders should be reviewed in the patient’s chart before the physician sees the patient. REF: Table 1-1, pg. 4
OBJ: 9
3. In which stage of patient–clinician interaction is the patient identification bracelet checked? a. Introductory stage b. Preinteraction stage c. Initial assessment stage d. Treatment stage ANS: A
The patient ID bracelet must be checked before moving forward with assessment and treatment. REF: Table 1-1, pg. 4
OBJ: 9
4. What should be done just before the patient’s ID bracelet is checked? a. Check the patient’s SpO2. b. Ask the patient for permission. c. Check the chart for vital signs. d. Listen to breath sounds. ANS: B
It is considered polite to ask the patient for permission before touching and reading his or her ID bracelet.
REF: pg. 3
OBJ: 3 | 5
5. What is the goal of the introductory phase? a. Assess the patient’s apparent age. b. Identify the patient’s family history. c. Determine the patient’s diagnosis. d. Establish a rapport with the patient. ANS: D
The introductory phase is all about getting to know the patient and establishing a rapport with him or her. REF: Table 1-1, pg. 4
OBJ: 3
6. Which of the following behaviors is not consistent with resistive behavior of a patient? a. Crossed arms b. Minimal eye contact c. Brief answers to questions d. Asking the purpose of the treatment ANS: D
If a patient asks about the purpose of the treatment you are about to give, this generally indicates that he or she is not upset. REF: Table 1-1, pg. 4
OBJ: 3
7. What is the main purpose oTfEthSeTinBiA tiaNl K asSseEsL sm LeEnRt s.taCgOe?M a. To identify any allergies to medications b. To document the patient’s smoking history c. To personally get to know the patient better d. To verify that the prescribed treatment is still needed and appropriate ANS: D
When you first see the patient, you are encouraged to perform a brief assessment to make sure the treatment order by the physician is still appropriate. The patient’s status may have changed abruptly recently. REF: Table 1-1, pg. 4
OBJ: 3
8. What is the appropriate distance for the social space from the patient? a. 3 to 5 feet b. 4 to 12 feet c. 6 to 18 feet d. 8 to 20 feet ANS: B
The social space is 4 to 12 feet. REF: pg. 5
OBJ: 5
9. What is the appropriate distance for the personal space?
a. b. c. d.
0 to 18 inches 18 inches to 4 feet 4 to 12 feet 6 to 15 feet
ANS: B
The personal space is about 2 to 4 feet from the patient. REF: pg. 5
OBJ: 5
10. Which of the following activities is best performed in the personal space? a. The interview b. The introduction c. The physical examination d. Listening for breath sounds ANS: A
The interview is best performed with you sitting about 2 to 4 feet from the patient. If you sit farther away, the patient will have to answer your questions in a louder voice, and because some of the information may be private, this would diminish communication. REF: Table 1-1, pg. 4
OBJ: 5
11. What type of behavior is least appropriate in the patient’s intimate space? a. Eye contact b. Pulse check c. Auscultation d. Simple commands ANS: A
Eye contact is inappropriate in the intimate space and will make the patient very uncomfortable. REF: pg. 3
OBJ: 5
12. You are riding in an elevator at the hospital where you are employed as an RT. The elevator is
full, but standing next to you is Joe, the RT who is scheduled to relieve you. He turns to you and asks, “How is Mr. Copper doing?” Earlier in the day, Mr. Copper had a cardiac arrest, and he is now being mechanically ventilated. How should you respond to Joe? a. “He took a turn for the worse.” b. “He is fine.” c. “Let’s talk later in the report room.” d. “He is on a ventilator and will keep you very busy.” ANS: C
The patient’s right to privacy prevents care providers from discussing a patient’s clinical status in public places. All answers other than “c” are unethical; giving such answers could cause an RT to be in legal trouble and get fired. REF: Table 1-2, pgs. 4-5
OBJ: 6
13. In 1996, Congress passed the HIPAA. What does the letter “P” stand for?
a. b. c. d.
Patient Payment Portability Personal
ANS: C
HIPAA stands for Health Insurance Portability and Accountability Act. REF: pg. 4
OBJ: 6
14. Which of the following techniques for expressing genuine concern is the most difficult to use
appropriately? a. Touch b. Posture c. Eye contact d. Proper introductions ANS: A
Touch is most difficult to use properly because gender and cultural differences often become an issue. REF: Table 1-1, pgs. 3-4
OBJ: 5 | 7
15. Which of the following techniques is not associated with the demonstration of active
listening? a. Good eye contact b. Taking notes while a patient is talking c. Asking for clarification d. Use of touch ANS: D
Use of touch helps with demonstrating empathy but has little to do with active listening. REF: pg. 2
OBJ: 2
16. Two respiratory care students are taking their lunch break and want to compare notes about
patients they have seen during the morning. Which of the following locations would be considered a violation of HIPAA standards? a. The unit nursing station in front of the unit clerk’s desk b. A table in the cafeteria with no one within hearing distance c. The respiratory department report room d. The intensive care unit (ICU) staff break room ANS: A
Patient Health Information (PHI) should be discussed only in nonpublic areas of the hospital. The space in front of the unit clerk’s desk is likely to be occupied with members of the public asking for information. REF: Table 1-2, pgs. 4-5
OBJ: 6
17. A 20-year-old respiratory care student enters the room of a 65-year-old female patient, saying,
“Hi, Linda! I am Joe from Respiratory Care.” He immediately approaches her, looks her in the eye, and places his stethoscope on her chest. a. Joe’s approach to this patient is appropriate. b. Joe has inappropriately entered the patient’s social space. c. Joe has inappropriately entered the patient’s personal space. d. Joe has inappropriately entered the patient’s intimate space. ANS: D
Joe has established no rapport with this patient, has touched her without asking permission, and looks her in the eye while examining her. REF: pg. 4
OBJ: 7
18. A respiratory care student returns from a clinical experience , excited that she has had the
opportunity to perform cardiopulmonary resuscitation (CPR) for the first time. She immediately goes to her Facebook page and describes her day. Which of the following entries would be a violation of HIPAA standards? a. “At clinical today got to do CPR on a patient on the 6th floor of Mercy Hospital. Patient survived! What a rush!!” b. “Got to do CPR for the first time today. Patient survived!! What a rush!” c. “Got to do CPR for the first time in clinical today! What a rush!!” d. All of the above ANS: D
Patient Health Information (PHI) must not be shared in a public location. Facebook is considered a public forum. Although the student did not give specific identifiers in answers a, OiMliar with either the patient or the b, or c, there was enough iT nfE orSmTaB tioAnNtK haSt E soLmLeE onRe.fC am student could possibly have deduced the identity of the patient. REF: Table 1-2, pgs. 4-5
OBJ: 6
19. Which of the following would be the most appropriate way for respiratory care student Amy
Long to initially approach a 58-year-old female patient, Mrs. Nora Jones? a. “Hello, Mrs. Jones. I am Amy from respiratory care, and with your permission I would like to assess you for your treatment.” (Amy stands 5 feet from the patient and makes direct eye contact.) b. Hey there, Nora! Isn’t this a great day!?! I’m Amy and I need to listen to you.” (Amy holds out her stethoscope in front of her and approaches the patient to within 1.5 feet.) c. “Hi, I’m Amy, here to give you your treatment.” (Amy makes no eye contact and looks around the room for a nebulizer.) d. Hi, Mrs. Jones. I’m here for your treatment.” (Amy makes direct eye contact.) ANS: A
The initial contact with a patient should be from the patient’s social space (4 to 12 feet). Patients should be addressed by their last name. When first speaking to a patient, the therapist should make direct eye contact, but he or she should not use direct eye contact when in the patient’s intimate space. REF: Table 1-1, pgs. 3-4
OBJ: 2 | 7
20. The umbrella term patient-centered care includes all of the following elements except: a. Individualized care. b. Assistance with financial and insurance issues. c. Patient involvement. d. Provider collaboration. ANS: B
Patient-centered care involves individualized care, patient involvement, and provider collaboration. REF: pg. 2
OBJ: 1
21. The golden rule of bedside care can be summarized as: a. Patients should be cared for primarily at the bedside. b. All patient services (e.g., x-ray, nursing care, respiratory care) should be delivered
to the patient at the bedside whenever possible. c. As a caregiver, at all times treat a patient as you would hope to be treated if you
were the patient. d. Make sure that all safety equipment is in place at the bedside for maximum patient protection from hazards such as falls. ANS: C
The golden rule is that as a caregiver, you treat patients the way you wish to be treated. REF: pg. 2
OBJ: 1
22. In interacting with patientsT ,E beShT avBioArsNsKuS chEaLsLbE odRy.m CoOvMements, touch and eye movements,
and facial expressions would be examples of: a. Nonverbal communication. b. Expressions of caregiver interest in patient welfare. c. Mechanisms to put patients at ease. d. None of the above. ANS: A
These are mechanisms of nonverbal communication that help to put patients at ease and can be used to communicate caregiver concern to patients. REF: pg. 2
OBJ: 2
23. In determining the course of treatment for a 20-year-old patient hospitalized for exacerbation
of cystic fibrosis, the most effective course of action would be: a. Formulating a treatment plan based on the therapist’s knowledge of the disease and its treatment and then presenting it to the patient. b. Formulating a treatment plan with the physician and nurse and then presenting it to the patient. c. Interviewing the patient and strictly following the patient’s preferences with regard to treatment. d. Interviewing the patient to determine his or her preferences for treatment, formulating a treatment plan in collaboration with the nurse and physician based on both patient preferences and the team’s knowledge of the disease and its
treatment, and presenting it to the patient. ANS: D
Patient-centered care must be highly collaborative, with input from both the patient and caregivers. REF: pg. 7
OBJ: 4 | 8
24. While interviewing a patient in a room with another patient and that patient’s family present,
the appropriate course of action for an RT would be to: a. Introduce himself or herself to the patient from a distance of about 5 to 7 feet and proceed with the interview. b. Introduce himself or herself to the patient from a distance of about 5 to 7 feet, move to within 2 to 3 feet of the patient, and proceed with the interview. c. Introduce himself or herself to the patient from a distance of about 5 to 7 feet, move to within 2 to 3 feet of the patient, draw the privacy curtain around the bed, and proceed with the interview. d. Introduce himself or herself to the patient from a distance of about 5 to 7 feet, draw the privacy curtain around the bed, sit on the bed about 1 foot from the patient, and proceed with the interview. ANS: C
Normally, interviews are carried out in the personal space (within 18 inches to 4 feet of the patient), not the intimate space (from 1 to 18 inches from the patient). The privacy curtain should be in place because others are in the room. REF: pg. 3
OBJ: 3
25. A male therapist is discussing a treatment plan with a female patient who is sitting up in bed,
dressed in a hospital gown and wearing a full head covering with only her face showing. Her husband is in the room, and from previous encounters it is clear that she defers to him. The most effective way to present this treatment plan would be for the therapist to: a. Present the patient with a written summary of the plan and ask her to look it over. b. Ask the woman’s husband to step out of the room while the plan is being discussed with the patient. c. Explain the plan to the patient, maintaining eye contact with her at all times and encouraging her to ask any questions she might have. d. Explain the plan to the patient and her husband, and encourage both to ask any questions they might have. ANS: D
From the woman’s dress and previous behavior, it is likely that she is Muslim, with traditional values and customs. Therefore, both she and her husband will find it inappropriate, if not offensive, that he not be included in discussions of treatment. Cultural values must be taken into account if truly effective patient treatment is to occur. REF: pg. 5
OBJ: 4 | 7
26. The therapist enters the room of a 6-year-old victim of an automobile accident who is
unconscious and receiving ventilation therapy. The therapist assesses the patient, gives a treatment, and suctions the patient. The patient’s mother then asks in a worried voice, “Is he going to be all right?” The appropriate response for the therapist would be: a. “I just looked at the CT scan of his brain, and I believe the swelling is going down. He should recover within the next couple of weeks.” b. “I’m unable to give you any information about your child’s condition.” c. “I am just the respiratory therapist, and I really do not know anything.” d. “I’m sorry, but our policy is that only the doctor can give you information about your child’s prognosis. Let me step out and find out when the doctor will be back in the unit.” ANS: D
Response “a” is inappropriate because it is not within the scope of practice of an RT to render medical opinions about a patient’s condition to the family. Answer “b,” while correct, is unnecessarily abrupt. Answer “c” is both abrupt and is probably not true. Answer “d” meets HIPAA and hospital policy requirements while also providing good patient care and customer service. REF: pgs. 4-5
OBJ: 6
27. In order to deliver effective patient education for use of a particular treatment, the first step
should be to: a. Describe to the patient the equipment that will be used for the treatments. b. Describe to the patient the medications that will be used for the treatments. c. Assess the patient’s learning needs by identifying learning barriers, determining the way the patient besT t lEeaSrT nsB, A anNdKeS vaEluLaL tinEgRt. heCpOaM tient’s readiness to learn. d. Describe to the patient the schedule for the treatments to be given. ANS: C
Although the patient will eventually have to learn about equipment, medications, and schedules, this learning will not occur effectively until the patient’s learning needs are determined. REF: pgs. 6-7
OBJ: 9
28. One effective teaching tool is the teach-back method. This is: a. A technique where the teacher explains the procedure to the learner and has the
learner repeat the information in his or her own words. b. A technique where the teacher explains the procedure to the learner and then has
the learner explain it to family members after the teacher has left the room. c. A technique where the teacher gives the equipment to the learner and has the
learner experiment with it until he or she can use it correctly. d. None of the above. ANS: A
The teach-back method has the learner hear the explanation and then give a “return demonstration” to the teacher to be sure that the learner has the correct information. REF: pg. 7
OBJ: 9
29. Prior to discharge, patients should receive a written action plan that establishes treatment
goals and self-care activities. The acronym SMART is helpful in establishing the action plan. The “M” in SMART stands for: a. Meaningful (the goal pertains to the action plan). b. Mastering (the goal). c. Modular (the goal is divided into sections). d. Measurable (the outcome should be measurable). ANS: D
In order for a treatment to be effective, its outcome must be measurable. REF: pg. 7
OBJ: 10
30. Failure of care providers to collaborate with one another potentially results in: a. Patient safety placed at risk. b. Duplication of effort by different caregivers. c. Delivery of less-than-optimum care. d. All of the above. ANS: D
In addition to “a,” “b,” and “c,” lack of collaboration also may result in an increased length of stay and wasted healthcare resources. REF: pgs. 9-11
OBJ: 13 | 14 | 15
31. An RT enters a room for a patient’s second treatment of the day and notes that the patient is
dyspneic and tachycardic and has rales that can be heard throughout all lung fields but are especially prominent in theTbEaS seTs.BAAtNreKaS tmEeL ntLiE sR gi. veCnOwMith little positive change. The therapist should: a. Note the treatment and its results in the patient chart. b. Note the treatment and its results in the patient chart and tell the unit clerk to have the nurse see the patient soon. c. Find the patient’s nurse and together determine what is needed for the patient (e.g., different drug therapy, call physician, call rapid response team). Once the patient’s condition has been adequately addressed, the therapist’s actions should be documented in the chart. d. Tell the patient you will check back with him in an hour to see how he is doing. ANS: C
It is critical that results of an assessment and treatment be communicated to other members of the health team, either in the chart or verbally if the situation demands immediate action. Chart documentation should include findings of the assessment, results of any treatments delivered, and actions taken to resolve the situation. REF: pg. 10
OBJ: 15
32. Good communication is especially critical when “handing off” a patient to another caregiver
at the end of the shift. To ensure that adequate information is transmitted, the SBAR format is useful. This acronym stands for situation, background, assessment, and . a. Recommendation. b. Results.
c. References. d. Repeat (information) ANS: A
The fourth letter in SBAR stands for recommendation. REF: pg. 10
OBJ: 14
33. According to the American Association for Respiratory Care (AARC), patient discharge plans
should include which of the following? a. Transportation for the patient when the patient goes home b. Ascertaining that once the patient has been discharged, the patient has adequate financial resources to pay rent and utilities c. Notification of the patient’s family and/or other caregivers of the patient’s imminent discharge d. Methods for the ongoing assessment of outcomes ANS: D
Discharge planning should focus on providing a continuum of care with transition from the hospital to the alternate site. REF: pg. 11
OBJ: 14
34. All of the following represent good hygiene practices except: a. Gloves should be worn when touching a patient only when a caregiver is likely to
come into contact with secretions or infectious materials. b. Hands should be washed when first encountering a patient, after leaving a patient, and before and after anT y EstS erTilB eA orNcKleSaE nL prL ocEeR du.rC eO isMperformed on a patient. c. All infectious waste should be disposed of in proper waste containers. d. Place all needles and blades in “sharps” containers when you are finished with them. ANS: A
Gloves should be worn whenever a patient is touched. REF: pg. 9, Box 1-4
OBJ: 12
35. The main objective of the I Speak Up initiative from the National Institute of Health (NIH) is: a. Be sure that all billing for patient medical expenses occurs correctly. b. Let caregivers know that the family wants to be informed at all times. c. Let the hospital know when ancillary services such as food quality and parking are
inadequate. d. Help ensure that a patient’s care is as safe and effective as possible. ANS: D
The I Speak Up initiative is a comprehensive program focused on making sure that patient care is as safe and effective as possible. It especially emphasizes the importance of active patient and family involvement in all aspects of patient care, including enhanced patient safety and reduction in medical errors. REF: pg. 8, Box 1-3
OBJ: 11
Chapter 2: The Medical History and the Interview Test Bank MULTIPLE CHOICE 1. Communication between two people can occur only if: a. The speaker speaks clearly. b. The receiver understands the message. c. Each person is willing to listen to the other. d. All of the above. ANS: D
Communication occurs only if both parties speak clearly and listen carefully. REF: pg. 16
OBJ: 1
2. Communication between individuals is affected by all of the following factors except: a. The time of day. b. The cultural heritage. c. The religious beliefs. d. The level of education. ANS: A
The time of day is not likely to affect communication in a significant way. REF: pg. 16
OBJ: 2
3. When one is conducting an interview with a patient, which of the following points is most
important in facilitating an effective interaction with the patient? a. Recognizing the nonverbal signals that the patient is sending b. Your ability to project a sense of undivided interest in the patient c. Introducing yourself appropriately at the beginning of the interview d. Answering all of the patient’s questions completely without using jargon ANS: B
Patients can sense when we are distracted and will not communicate well in such a situation. Glancing out the window or at the television will tell the patient that you are distracted and are not that interested in what he or she has to say. REF: pgs. 16-17
OBJ: 3
4. Which of the following types of questions are preferred for all interactions with a patient? a. Direct questions b. Neutral questions c. Indirect questions d. Open-ended questions ANS: B
Neutral questions encourage the patient to respond with sentences and honest answers. REF: pg. 17
OBJ: 3
5. If a patient is unable to provide an accurate history, the respiratory therapist (RT) should: a. Ask a family member or friend to supply the information. b. Ask shorter and more direct questions to determine the cause of the problem. c. Proceed to treat the patient’s symptoms based on information already obtained. d. Refuse to treat the patient because effective therapy cannot be initiated without a
complete history. ANS: A
In many situations, the patient is unable to answer questions. Family members are often the next best source for important information. REF: pg. 18
OBJ: 4
6. Which of the following should the RT keep in mind when obtaining a pulmonary history? a. Assessment usually is limited to the respiratory system. b. Evaluation of the patient’s entire health status is essential. c. Signs and symptoms of pulmonary disease will rarely be seen outside the
cardiopulmonary system. d. Patients with long-standing chronic disease can give a detailed account of how their lives have changed and of the signs and symptoms that the disease has caused. ANS: B
Pulmonary problems often cause health issues in other body systems, and health problems in other systems often cause pulmonary problems. REF: pg. 18
OBJ: T2E| 6STBANKSELLER.COM
7. Obtaining background information during an interview is very important because it allows the
interviewer to: a. Learn the impact of culture, relationships, and finances on his or her health. b. Predict whether the patient is willing to cooperate in the treatment of his or her disease. c. Develop a basic understanding of the patient’s experience with his or her disease. d. Achieve all of the above. ANS: D
Background information is often overlooked but is important for assessing the impact of chronic illness on the patient and for identifying how the patient is coping with the illness. It also tells the interviewer whether the patient is able to cooperate with the treatment plan. REF: pg. 18
OBJ: 2 | 6
8. Screening information is: a. Obtained at health fairs to determine whether the person should see a physician. b. Designed to uncover problem areas that the patient forgot to mention or omitted. c. Given to the interviewer by the patient at the very beginning of the interview
process. d. Obtained by the triage nurse in the emergency department to determine how life-
threatening the patient’s symptoms are.
ANS: B
Screening information is designed to identify important facts that the patient may have overlooked in the interview. REF: pgs. 18-19
OBJ: 2
9. The review of systems is very important because it provides the interviewer with: a. Information grouped by major organs and physiologic systems. b. Additional subjective information about the patient’s problem. c. A combination of subjective and objective data in a narrative form. d. Information relevant to the patient’s problem that may have been overlooked. ANS: D
The review of systems helps identify key information that may have been overlooked in the interview. REF: pg. 19
OBJ: 2
10. A pertinent negative is defined as: a. Any negative response made by the patient during the interview. b. Refusal by the patient to answer questions about a certain topic. c. Any negative response by the patient to an important question about possible
symptoms. d. A negative response by a patient to a therapist who asks whether the patient needs a breathing treatment. ANS: C
If the patient appears to haT veEpSnT euBmAoNnK iaSbE utLdLeE niR es.hCaO viM ng a cough, the patient’s negative response to the question about coughing would be a pertinent negative. REF: pg. 19
OBJ: 3
11. A pertinent positive is defined as: a. Any positive response made by the patient during the interview. b. A direct question asked during the interview that elicits a positive response. c. An affirmative response by the patient when asked whether he or she needs a
breathing treatment. d. An affirmative response to an important interview question about the patient’s
symptoms. ANS: D
A positive response from the patient regarding a symptom associated with the possible diagnosis would be a pertinent positive. REF: pg. 19
OBJ: 3
12. The main purpose of the chief complaint is to: a. Give a brief explanation about why the patient sought health care. b. Direct the interviewer to the organ system in which the problem is located. c. Present a diagnosis that is based on information obtained during the interview. d. List symptoms in order from most severe to least severe according to organ system
involvement.
ANS: A
The chief complaint explains why the patient sought medical help. REF: pg. 19
OBJ: 2 | 3
13. Which of the following cardiopulmonary conditions would not be found in the chief
complaint list? a. Asthma b. Wheezing c. Chest pain d. Hemoptysis ANS: A
Asthma is a diagnosis and not a complaint. REF: pg. 19 | pg. 21
OBJ: 2
14. When a patient is interviewed so the chief complaint can be determined, the best questions
that can be asked to elicit this information are: a. Direct questions. b. Neutral questions. c. Open-ended questions. d. A combination of the above. ANS: D
Use of a variety of types of questions usually yields the best interview results. REF: pg. 17 | pg. 19
TESTBANKSOEBL J: LE 2 R.COM
15. Which of the following would not be found in the past medical history? a. Injuries and accidents b. Surgeries and hospitalizations c. Associated symptoms and aggravating factors d. Over-the-counter medications, vitamins, and “home remedies” ANS: C
Associated symptoms and aggravating factors would be found in the history of present illness. REF: pg. 22
OBJ: 2
16. Which of the following formulas should be used to calculate the pack-year history of cigarette
consumption? a. Packs per day times Years smoked b. Packs per day minus Years smoked c. Packs per day plus Years smoked d. Packs per day plus Years smoked ANS: A
Pack-years is the standard way to document a patient’s smoking history. This is determined by multiplying the number of packs smoked times the number of years smoked. REF: pg. 24
OBJ: 2
17. One disadvantage of using the pack-year method for calculating cigarette consumption is that: a. The method is not used widely throughout the United States. b. Advanced mathematical calculations are required to obtain the figure. c. The method does not reveal how many packs per day were smoked over how many
years. d. The values are more meaningful to RTs and physicians than to other healthcare
providers. ANS: C
Although pack-years is an industry standard for documenting quantity of tobacco consumption, this value does not provide the details. REF: pg. 24
OBJ: 2
18. Which of the following is not a reason for obtaining a family history? a. To assess the current health status of the extended family b. To learn about the health status of the patient’s blood relatives c. To determine whether the patient is adopted d. To identify the presence in the family of diseases with a hereditary tendency ANS: C
The reason for obtaining a family history is to identify blood relatives who may have a disease that the patient has inherited. REF: pg. 25
OBJ: 2
19. Which of the following disT eaEsS esTw ldKnSoE t bLeLrE ecR or.dC edOiM n the family history as a hereditary BoAuN
disorder? a. Asthma b. Pneumonia c. Cystic fibrosis d. Alpha1-antitrypsin deficiency ANS: B
Pneumonia is not hereditary. REF: pg. 25
OBJ: 2
20. Modern-day office workers may be exposed to which of the following occupational and
environmental diseases? a. Bagassosis b. Monday fever c. Pneumoconiosis d. Sick building syndrome ANS: D
Sick building syndrome occurs when employees are exposed to low doses of toxic gases in a building that is airtight and has poor ventilation. REF: Table 2-1, pg. 27
OBJ: 2
21. Which of the following symptoms is often seen in patients with tight building syndrome? a. Headache b. Stuffy nose c. Cough d. All of the above ANS: D
Tight building syndrome can cause a variety of symptoms. REF: Table 2-1, pg. 27
OBJ: 2
22. What pulmonary disorder is associated with visiting or living in Ohio, Maryland, and the
central Mississippi Valley? a. Blastomycosis b. Histoplasmosis c. Coccidioidomycosis d. Silicosis ANS: B
Histoplasmosis occurs only in the midwestern regions of the country. REF: Table 2-1, pg. 28
OBJ: 2
23. Who writes the initial admission note? a. The physician b. The head nurse c. The head RT d. Any of the above ANS: A
Only the physician writes the initial admission note, which describes why the patient is being admitted. REF: pg. 28
OBJ: 2 | 6
24. Who writes the progress notes each day? a. The physician b. The physical therapist c. The nurse d. Any of the above ANS: D
Any clinician who provides treatment to the patient can write a progress note in the chart. REF: pg. 28
OBJ: 2 | 6
25. A good interview should contain all of the following elements except: a. The interviewer should dress and act professionally. b. The interviewer should project a sense of undivided interest. c. The interviewer should use a formal speaking style. d. The interviewer should respect the patient’s beliefs and attitudes. ANS: C
The interviewer should use an informal, relaxed, conversational style. REF: pg. 17
OBJ: 2 | 3
26. In the physical examination, objective data gathered are referred to as: a. Measurements. b. Symptoms. c. Variables. d. Signs. ANS: D
Objective data are referred to as “signs” in the physical examination. These are data that can be perceived by the examiner, either by measurements or observations. REF: pg. 19
OBJ: 5
27. In the physical examination, subjective data gathered are referred to as: a. Measurements. b. Symptoms. c. Variables. d. Signs. ANS: B
Subjective data in the physical examination are referred to as symptoms. These are data that can only be perceived by the patient. A good interviewer, however, is able to ask questions that prompt the patient to give complete and accurate descriptions of his or her symptoms. REF: pg. 19
OBJ: T5ESTBANKSELLER.COM
28. An RT examining a patient auscultates wheezes in the right lower lobe (RLL). This would be
an example of: a. Objective data (a sign). b. Objective data (a symptom). c. Subjective data (a sign). d. Subjective data (a symptom). ANS: A
Because wheezes are perceived by the examiner, this would be an example of a sign. REF: pg. 19
OBJ: 5
29. A patient being interviewed says that he wakes up nearly every night feeling very hot and
with his body bathed in sweat. This fact, along with his hemoptysis, is a very strong indication that he may have tuberculosis. The night sweats would be an example of: a. Objective data (a sign). b. Objective data (a symptom). c. Subjective data (a sign). d. Subjective data (a symptom). ANS: D
Because the night sweats are perceived only by the patient and then described to the examiner, they would be an example of subjective data (a symptom).
REF: pg. 19
OBJ: 5
30. A “constitutional” symptom would include all of the following except: a. Chills and fever. b. Anorexia and/or weight loss. c. Fatigue. d. Wheezing. ANS: D
Constitutional symptoms generally involve the whole body and are not specific to individual systems. Wheezing is definitely specific to the respiratory system. REF: pg. 21
OBJ: 6
31. All of the following questions would be effective in eliciting additional information about a
patient’s location and level of pain except for question . a. Earlier, you briefly mentioned some pain around your lower ribs. Could you tell me more about it? b. Earlier, you briefly mentioned some pain around your lower ribs. Could you point to exactly where it is? c. Earlier, you briefly mentioned some pain around your lower ribs. What do you think is causing it? d. Earlier, you briefly mentioned some pain around your lower ribs. Could you rate this pain for me on a scale of 1 to 10, with 1 meaning no pain and 10 meaning the worst pain possible? ANS: C
A good interviewer is looking for a precise description of a symptom. It is unlikely that the patient has any idea what is causing the pain. REF: pg. 21
OBJ: 2
32. A history of a patient’s tobacco use is important for all of the following reasons except: a. There is a strong relationship between smoking and chronic obstructive pulmonary
disease (COPD). b. There is a strong relationship between smoking and the use of illicit drugs such as
marijuana, cocaine, and heroin. c. There is a strong relationship between smoking and cardiovascular disease. d. There is a strong relationship between smoking and lung cancer. ANS: B
There is no published evidence that shows that smoking increases the likelihood that a person will use illicit drugs. There is ample evidence of the relationship between smoking and COPD, cardiovascular disease, and lung cancer. REF: pg. 22
OBJ: 2 | 3 | 6
33. Monday fever is: a. The term describing workers who habitually do not want to return to work after the
weekend. b. The tendency of workers to have poor levels of concentration in the workplace
after the weekend. c. An example of a hypersensitivity reaction to a toxic inhaled substance that is worst
after the initial exposure but becomes progressively less pronounced as the exposure continues. d. None of the above. ANS: C
First exposure to a toxic inhaled substance may cause an immediate reaction (as on Monday following a weekend), which gradually becomes less severe with continued exposure (i.e., exposure throughout the rest of the week). REF: pg. 27
OBJ: 6
34. In which section of the patient record should the following entry appear? “Day 4 of
hospitalization. Patient febrile (39o C). Rales in both lung bases; sputum moderate, thick, green, tinged with blood. Plan: Continue ordered antibiotic therapy. Bronchoscopy tomorrow in AM.” a. Admission note b. Physician orders c. Progress notes d. Discharge plan ANS: C
The physician should see the hospitalized patient at least once daily to identify the patient’s general condition, progress, and response to treatment. These findings are summarized in the progress notes. REF: pg. 28
OBJ: T6E| 7STBANKSELLER.COM
35. The RT has been called STAT to the bedside of a patient who is in extreme respiratory
distress in the intensive care unit (ICU). Shortly after arriving, the patient’s cardiac monitor shows a pattern of ventricular fibrillation, and shortly thereafter the patient ceases breathing. The nurse states that the patient has a do not resuscitate (DNR) order. The RT should take which of the following actions? a. Do nothing, because a DNR order means that no resuscitative measures should be attempted. b. Call a full code and institute full resuscitative measures because the therapist knows that the physician who wrote the DNR order is not working in the unit this week. c. Call a code so that cardiac medications can be given to correct the ventricular fibrillation; the DNR order means that the patient cannot be intubated. d. Place a non–rebreather mask on the patient for “comfort measures.” ANS: A
A DNR order signed by a physician means that no resuscitative measures should be undertaken. REF: pg. 28
OBJ: 8
Chapter 3: Cardiopulmonary Symptoms Test Bank MULTIPLE CHOICE 1. Which of the following sequences of events best describes the cough mechanism? a. Inspiration, closure of glottis, forceful opening of glottis b. Opening of glottis, relaxation of diaphragm, closure of glottis c. Inspiration, forceful opening of glottis, contraction of diaphragm d. Opening of glottis, contraction of diaphragm, explosive release of trapped
intrathoracic air ANS: A
The cough mechanism can be divided into three phases: 1. Inspiratory phase: Reflex opening of the glottis and contraction of the diaphragm and the thoracic and abdominal muscles cause a deep inspiration with a concomitant increase in lung volume accompanied by an increase in the caliber and length of the bronchi. 2. Compression phase: Closure of the glottis and relaxation of the diaphragm while the expiratory muscles contract against the closed glottis can generate very high intrathoracic pressures and narrowing of the trachea and bronchi. 3. Expiratory phase: Opening of the glottis and explosive release of trapped intrathoracic air occur, along with vibration of the vocal cords and of the mucosal lining of the posterior laryngeal wall, which shakes secretions loose from the larynx and moves undesired material out of the respiratory tract. REF: pg. 33
OBJ: T1ESTBANKSELLER.COM
2. Which of the following mechanisms does not explain why patients with chronic obstructive
pulmonary disease (COPD) have a poor cough? a. Weak muscles of breathing b. Increased elastic recoil c. Airway obstruction d. Glottis function failure ANS: B
The effectiveness of a cough is reduced when one or more of the following conditions exist: (1) weakness of the inspiratory or expiratory muscles; (2) inability of the glottis to open or close correctly; (3) obstruction, collapsibility, or alteration in shape or contours of the airways; (4) decrease in lung recoil, as occurs with emphysema; and (5) abnormal quantity or quality of mucus production (e.g., thick sputum). REF: pg. 33
OBJ: 1
3. Chronic productive cough is caused most commonly by which of the following clinical
conditions? a. Postnasal drip b. Viral infection c. Upper airway infection d. Bronchopulmonary disease
ANS: A
Chronic persistent cough is caused most commonly by postnasal drip syndrome, followed by acute asthma, acute exacerbation of COPD, allergic rhinitis, gastroesophageal reflux disease (GERD), chronic bronchitis, bronchiectasis, and other conditions such as left heart failure, bronchogenic cancer, and sarcoidosis. REF: pg. 34
OBJ: 1
4. Which of the following is not a complication of forceful coughing? a. Rib fracture b. Pneumothorax c. Pleural effusion d. Torn chest muscle ANS: C
The vigorous muscular activity and high intrathoracic pressures created by forceful coughing may produce a number of complications, such as torn chest muscles, rib fractures, disruption of surgical wounds, pneumothorax or pneumomediastinum, syncope (fainting), arrhythmia, esophageal rupture, and urinary incontinence. REF: pg. 34
OBJ: 1
5. Which of the following conditions is not associated with a characteristic “hacking” cough? a. Smoking b. Viral infection c. Nervous habit d. Lung cancer ANS: D
Hacking (frequent brief periods of coughing or clearing the throat) may be dry and may be the result of smoking, a viral infection, a nervous habit, or difficult-to-move secretions, as occur with postnasal drip. REF: pg. 34
OBJ: 1
6. Which of the following is the best definition of sputum? a. Secretions b. Thick tenacious secretions c. Secretions from the lungs and lower airways d. Secretions from the nose, mouth, and tracheobronchial tree ANS: D
Sputum is the substance expelled from the tracheobronchial tree, pharynx, mouth, sinuses, and nose by coughing or clearing the throat. REF: pg. 35
OBJ: 1
7. What term is used to describe secretions strictly from the lungs and lower airways? a. Phlegm b. Sputum c. Mucus
d. Empyema ANS: A
The term phlegm refers strictly to secretions from the lungs and tracheobronchial tree. REF: pg. 35
OBJ: 1
8. Excessive sputum production is associated with all the following conditions except: a. Allergies. b. Cigarette smoking. c. Airway infection. d. Pleural infection. ANS: D
Excessive sputum production is caused most often by inflammation of the mucous glands that line the tracheobronchial tree. Inflammation of these glands occurs most often with airway infection, cigarette smoking, and allergies. REF: pg. 35
OBJ: 1
9. Which of the following terms best describes foul-smelling sputum? a. Fetid b. Mucoid c. Purulent d. Mucopurulent ANS: A
Copious, foul-smelling (fetid) sputum that separates into layers when standing occurs with bronchiectasis and lung abscess when the patient’s position is changed. REF: pg. 36
OBJ: 1
10. A patient presents in the emergency department with blood-tinged sputum. The term
associated with such sputum is: a. Gelatinous. b. Hemoptysis. c. Hematemesis. d. Mucopurulent. ANS: B
Hemoptysis, expectoration of sputum that contains blood, varies in severity from slight streaking to frank bleeding. REF: pg. 36
OBJ: 1
11. Which of the following conditions is believed to be the most common cause of hemoptysis? a. Tuberculosis b. Erosive bronchitis c. Bronchogenic carcinoma d. Pneumonia ANS: B
Erosive bronchitis in smokers with chronic bronchitis now accounts for nearly half the cases of hemoptysis. REF: pg. 36
OBJ: 1
12. Which of the following definitions is consistent with massive hemoptysis? a. 400 mL in 3 hours b. 400 mL in 12 hours c. 600 mL in 48 hours d. 600 mL in 72 hours ANS: A
Massive hemoptysis (400 mL in 3 hours, or more than 600 mL in 24 hours) is seen with lung cancers, tuberculosis, bronchiectasis, and trauma. REF: pg. 37
OBJ: 1
13. Hemoptysis in a patient with sudden onset of chest pain who is at risk for venostasis is
suggestive of which of the following conditions? a. Pneumonia b. Pulmonary embolism c. Myocardial infection d. Atelectasis ANS: B
Hemoptysis associated with sudden onset of chest pain and dyspnea in a patient at risk for venous stasis of the legs must lead to assessment of that patient for pulmonary embolism and possible infarction. REF: pg. 37
OBJ: 1
14. In the presence of nausea and vomiting, a history of cirrhosis of the liver suggests which of
the following organs as the source of hematemesis? a. Kidneys b. Duodenum c. Esophagus d. Liver ANS: C
The presence of symptoms such as nausea and vomiting, especially with a history of alcoholism or cirrhosis of the liver, may suggest the esophagus or stomach as the source of hematemesis. REF: pg. 37
OBJ: 1
15. Dyspnea is defined as: a. A visible increase in the work of breathing. b. An increase in respiratory rate and depth. c. Breathlessness as perceived by the patient. d. Difficulty breathing in the upright position. ANS: C
Dyspnea (dys, difficult; pnea, breathing) is defined as a subjective experience of breathing discomfort that consists of qualitatively distinct sensations that vary in intensity. REF: pg. 38
OBJ: 1
16. Which of the following grading systems is useful in qualifying the degree of dyspnea? a. Murray lung injury score b. Modified Borg scale c. Dubowitz score d. Asthma score ANS: B
The modified Borg scale, as shown as Table 3-5, pg. 39, uses a 0 to 10 grading system, with descriptive terms to depict the perceived intensity of a symptom such as dyspnea after a specified task. REF: Table 3-5, pg. 39
OBJ: 1
17. Dyspnea tends to occur when which of the following is present? a. Work of breathing that is adequate for a given level of exertion b. Decrease in ventilatory drive to breathe c. Increase in ventilatory capacity d. Increase in ventilatory drive to breathe ANS: D
It is helpful to remember that patients with respiratory disorders will complain of dyspnea when any of the following is present alone or in combination: 1. The work of breathing isTaEbS noTrB mA alN lyKhSigEhLfL orEtR he.gCivOeM n level of exertion. This is common with narrowed airways, as in asthma, and when the lung is stiff, as in pneumonia. 2. The ventilatory capacity is reduced. This is common when the vital capacity is abnormally low, as is seen in patients with neuromuscular disease. 3. The drive to breathe is elevated beyond normal (e.g., hypoxemia, acidosis, exercise). REF: pg. 39
OBJ: 1
18. Breathing at a rate and depth in excess of the body’s metabolic need is known as: a. Hyperventilating. b. Hyperpnea. c. Platypnea. d. Psychogenic dyspnea. ANS: A
Hyperventilating is breathing at a rate and depth in excess of the body’s metabolic need, which causes a decrease in arterial carbon dioxide (PaCO2) and results in a decrease in cerebral blood flow. REF: pg. 40
OBJ: 1
19. All of the following conditions are associated with acute dyspnea in children except: a. Asthma. b. Cystic fibrosis. c. Bronchiolitis.
d. Croup. ANS: B
Acute dyspnea in children is associated most frequently with asthma, bronchiolitis, croup, and epiglottitis. REF: pg. 41
OBJ: 1
20. Which of the following diseases is one of the most common causes of chronic dyspnea in
adults? a. Asthma b. Bronchiolitis c. Cystic fibrosis d. Congestive heart failure (CHF) ANS: D
COPD and chronic CHF are the most common causes of chronic dyspnea in adults. REF: pg. 41
OBJ: 1
21. Paroxysmal nocturnal dyspnea is associated commonly with which of the following
conditions? a. Asthma b. COPD c. CHF d. Obstructive sleep apnea ANS: C
Paroxysmal nocturnal dyspnea (PND) is the sudden onset of difficulty in breathing that occurs when a sleeping patient is in the recumbent position. It often is associated with coughing and is relieved when the patient assumes an upright position. In patients with CHF, PND usually occurs 1 to 2 hours after lying down and is caused by the gradual transfer to the lungs of fluid in the lower extremities. REF: pg. 41
OBJ: 1
22. An inability to breathe while lying down is known as: a. Dyspnea. b. Platypnea. c. Orthopnea. d. PND. ANS: C
Orthopnea is the inability to breathe when lying down. REF: pg. 41
OBJ: 1
23. Chest pain is the cardinal symptom of which of the following diseases? a. Heart disease b. Pleural disease c. Chest wall disease d. Neuromuscular disease
ANS: A
Chest pain is the cardinal symptom of heart disease. REF: pg. 42 | pg. 44
OBJ: 1
24. What is the difference between pleuritic and nonpleuritic chest pain? a. Pleuritic pain is inspiratory; nonpleuritic pain is expiratory. b. Pleuritic pain is sharp and stabbing; nonpleuritic pain is dull and crushing. c. Pleuritic pain usually involves the pleura; nonpleuritic pain involves the lungs. d. Pleuritic pain involves the lung itself; nonpleuritic pain involves the chest wall. ANS: B
Pleuritic pain, often described as inspiratory pain, is the most common symptom of disease; it is the result of inflammation of the pleura (pleurisy). It is sharp, often abrupt in onset, and severe enough to cause the patient to seek medical help (often within hours of onset). It increases with inspiration, a cough, a sneeze, a hiccup, or laughing. Pleuritic pain usually is localized to one side of the chest—frequently, the lower lateral aspect. It may be only partially relieved by splinting and pain medication. Pleuritic pain increases with pressure and movement but not to the same degree as pain originating from the outer chest wall. In contrast, the lung parenchyma and the visceral pleura that cover the lungs are relatively insensitive to pain; therefore, pain with breathing usually indicates involvement of the parietal pleura. REF: pg. 44
OBJ: 1
25. All the following pulmonary conditions are associated with syncope except: a. Pulmonary embolism. b. Prolonged bouts of coughing. c. Hyperoxia. d. The Valsalva maneuver. ANS: C
Pulmonary causes of syncope include pulmonary embolism (obstruction of blood flow from the right heart to the left heart), prolonged bouts of coughing (tussive syncope), and hypoxia (low levels of blood oxygen) or hypocapnia (low levels of carbon dioxide). Holding one’s breath following a deep inspiration (Valsalva maneuver) results in high intrathoracic pressure and decreased venous return to the heart. REF: pg. 47
OBJ: 1
26. Cough syncope is associated most often with which of the following types of patients? a. Middle-aged men with underlying COPD b. Pregnant women c. Children with asthma d. Older obese men with diabetes ANS: A
Cough (tussive) syncope is the transient loss of consciousness that may follow severe coughing. It occurs most commonly in middle-aged men with underlying COPD who are outgoing and moderately obese and have a large appetite for food, alcohol, and smoking.
REF: pg. 47
OBJ: 1
27. The presence of anasarca is associated commonly with which of the following conditions? a. Dyspnea b. Edema c. Hepatomegaly d. Pleural effusion ANS: B
Edema is soft tissue swelling that results from an abnormal accumulation of fluid. It may be generalized (anasarca), may appear only in dependent body areas (feet and ankles in ambulatory patients; sacral area in patients restricted to bed rest), or may be limited to a single extremity or organ (such as pulmonary edema). REF: pg. 48
OBJ: 1
28. Which of the following conditions is often associated with right heart failure? a. Organomegaly b. Hepatomegaly c. Loud A2 heart sound d. Shallow breathing ANS: B
As right heart failure worsens, dependent edema is no longer relieved by rest or changing position, and edema occurs in the abdominal organs, as well as the extremities and dependent areas of the body. As the liver becomes enlarged (hepatomegaly) as a result of edema, the patient may complain of pain just below the ribs on the right side (right upper quadrant pain). REF: pg. 48
OBJ: 1
29. What is the most common manifestation of infection in a patient with pulmonary disease? a. Fever b. Cough c. Diaphoresis d. Increased sputum production ANS: A
Fever is the most common manifestation of infection. It usually is assumed to be caused by an infectious process until proven otherwise. REF: pg. 49
OBJ: 1
30. The presence of early morning headache may be caused by which of the following
conditions? a. Hypoglycemia b. Anoxia c. Hypoxemia d. Hypercapnia ANS: D
Because hypercapnia worsens during sleep in patients with pulmonary disorders, early morning headaches may be the first indication that the patient is retaining abnormally high amounts of carbon dioxide. If hypercapnia persists, headaches may be present throughout the day. REF: pg. 51
OBJ: 1
31. The peak incidence of snoring in adult males occurs at ages a. 60 to 64 b. 50 to 59 c. 51 to 55 d. 45 to 50
years.
ANS: B
The peak incidence of snoring occurs at ages 50 to 59 years in males and at ages 60 to 64 years in women. REF: pg. 52
OBJ: 1
32. Gastroesophageal reflux disease (GERD) is defined as reflux that occurs more than: a. Twice a month. b. Once a month. c. Twice a week. d. Once a week. ANS: C
Persistent reflux that occurs more than twice a week is often referred to as GERD. REF: pg. 53
OBJ: 1
33. You are a respiratory therapist working in a pediatrician’s office. You see a 16-year-old girl
who presents with two chief complaints but otherwise appears to be healthy. The complaints are (1) extremely heavy menstrual bleeding and (2) severe dyspnea following relatively mild exercise. The dyspnea has been getting progressively worse over the past 3 months. Which of the following laboratory studies would you suggest to the physician in order to determine the cause of the dyspnea? a. Pulmonary function studies b. Hemoglobin and hematocrit studies c. Sputum culture and sensitivity studies d. Chest x-ray ANS: B
Dyspnea during exercise in an otherwise healthy individual may be caused by extreme anemia (hemoglobin <7g/dL). In this case, anemia would likely be associated with the extremely heavy menstrual bleeding. REF: pg. 42
OBJ: 1
34. A sharp, “viselike” chest pain radiating usually down the left arm and possibly spreading to
the shoulders, neck, and jaw is called: a. Pleuritic pain. b. Thoracic pain.
c. Angina. d. Epigastric pain. ANS: C
Angina is the pain described and is an indication of myocardial ischemia. It often precedes a myocardial infarction. REF: pg. 44
OBJ: 1
35. A patient comes to the emergency department following an episode of unexpected fainting
(syncope). In order to determine the potential cause, all of the following could contribute valuable information except: a. A sensation of an irregular heartbeat or history of cardiac arrhythmias. b. The medications taken. c. The activities at the time of the episode. d. The time of day. ANS: D
It is unlikely that the time of day will have any bearing on the cause of a syncopal episode, especially when factors such as arrhythmias present, medications being taken, and activities being undertaken at the time of the episode are taken into account. REF: pg. 47
OBJ: 1
36. A patient presents at the physician’s office with a complaint of severe, aching pain in her right
leg. An examination of the patient’s lower body reveals that the right ankle is badly bruised and severely swollen. The patient states that she fell 2 days ago and “sprained” her ankle but did not seek medical attentT ioEnSaT t tB haAtNtiK mS e.ESLhL eE haRs.bC eeOnMmostly either sitting or lying down since the accident. Her leg pain and swelling could be due to: a. Her sprained ankle. b. Deep vein thrombosis (DVT) in her right leg. c. A side effect from the bruising. d. All of the above. ANS: D
Any soft tissue, bone, or joint injury in the leg will lead to swelling in the area surrounding the injury. In addition, venous stasis caused by the inactivity as well as bruising can lead to formation of DVT. This is especially dangerous because a deep vein thrombus can break loose and cause a pulmonary embolus, a potentially fatal condition. REF: pg. 48
OBJ: 1
37. A patient is being evaluated for surgery. The therapist hears rales in the lung bases and notes
prominent swelling of the ankles. She presses a finger into the edema and notices that a depression remains when she lifts her finger away. The depression is clearly visible for slightly more than 1 minute. This would be classified as pitting edema: a. 1+. b. 2+. c. 3+. d. 4+. ANS: C
Pitting edema that remains visible for 1 to 2 minutes is classified as 3+ pitting edema. REF: Table 3-12, pg. 49
OBJ: 1
38. A patient who appears emaciated presents to the emergency department with a history of
low-grade, intermittent, afternoon fevers; frequent, paroxysmal bouts of coughing; hemoptysis; and night sweats. The following should be considered as part of the differential diagnosis (a list of possible disease states that could account for the observed signs and symptoms): a. Tuberculosis b. Pneumonia c. Some types of cancers (e.g., lymphoma) d. All of the above ANS: D
Choices a, b, and c could result in the observed symptoms. However, the most likely choice is tuberculosis. REF: pg. 36
OBJ: 1
39. Snoring is caused by all of the following except: a. Excessive daytime sleepiness. b. Obesity. c. Excessive alcohol consumption. d. Use of sleeping medications. ANS: A
Excessive daytime sleepinT esE sS isTaBreAsN ulK t oSfEoL bsLtrEuR ct. ivC eO slM eep apnea and is associated with snoring; however, it does not cause snoring. Snoring is due primarily to narrowing of the hypopharynx. This can result from extra tissue in the neck that accompanies obesity, as well as excessive relaxation of the muscles surrounding the hypopharynx caused by alcohol intoxication or sleeping medications. REF: pg. 52
OBJ: 1
40. Gastroesophageal reflux is caused by: a. High activity levels in gastric muscles. b. Blockage of the duodenum. c. Abnormal opening of the pyloric sphincter. d. Abnormal opening of the cardiac (lower esophageal) sphincter. ANS: D
Inappropriate opening of the cardiac sphincter allows stomach contents to move into the esophagus, resulting in heartburn. Gastric contents reaching the larynx may be aspirated and cause symptoms of gastroesophageal reflux disease (GERD). REF: pg. 53
OBJ: 1
Chapter 4: Vital Signs Test Bank MULTIPLE CHOICE 1. Which of the following is not classified as a classic vital sign? a. Pulse b. Sensorium c. Blood pressure d. Respiratory rate ANS: B
The four classic vital sign measurements are temperature, pulse, respirations, and blood pressure. REF: pg. 57
OBJ: 1
2. How often should vital signs be recorded in a patient who is hospitalized but is not in
intensive care? a. Every hour b. Every 2 hours c. Every 4 to 6 hours d. Every 12 hours ANS: C
For patients who are hospitalized but are not in intensive care, routine vital signs are recorded most commonly every 4 toT6EhSoT urBs A (aNt K thS eE beLgL inEnR in. gC ofOaMshift and in the middle of a shift). REF: pg. 58
OBJ: 1
3. Which of the following statements regarding vital signs is most accurate? a. Vital signs are used only to establish a baseline. b. Vital signs monitor only the response to therapy. c. Trends in vital signs are far more important than a single measurement. d. Vital signs at baseline should never be compared with those that were obtained 24
hours earlier. ANS: C
The initial reading generally is referred to as the baseline measurement. A series of vital sign measurements over time establishes a trend and is far more important clinically than any single measurement. Each time vital signs are measured, they should be compared with baseline values and the most recent measurements. Sometimes the patient’s condition may be changing slowly, and comparison with one or two previous measurements does not indicate a trend, whereas comparison over the span of an entire shift or 24-hour period may indicate clearly that the patient is deteriorating slowly. REF: pg. 58
OBJ: 2
4. Which of the following statements best reflects the importance of comparing changes in vital
signs?
a. b. c. d.
It allows assessment of important subjective patient data. It allows recognition of the development of a particular problem. It allows detection of additional symptoms. It does not allow changes in therapy.
ANS: B
Information about the probability that a patient is experiencing or may be developing a particular problem is obtained by comparing changes in vital signs and other signs and symptoms. REF: pgs. 58-59
OBJ: 2
5. When vital signs are compared with other signs and symptoms to arrive at a conclusion about
what is wrong with a patient, this is known as the: a. review of systems. b. differential diagnosis. c. general clinical presentation. d. objective assessment information. ANS: B
In the field of medicine, this comparison of multiple signs and symptoms to arrive at the patient’s diagnosis is called a differential diagnosis. REF: pg. 59
OBJ: 2
6. If dehydration is suspected in a patient, which of the following parameters should be
monitored often? a. Height b. Weight c. Fluid intake and output d. Blood pressure ANS: C
If a question arises regarding dehydration or fluid overload, fluid intake and output (I & O) and weight may be recorded during each shift until the patient’s fluid balance is stable. REF: pg. 59
OBJ: 2
7. A patient’s general clinical presentation indicates to the respiratory therapist (RT) that the
patient is in distress. The RT’s first step should be to: a. call a code and stand by to assist with cardiopulmonary resuscitation (CPR). b. do a complete physical examination to determine the precise cause of the distress. c. evaluate the problem quickly and intervene or locate someone to assist the patient. d. follow the procedure learned in class to introduce himself or herself to the patient and ask the patient what is wrong. ANS: C
If the patient is in distress, the priorities are to evaluate the problem in the most efficient and rapid way possible and to intervene or locate someone who can assist the patient. REF: pg. 59
OBJ: 2
8. A well-written description of an RT’s initial observations is beneficial to other healthcare
providers because it: a. shows that the RT has established rapport with the patient. b. helps them know how to plan care and relate to the patient’s needs. c. decreases the length of hospitalization of the patient and keeps costs down. d. allows them to read the information quickly without confusion and misunderstandings. ANS: B
A written description of these initial observations helps others involved in the patient’s care to know how to plan care and relate to the patient’s needs. REF: pg. 60
OBJ: 2
9. Assessing the patient’s level of consciousness is important because it: a. suggests the types of questions that the therapist should ask. b. evaluates the adequacy of cerebral perfusion and oxygenation. c. indicates whether the patient will take an active role in his or her treatment. d. documents the patient’s mental status at the time of the physical examination. ANS: B
Adequate cerebral oxygenation must be present for the patient to be conscious, alert, and well oriented. REF: pg. 61
OBJ: 3
10. Which of the following scales is the gold standard for assessing trends in the neurologic
function of patients who haTvE eS suTfB feA reN d KhS eaE dLtrL auEmRa.?COM a. Murray scale b. PRISM c. APACHE d. Glasgow scale ANS: D
The Glasgow coma scale has become the gold standard for assessing trends in the neurologic function of patients who have been sedated, have received anesthesia, have suffered head trauma, or are near coma. REF: pg. 61
OBJ: 3
11. The normal range for oral body temperature in most people is a. 97; 99.5 b. 96.7; 98.5 c. 98.7; 100.5 d. 97.7; 98.5
to
° F.
ANS: A
Normal body temperature for most persons is approximately 98.6° F (37° C), with a normal range from 97 to 99.5° F and daily variations of 1° to 2° F. REF: pg. 61
OBJ: 3
12. Fever is defined as an: a. infection in the body. b. elevation of body temperature above normal. c. increase in the oxygen requirements of the body. d. elevation of body temperature above normal because of disease. ANS: D
When the body temperature is elevated from disease, this elevation is called fever, and the patient is said to be febrile. REF: pg. 62
OBJ: 3
13. What is the most common cause of fever above 102° F? a. Aspiration pneumonitis b. Infection c. Blood transfusion reaction d. Head injury ANS: B
Infection is most likely to be the cause of fever when the body temperature exceeds 102° F. REF: pg. 62
OBJ: 3
14. For every elevation in body temperature of 1° C, oxygen consumption and carbon dioxide
production increase by approximately a. 2 b. 5 c. 10 d. 20
%.
ANS: C
For every elevation in body temperature of 1° C, oxygen consumption and carbon dioxide production increase by approximately 10%. REF: pg. 62
OBJ: 3
15. Which of the following conditions causes hypothermia? a. Damage to the hypothalamus b. Excessive shivering and vasoconstriction c. Exposure to high environmental temperatures d. Invasion of the body by microorganisms that produce an infection ANS: A
Hypothermia is not common but can occur in persons with a severe head injury that damages the hypothalamus and in those suffering from exposure to cold environmental temperatures. REF: pg. 62
OBJ: 3
16. Which of the following is not considered a typical site for temperature measurement? a. Oral b. Rectal c. Axillary
d. Fingertip ANS: D
Body temperature is measured most often at one of four sites: the mouth, ear, axilla, or rectum. REF: pg. 62
OBJ: 4
17. What site is used to measure body temperature in comatose patients? a. Oral b. Axillary c. Rectal d. Ear ANS: C
Rectal temperatures may be used for patients who are comatose, in intensive care, or confused. REF: pg. 63
OBJ: 4
18. Which of the following is the method of choice for measuring temperature in neonates? a. Oral b. Rectal c. Axillary d. Tympanic ANS: C
Axillary is the method of choice for neonates because it approximates their core temperature and avoids injury to the rectal tissues. REF: pg. 63
OBJ: 4
19. Which of the following statements is true regarding oral temperature measurements? a. They are the most convenient to obtain and are most readily accepted by alert
adults. b. They are useful in infants. c. They are acceptable in orally intubated patients. d. They are affected by delivery of oxygen by nasal cannula. ANS: A
Oral temperature measurement remains a common, convenient, and acceptable method for awake adult patients. REF: pg. 63
OBJ: 4
20. What is the main advantage of tympanic thermometry over the classic forms of temperature
measurement? a. It is fast, clean, and noninvasive. b. Multiple studies have shown a high correlation with core temperature. c. It is the preferred method of temperature measurement in the pediatric population. d. It uses infrared emissions from the surface of the tympanic membrane and is not in contact with skin or other tissue.
ANS: A
This method has the advantage of being fast, clean, and noninvasive and avoids the embarrassment and time delays associated with the classic forms of temperature measurement. REF: pg. 63
OBJ: 4
21. Which of the following reasons has been listed as the number one concern regarding use of
the tympanic site for body temperature measurement? a. Speed b. Safety c. Accuracy d. Cost ANS: C
Concerns have been expressed about its accuracy and use in the hospital setting. REF: pg. 63
OBJ: 4
22. The normal pulse rate range for adults is a. 60 to 100 b. 70 to 110 c. 80 to 120 d. 90 to 160
beats/min.
ANS: A
The normal pulse rate for adults is 60 to 100 beats/min and is regular in rhythm. REF: pg. 64
OBJ: 5
23. A pulse rate above the normal range is the definition of which of the following terms? a. Tachypnea b. Arrhythmia c. Bradycardia d. Tachycardia ANS: D
A pulse rate exceeding 100 beats/min in an adult is termed tachycardia. REF: pg. 64
OBJ: 6
24. What is an important reason to monitor the heart rate in patients with lung disease? a. A heart rate above or below the normal range is dangerous. b. As the heart rate increases, the tissue’s demand for oxygen decreases. c. The more efficiently the heart is pumping, the lower is the resting pulse rate. d. Tachycardia is a common finding when hypoxemia is present. ANS: D
When the oxygen content of arterial blood falls below normal, usually from lung disease, the heart tries to compensate by increasing cardiac output to maintain adequate oxygen delivery to the tissues. An increase in cardiac output is attained by an increase in heart rate in most persons. For this reason, it is important to monitor the heart rate in patients with lung disease.
REF: pg. 64
OBJ: 5
25. Which of the following characteristics should be included in the assessment of a patient’s
pulse? a. Age, rate, and strength b. Rate, rhythm, and strength c. Age, rhythm, and status d. Strength, amplitude, and volume ANS: B
The rhythm and strength of the pulse are evaluated, the pulse rate is evaluated, and then the pulse rate is counted. REF: pg. 64
OBJ: 5
26. Which of the following arteries is most appropriate for measuring the pulse of a patient with
very low blood pressure? a. Radial artery b. Popliteal artery c. Brachial artery d. Femoral artery ANS: D
When the blood pressure is abnormally low, the more centrally located pulses, such as the carotid pulse in the neck and femoral pulses in the groin, can be identified more easily than the peripheral pulse. REF: pg. 64
OBJ: 7
27. Which of the following terms is used to describe a significant decrease in pulse pressure
during spontaneous inspiration? a. Pulsus alternans b. Pulsus paradoxus c. Pulsus respiratory d. Pulsus asymmetrical ANS: B
When the patient’s pulse strength decreases with spontaneous inhalation, this is referred to as pulsus paradoxus. REF: pg. 65
OBJ: 6
28. An alternating succession of strong and weak pulses that usually is not related to respiratory
disease is known as pulsus: a. tardus. b. alternans. c. paradoxus. d. asymmetrical. ANS: B
Pulsus alternans is an alternating succession of strong and weak pulses that usually is not related to respiratory disease. REF: pg. 65
OBJ: 6
29. A respiratory rate of greater than a. 60 b. 50 c. 40 d. 30
breaths/min is considered abnormal at any age.
ANS: A
A respiratory rate of 40 breaths/min is unusual for an adult, and a rate greater than 60 breaths/min is abnormal at any age. REF: pg. 65
OBJ: 3
30. In postoperative patients, the degree of tachypnea typically is related to the degree of: a. anesthetic received during surgery. b. pain medication received. c. pneumonia. d. atelectasis. ANS: D
Tachypnea in the postoperative patient is common when significant fever develops or when the lungs partially collapse (atelectasis) as a side effect of surgery. Atelectasis causes the lungs to become stiffer than normal, and the patient adopts a breathing pattern that is made up of rapid and shallow breathTsE ,w esEaL sL aE coRm.pCenOsM atory mechanism. The degree of ShTicBhAsNerKvS atelectasis determines the degree of tachypnea in such cases. REF: pg. 65
OBJ: 6
31. Which of the following breathing patterns describes Kussmaul breathing? a. Normal b. Shallow c. Fast and shallow d. Fast and deep ANS: D
In the case of ketoacidosis resulting from uncontrolled diabetes, a fast and deep breathing pattern known as Kussmaul breathing is often evident. REF: pg. 65
OBJ: 6
32. Bradypnea may be caused by which of the following? a. Hypothermia b. Narcotic overdose c. Head injury d. All of the above ANS: D
A slow respiratory rate, referred to as bradypnea, is uncommon but may occur in patients with head injury or hypothermia, as a side effect of certain medications such as narcotics, and in patients with drug overdose. REF: pg. 65
OBJ: 6
33. Which of the following techniques is not useful in measuring the respiratory rate in patients
who are spontaneously breathing and alert? a. Ask the patient to breathe normally. b. Lay the hand you are using to count the patient’s pulse on the patient’s abdomen, and watch abdominal movements. c. Count the rate for 30 seconds and multiply by 2. d. Pretend to be counting the pulse while counting the respiratory rate. ANS: A
Never ask the patient to “breathe normally” while you are assessing the rate of respiration. When individuals think about their breathing, they often voluntarily change their breathing rate and pattern. REF: pg. 66
OBJ: 8
34. The diastolic blood pressure is a reflection of: a. the pressure exerted during left ventricular relaxation. b. the peak pressure exerted during left ventricular contraction. c. the difference between systolic and diastolic pressures. d. the force exerted against the walls of the arteries as blood flows through them. ANS: A
Diastolic blood pressure is the force that occurs when the heart is relaxed. REF: pg. 66
OBJ: 8
35. Which of the following is not true about pulse pressure? a. It normally is between 35 and 40 mm Hg. b. It is calculated from the 12-lead electrocardiographic printout. c. It is the difference between the systolic and diastolic pressures. d. The peripheral pulse is difficult to detect when the pulse pressure falls to below 30
mm Hg. ANS: B
Pulse pressure is the difference between the systolic and diastolic pressures. The normal pulse pressure is 35 to 40 mm Hg. When the pulse pressure is less than 30 mm Hg, the peripheral pulse is difficult to detect. Patients with heart failure and inadequate stroke volume usually have a reduced pulse pressure. REF: pg. 66
OBJ: 7
36. Which of the following organs is not at risk for disease in the presence of systemic
hypertension? a. Heart b. Kidneys c. Blood vessels
d. Pancreas ANS: D
Persistent pressures at the level of systemic hypertension are associated with an escalating risk for the development of heart, vascular, and renal diseases. REF: pg. 66
OBJ: 9
37. Which of the following is not a cause of hypotension? a. Left ventricular failure b. Peripheral vasodilation c. Hypovolemia d. Right heart failure ANS: D
Hypotension may occur as the result of peripheral vasodilation, left ventricular failure, or low blood volume. REF: pg. 67
OBJ: 9
38. Hypotension associated with changes in posture in patients with hypovolemia is known as a. b. c. d.
hypotension. persistent orthostatic hypovolemic peristaltic
ANS: B
Changes in posture may produce abrupt changes in arterial blood pressure, especially in the hypovolemic patient. Normally, when the patient moves from the supine to the sitting position, the blood pressure changes very little, but when hypovolemia or vasodilation is present, the blood pressure may fall significantly; this is referred to as postural (or orthostatic) hypotension. REF: pg. 67
OBJ: 9
39. When the blood pressure is measured, the sounds that are heard through the stethoscope
between the systolic and diastolic pressures are known as a. Meckel b. brachial c. Korotkoff d. succussion
sounds.
ANS: C
The auscultatory method of pressure measurement uses the stethoscope to listen for the sounds produced by arterial pulse waves (Korotkoff sounds) when blood flow in the artery resumes. As the pressure is reduced during deflation of the occluding cuff, the Korotkoff sounds change in quality and intensity. REF: pg. 68
OBJ: 8
40. What is the most probable cause for the significant decrease in blood pressure that occurs
during inhalation in some patients? a. Hypovolemia b. Variable systemic vascular resistance c. Early left ventricular failure caused by coronary artery insufficiency d. Negative intrathoracic pressure that diminishes blood flow out of the left ventricle ANS: D
This decrease in systolic blood pressure is more significant during a forced maximal inhalation. When the systolic pressure drops by more than 10 mm Hg during inhalation at rest, a definite abnormality exists; this is termed paradoxic pulse. Paradoxic pulse, also called pulsus paradoxus, occurs in various circulatory and respiratory conditions such as asthma and cardiac tamponade. The most probable mechanism responsible for this fluctuation in blood pressure centers around the negative intrathoracic pressure created by the respiratory muscles during inhalation. REF: pg. 70
OBJ: 11
41. Which of the following conditions is not associated with the presence of pulsus paradoxus? a. Hypervolemia b. Cardiac tamponade c. Constrictive pericarditis d. Asthma ANS: A
Pulsus paradoxus is commonly seen in patients with restrictions around the heart, such as cardiac tamponade, constrictive pericarditis, or restrictive cardiomyopathy. It also may occur TE in patients with severe pulm onSaT ryBdAisNeaKsS esE, L suLchER as.aC cuOteMasthma. REF: pg. 70
OBJ: 11
42. A 42-year-old male patient with a history of poorly controlled asthma is seen in the
emergency department by the respiratory therapist (RT). The patient is seated in a chair and clearly is in extreme respiratory distress, with inspiratory and expiratory wheezing throughout. He reports that he has had a fever for the past 2 days and has eaten and drunk very little. He also reports some chest pain, and his blood pressure is 95/55 mm Hg. At one point, the patient stands to move from the chair to the stretcher and has an episode of syncope. The differential diagnosis for this patient should include all of the following except: a. Myocardial infarction (MI). b. Acute asthma exacerbation. c. Dehydration. d. Cerebrovascular accident (CVA; stroke). ANS: D
The chest pain, although minor, requires evaluation for MI. The wheezing and respiratory distress strongly suggest asthma exacerbation, and the combination of syncope, restricted fluid restriction, and low blood pressure strongly suggest hypovolemia. No signs or symptoms of CVA are present. REF: pg. 70
OBJ: 9
43. An RT attempts to take the blood pressure of the patient in question 42. The RT notes that the
pulse intensity determined from palpation of the radial artery decreases during inspiration. The most likely explanation for this would be: a. an (MI). b. an acute asthma exacerbation. c. dehydration. d. a cerebrovascular accident (CVA; stroke). ANS: B
Because of the high airway resistance caused by the asthma exacerbation, the intrathoracic pressure decreases abnormally during patient inspiration. This results in decreased cardiac filling, decreased blood pressure, and decreased pulse intensity during inspiration. This is an example of pulsus paradoxus. REF: pg. 70
OBJ: 11
44. An RT is called to the emergency department on New Year ’s Eve to assist in the treatment of
a 5-year-old child pulled from under the ice of a local pond. When the child arrives, he has been intubated and has a weak, irregularly irregular pulse of 55 beats/min, no spontaneous respiratory effort, and a body temperature of 30o C. The normal practice in this hospital is to use heat–moisture exchangers (HMEs) for all mechanically ventilated patients. Which of the following should the therapist suggest for the ventilator setup? a. Greater than normal rate and tidal volume b. Higher than usual inspiratory flow rate c. Heated humidity in place of the HME d. All of the above ANS: C
This patient is clearly hypothermic; therefore, along with other warming measures, heated, humidified air will significantly aid in raising the patient’s body temperature. Normally, ventilator settings for these patients require lower minute ventilation than usual. An increased rate and tidal volume would result in increased minute ventilation, whereas an increased inspiratory flow rate would result in an elevated peak inspiratory pressure. REF: pg. 62
OBJ: 3
45. An RT has just finished administering an aerosol treatment by a small-volume nebulizer to a
patient who has a serious case of pneumonia. For the past 24 hours, the patient has had a fever of 39 to 40o C. Just as the therapist is finishing, a nursing aide comes in to record the vital signs. She uses an electronic thermometer to record an oral temperature of 36.5o C and comments to the patient, “Well, Mr. Jones, you must be getting better because your fever is completely gone.” The therapist should: a. also verbally encourage the patient by saying that he is getting better. b. totally ignore the situation and continue with the posttreatment assessment. c. complete the treatment and assessment, record the results in the patient’s chart, then go on to the next patient. d. go to the nursing aide after she leaves the room and tell her that the temperature she recorded is probably not accurate and should be repeated. ANS: D
A small-volume nebulizer produces an aerosol mist that may temporarily cool the inside of the mouth. Hence, an oral temperature should not be measured for at least 10 to 15 minutes following the completion of the nebulizer treatment. REF: pg. 63
OBJ: 3
46. An RT is taking the blood pressure in the arm of a 12-year-old girl. The only
sphygmomanometer available has a cuff in which the ends overlap slightly when it is inflated. The pressure that the RT records: a. should be accurate if the measurement is otherwise done properly. b. is likely to be in error, showing a value that is too low. c. is likely to be in error, showing a value that is too high. d. will have a value that cannot be predicted to be either too high or too low. ANS: A
When measuring the blood pressure in a child, it is permissible to use a cuff where the ends overlap slightly. REF: pg. 68
OBJ: 10
Chapter 5: Fundamentals of Physical Examination Test Bank MULTIPLE CHOICE 1. All of the following are components of the physical examination except: a. inspection. b. auscultation. c. manipulation. d. percussion. ANS: C
The four components of the physical examination are inspection, palpation, percussion, and auscultation. REF: pg. 75
OBJ: 1
2. A review of the patient’s history of present illness and past clinical history prior to a physical
examination is helpful in that it: a. gives insight into the expected findings during the examination. b. suggests examination techniques to emphasize. c. helps establish a rapport with the patient. d. all of the above. ANS: D
A review of the history helps direct the course of the physical examination, allows the medical care team to begin to estabT liE shSrT apBpA orNt K wS ithEL thL eE paRti. enCt,OaMnd may suggest the success or failure of past treatments. REF: pg. 76
OBJ: 2
3. What problem is associated with cyanosis of the oral mucosa? a. Low cardiac output b. Reduced arterial oxygenation c. Hyperventilation d. Hypoventilation ANS: B
Cyanosis of the oral mucosa is a sign of central cyanosis that occurs only with severe hypoxemia. REF: pg. 76
OBJ: 3
4. What problem is associated with nasal flaring? a. Hypoxemia b. Increased lung compliance c. Increased work of breathing d. Increased dead space ventilation ANS: C
Nasal flaring most often occurs in infants and children who are working very hard to breathe. Nasal flaring usually occurs when the lungs stiffen (e.g., with pneumonia) or when the airways narrow (e.g., with asthma). REF: pg. 76
OBJ: 3
5. Which of the following is not associated with dilated and unresponsive pupils? a. Opiates b. Atropine c. Brain death d. Catecholamines ANS: A
Atropine, brain hypoxia, and catecholamines can cause the pupils to become fixed and unresponsive to light. Opiates have no such effect. REF: pg. 76
OBJ: 3
6. Which of the following terms is used to describe constriction of the pupil of the eye? a. Lytosis b. Miosis c. Mydriasis d. Nystagmus ANS: B
Miosis refers to constriction of the pupils. REF: pg. 76
OBJ: T3ESTBANKSELLER.COM
7. Which of the following terms is used to describe drooping of the eyelids? a. Ptosis b. Miosis c. Diplopia d. Chondrosis ANS: A
Ptosis is defined as drooping of the eyelids. REF: pg. 77
OBJ: 3
8. Which of the following physical examination findings is associated with myasthenia gravis? a. Ptosis b. Miosis c. Mydriasis d. Hepatomegaly ANS: A
Patients with myasthenia gravis often develop ptosis as part of the clinical picture. REF: pg. 77
OBJ: 3
9. Which of the following terms is used to describe double vision?
a. b. c. d.
Miosis Diplopia Nystagmus None of the above
ANS: B
Diplopia refers to double vision. REF: pg. 77
OBJ: 3
10. Which of the following is most likely to cause tracheal deviation? a. Left upper lobe pneumonia b. Right upper lobe pneumothorax c. Left lower lobe pleural effusion d. Right lower lobe pleural effusion ANS: B
Upper lobe changes are more likely to influence tracheal position than are lower lobe problems. Pneumothorax usually causes a volume change in the upper lobe and alters the position of the trachea. REF: pg. 77
OBJ: 3
11. Which of the following is the most common cause of jugular venous distention? a. Auto-positive end-expiratory pressure (auto-PEEP) b. Pneumonia c. Pneumothorax d. Right heart failure ANS: D
Right heart failure causes a backup of venous blood into the venous system. This causes the neck veins to distend. REF: pg. 78
OBJ: 3
12. At what angle should the head of the bed be elevated to assess jugular vein pressure (JVP)? a. 15 degrees b. 30 degrees c. 45 degrees d. 90 degrees ANS: C
The current standardized procedure for checking the JVP at the bedside is to elevate the head of the bed to 45 degrees. If the head of the bed were placed at 90 degrees, gravity would make it difficult to see the JVP. If the head of the bed were placed at less than 45 degrees, venous blood would tend to pool in the neck, which would cause most patients to appear to have JVP. REF: pg. 77
OBJ: 4
13. In a patient’s chart, the physician has documented finding tender lymph nodes in the neck
region during the physical examination. This problem is consistent with: a. respiratory infection.
b. human immunodeficiency virus (HIV) infection. c. lymphoma. d. lung cancer. ANS: A
Tender lymph nodes often are associated with infection. REF: pg. 78
OBJ: None
14. The horizontal fissure that separates the right upper lobe from the right middle lobe begins at
what rib in the midsternal line? a. Rib 2 b. Rib 4 c. Rib 6 d. Rib 8 ANS: B
The horizontal fissure is located at the fourth rib at the midsternal line. REF: pg. 79
OBJ: 5
15. At what point on the posterior chest wall can the inferior border of the lung normally be
found? a. Between T4 and T6 b. Between T6 and T8 c. Between T9 and T12 d. None of the above ANS: C
Normally, the inferior border of the lung is located between T9 and T12 on the posterior chest. REF: pg. 80
OBJ: 5
16. Which of the following terms describes an abnormal lateral curvature of the spine? a. Kyphosis b. Scoliosis c. Anhidrosis d. Myelosis ANS: B
The term for lateral curvature of the spine is scoliosis. Kyphosis refers to an abnormal anteroposterior curvature of the spine. REF: pg. 80
OBJ: 6
17. Which of the following disorders is associated with a barrel chest? a. Chronic bronchitis b. Pneumonia c. Emphysema d. Chest trauma ANS: C
Patients with emphysema lose their lung recoil. This allows the ribs to straighten somewhat, causing the patient’s anteroposterior diameter to increase significantly. REF: pg. 80
OBJ: 6
18. Which of the following patterns of breathing is associated with a loss in lung volume? a. Slow and deep b. Rapid and deep c. Slow and shallow d. Rapid and shallow ANS: D
A loss of lung volume causes lung compliance to decrease. This makes it more difficult for the patient to breathe at normal tidal volumes. As a result, patients breathe with smaller volumes but at a faster rate. REF: pg. 80
OBJ: 8
19. What disease is most likely to cause a prolonged expiratory time? a. Asthma b. Pneumonia c. Pneumothorax d. Pulmonary fibrosis ANS: A
Asthma causes the intrathoracic airways to become narrow, which makes exhalation slow and prolonged. REF: pg. 80
OBJ: 8
20. Which of the following diseases is most likely to cause a prolonged inspiratory time? a. Chronic obstructive pulmonary disease (COPD) b. Asthma c. Epiglottitis d. Acute bronchitis ANS: C
Epiglottitis causes narrowing of the upper airway. This causes a prolonged inspiratory time because the narrowed upper airway slows inspiratory flow more than expiratory flow. REF: pg. 82
OBJ: 8
21. Which of the following terms describes the sinking inward of the skin overlying the rib cage
with each inspiratory effort? a. Ataxia b. Bulging c. Retractions d. Mydriasis ANS: C
When intrathoracic pressures become extremely negative with inspiration, the skin overlying the chest wall may sink inward between the ribs. This is known as retractions, and it indicates a severe increase in the work of breathing. REF: pg. 82
OBJ: 10
22. A respiratory therapist is examining a patient in respiratory distress. She observes that the
patient has a barrel chest, is using his accessory muscles of inspiration even when sitting at rest, and has a prolonged expiratory phase with pursed-lip breathing. This finding is most consistent with: a. an upper respiratory infection. b. an acute exacerbation of asthma. c. severe COPD. d. none of the above. ANS: C
Patients with COPD tend to use their accessory muscles to breathe when the work of breathing increases and the diaphragm becomes less effective. REF: pg. 82
OBJ: 8
23. Which of the following breathing patterns is associated with fatigue of the diaphragm? a. Bradypnea b. Deep and fast breathing c. Biot breathing d. Abdominal paradox ANS: D
When the diaphragm is fatigued and can no longer assist with breathing, it needs a rest. As a result, the accessory muscles take over, and when they cause negative intrathoracic pressure to occur with inspiration, the abdominal contents tend to be pulled inward and upward. This is known as abdominal paradox because the abdomen is moving in the opposite direction from normal with breathing. REF: pg. 82
OBJ: 11
24. The muscles of inspiration include all of the following except: a. the diaphragm, b. the sternocleidomastoids, c. the scalene muscles. d. the rectus abdominus muscle. ANS: D
The rectus abdominus muscle is an accessory muscle of expiration. REF: pg. 81
OBJ: 9
25. Retractions and bulging are observed: a. when the work of breathing is very high. b. during heavy exercise. c. only in newborns and children. d. primarily in patients with pulmonary fibrosis.
ANS: A
Retractions and bulging are signs of dramatically increased work of breathing. REF: pg. 82
OBJ: 10
26. In patients with severe COPD and hyperinflation, the low, flat diaphragm causes inward
movement of the lateral chest walls with inspiration. What is this called? a. Abdominal paradox b. Biot breathing c. Hoover sign d. Respiratory alternans ANS: C
Inward movement of the lateral chest wall with inspiration in patients with COPD is known as the Hoover sign. REF: pg. 83
OBJ: 11
27. Which of the following clinical conditions is associated with central cyanosis? a. Hypercarbia b. Renal failure c. Poor circulation d. Respiratory failure ANS: D
Central cyanosis is a sign of severe hypoxemia and respiratory failure. REF: pg. 83
2 STBANKSELLER.COM OBJ: T1E
28. Which of the following will cause an increase in vocal fremitus upon palpation? a. COPD b. Obesity c. Pneumonia d. Pneumothorax ANS: C
Pneumonia causes an increase in lung tissue density. This causes vocal fremitus to increase. REF: pg. 83
OBJ: 13
29. Which of the following will cause a decrease in vocal fremitus upon palpation? a. Atelectasis b. Pneumonia c. Lung tumor d. Pneumothorax ANS: D
Pneumothorax puts more air between the outer chest wall and the source of the vocal fremitus and reduces the ability of vibrations to be felt at the surface. REF: pg. 83
OBJ: 13
30. What clinical disorder will cause a unilateral decrease in thoracic expansion? a. COPD b. Lobar pneumonia c. Acute asthma attack d. Neuromuscular disease ANS: B
Unilateral refers to only one side. Lobar pneumonia reduces expansion of the lung on the affected side. REF: pg. 84
OBJ: 14
31. A respiratory therapist (RT) examining a patient notes that on assessment of chest expansion
with palpation, the right lower chest is found to lag behind the left lower chest. With which of the following problems is this finding consistent? a. COPD b. Right lower lobe pneumonia c. Left lower lobe pneumonia d. Severe asthma ANS: B
Pneumonia reduces the ability of the lung to expand normally. REF: pg. 84
OBJ: 14
32. A therapist examining a patient encounters an area of skin along the right midaxillary line,
which, when palpated, produces a crackling sound and sensation. This is an indication of in this paTtiEenStT . BANKSELLER.COM a. previous scarring due to a burn injury b. a collection of fluid under the skin c. lymphadenopathy d. subcutaneous emphysema ANS: D
The sensation and sound of crackling just under the skin is an indication of air trapped in pockets in the subcutaneous tissue, which is known as subcutaneous emphysema. REF: pg. 85
OBJ: 15
33. Which of the following clinical disorders will cause increased resonance to percussion? a. Atelectasis b. Pneumonia c. Pneumothorax d. Pleural effusion ANS: C
Air in the pleural space will cause an increase in the resonance to percussion. REF: pg. 86
OBJ: 16
34. Which of the following clinical disorders will cause decreased resonance to percussion? a. COPD
b. Asthma c. Pneumonia d. Pneumothorax ANS: C
Consolidation of the lung will cause a decrease in resonance to percussion. REF: pg. 86
OBJ: 16
35. The bell portion of the stethoscope is best designed to listen to a. low; lung b. low; heart c. high; lung d. high; heart
-pitched
sounds.
ANS: B
The bell portion of the stethoscope is designed to capture low-pitched heart sounds. REF: pg. 86
OBJ: 17
36. Which of the following techniques is most useful for application of the bell portion of the
stethoscope? a. Press it lightly against the chest wall. b. Press it firmly against the chest wall. c. Hold it about 1 cm from the chest wall. d. Slide it gently in a circular motion on the chest wall. ANS: A
The bell is to be placed lightly against the chest wall. If it is placed firmly against the chest wall, it will cause the underlying skin to act as a diaphragm and filter out certain sounds. REF: pg. 86
OBJ: 18
37. The diaphragm portion of the stethoscope is used to listen for: a. heart murmurs. b. pleural friction rubs. c. low-pitched heart sounds. d. most lung sounds. ANS: D
The diaphragm is most useful for listening to high-pitched sounds. Most lung sounds are relatively high pitched compared with heart sounds. REF: pg. 87
OBJ: 18
38. Where is the best anatomic place to begin auscultation in most patients? a. Over the trachea b. Over the upper lobes c. In the axillary regions d. In the dependent regions ANS: D
Starting auscultation in the lower lobes can be useful for identifying late-inspiratory crackles that would clear with several deep breaths. If the clinician started at the top of the lung and worked downward, the late-inspiratory crackles would be gone by the time the lower lobes were reached. REF: pg. 87
OBJ: 18
39. All of the following are characteristics of breath sounds that should be evaluated whenever
chest auscultation is performed except: a. the breath rate. b. the pitch (vibration frequency). c. the amplitude or intensity (loudness). d. the duration of inspiration versus expiration. ANS: A
Normally, the breath rate is counted while one observes the patient, often in conjunction with determining the heart rate. In addition to the pitch, amplitude, and duration of inspiration versus expiration, the characteristics of the breath sounds should be noted. REF: pg. 87
OBJ: 19
40. Which of the following characteristics does not apply to normal tracheal breath sounds? a. Loud b. Low pitched c. High pitched d. Fairly equal inspiratory and expiratory components ANS: B
Tracheal breath sounds are high pitched. REF: pg. 87
OBJ: 20
41. Which of the following terms is used to describe the normal breath sounds heard around the
sternum on the anterior chest and between the scapulae on the posterior chest? a. Tubular b. Vesicular c. Bronchial d. Bronchovesicular ANS: D
Although rarely assessed, the normal breath sounds heard around the sternum are known as bronchovesicular breath sounds. REF: Table 5-2, pg. 87
OBJ: 20
42. Which of the following descriptions does not apply to normal vesicular breath sounds? a. Soft with low intensity b. Minimal expiratory component c. Heard primarily over the lung parenchyma d. High pitched as compared with tracheal breath sounds ANS: D
Normal breath sounds are low pitched. REF: pg. 87
OBJ: 20
43. Which of the following terms best describes a louder version of a vesicular sound that can be
heard over lung consolidation? a. Tubular b. Tracheal c. Crepitation d. Harsh or bronchial ANS: D
Lung consolidation reduces the ability of the lung to attenuate the turbulent flow sounds created in the larger airways. As a result, harsh or bronchial breath sounds are heard over the lung. REF: pg. 87
OBJ: 20
44. Which of the following terms has had a confusing history with regard to its use and has been
replaced with the term crackles? a. Rales b. Wheezes c. Rhonchi d. Crepitations ANS: A
The term rales has had a long and confusing history. It was used originally by a French physician and is the FrenchTw dB foAr N “rKatStlE e.L ” LER.COM EoSrT REF: pg. 88
OBJ: 21
45. Which of the following terms best describes continuous types of adventitious lung sounds? a. Rhonchi b. Stridor c. Wheezes d. Sibilant rales ANS: C
Continuous adventitious lung sounds are best described as wheezes. REF: pg. 89
OBJ: 21
46. Which of the following terms is used to describe a continuous type of adventitious lung sound
heard loudest over the neck of a patient with upper airway obstruction? a. Wheezes b. Stridor c. Rhonchi d. Sonorous rales ANS: B
The term stridor is reserved for describing a low-pitched, monophonic wheeze that is heard loudest over the neck.
REF: pg. 90
OBJ: 21
47. Which of the following mechanisms is believed to be responsible for the normal vesicular
breath sound? a. Air entering the alveoli b. Contraction of the inspiratory muscles c. Rubbing together of the two layers of the pleurae d. Filtration of turbulent flow sounds in the larger airways ANS: D
Normal lung tissue is known to act as a filter for sound. This attenuation causes the turbulent flow sounds of the larger airways to be converted to softer, vesicular breath sounds. REF: pg. 90
OBJ: 22
48. Which of the following statements is not true about vesicular or normal breath sounds? a. They have a minimal expiratory sound. b. They are produced somewhat regionally. c. They are produced when air enters the alveoli. d. They are used to help confirm proper placement of an endotracheal tube. ANS: C
Air entering the alveoli does so by diffusion, which is silent. REF: pg. 90
OBJ: 20
49. Which of the following lunTgEcS onTdBitA ioN nsKiSs E asLsoLcE iaRte.dCwOitM h diminished breath sounds because
of airflow limitation and poor transmission of sound through the lung? a. Bronchitis b. Pneumonia c. Lung tumor d. Emphysema ANS: D
Emphysema causes extreme hyperinflation of the lung and obstruction of the bronchi. These two factors contribute to reduced sound production and transmission and, therefore, very diminished breath sounds. REF: pg. 90
OBJ: 22
50. Which of the following factors most closely relates to the pitch of a wheeze? a. The patient’s inspiratory-to-expiratory (I:E) ratio b. The patient’s respiratory rate c. The length of the involved airway d. The degree of airway compression ANS: D
The narrower the airway, the higher the pitch of the wheeze. REF: pg. 91
OBJ: 21
51. Which of the following characteristics of wheezing is least reliable for predicting the degree
of airway obstruction? a. Pitch b. Duration c. Intensity d. Sound frequency ANS: C
The loudness of the wheeze is independent of the degree of airway obstruction. REF: pg. 91
OBJ: 21
52. Which of the following types of wheezing is likely to be heard over the chest of a patient with
a tumor that is partially obstructing a single airway? a. Sibilant wheezing b. Sonorous wheezing c. Polyphonic wheezing d. Monophonic wheezing ANS: D
A tumor that obstructs a single airway will produce a wheeze with a single (monophonic) note. REF: pg. 92
OBJ: 23
53. Which of the following adventitious lung sounds is related most closely to a life-threatening
problem? a. Stridor b. Rhonchi c. Crackles d. Wheezing ANS: A
Stridor represents narrowing of the upper airway. Because humans have only one upper airway, its obstruction can be deadly. REF: pg. 92
OBJ: 23
54. Which of the following adventitious lung sounds is associated most closely with restrictive
types of lung diseases? a. Polyphonic wheezing b. Monophonic wheezing c. Late-inspiratory crackles d. Early-inspiratory crackles ANS: C
Late-inspiratory crackles are produced when distal airways pop open, as occurs in atelectasis and pulmonary fibrosis. These are restrictive lung diseases. REF: pg. 92
OBJ: 23
55. Which of the following adventitious lung sounds is most likely to be heard in a patient with
croup? a. Stridor b. Rhonchi c. Polyphonic wheezes d. Late-inspiratory crackles ANS: A
Croup causes narrowing of the upper airway and, therefore, stridor. REF: pg. 92
OBJ: 23
56. Which of the following terms applies to an abnormal increase in vocal resonance? a. Fremitus b. Egophony c. Bronchophony d. None of the above ANS: C
Bronchophony refers to an abnormal increase in vocal resonance. REF: pg. 93
OBJ: 24
57. Which of the following abnormalities would be associated with an increase in vocal
resonance? a. Pneumonia b. Emphysema c. Pneumothorax d. Large pleural effusion ANS: A
Pneumonia produces an increase in lung tissue density, which causes sound vibrations to travel more rapidly through the lung. REF: pg. 93
OBJ: 24
58. Which of the following terms describes the area of the chest wall that overlies the heart? a. Periosteum b. Precordium c. Pericardium d. None of the above ANS: B
The precordium is the area that overlies the heart on the chest wall where auscultation is performed. REF: pg. 94
OBJ: 25
59. In which area is the point of maximal impulse normally felt during palpation of the chest
wall? a. Midaxillary line at the fifth intercostal space b. Midclavicular line at the fifth intercostal space
c. Just left of the sternum at the third intercostal space d. Just right of the sternum at the fourth intercostal space ANS: B
The point of maximal impulse is created by left ventricular contraction. The left ventricle normally is located at the midclavicular line at the fifth intercostal space. REF: pg. 94
OBJ: 26
60. What may cause the point of maximal impulse to shift to the anterior axillary region on the
left? a. A tension pneumothorax on the right b. Right ventricular hypertrophy c. Cor pulmonale d. Pericarditis ANS: A
A tension pneumothorax will cause a buildup of pressure in the right chest and will push the mediastinal contents to the left. This shifts the point of maximal impulse laterally. REF: pg. 94
OBJ: 26
61. Which of the following clinical disorders is characterized by the point of maximal impulse
being felt in the epigastric area? a. Atelectasis b. Emphysema c. Pneumothorax d. Chronic bronchitis ANS: B
Emphysema causes the diaphragm to be low and flat in the chest. This pulls the heart into a low, centrally located position in the epigastric area. REF: pg. 94
OBJ: 26
62. At which of the following sites on the chest wall is the pulmonic valve best auscultated? a. Second left intercostal space near the sternal border b. Fifth intercostal space on the left near the sternal border c. Sixth intercostal space on the left at the midaxillary line d. First intercostal space on the right at the midclavicular line ANS: A
The pulmonic valve is best heard at the second left intercostal space near the sternal border. REF: pg. 94, Figure 5-25
OBJ: 27
63. What is believed to be responsible for production of the first heart sound? a. Closure of the atrioventricular (AV) valves b. Opening of the AV valves c. Closure of the semilunar valves d. Opening of the semilunar valves ANS: A
Systole causes pressure to build in the ventricles and sudden closure of the AV valves. This is the first heart sound. REF: pg. 94
OBJ: 28
64. Which of the following mechanisms is responsible for production of an abnormal third heart
sound? a. Forceful opening of the aortic valve b. Forceful closure of the pulmonic valve c. Rapid filling of the atrium during ventricular systole d. Rapid filling of the ventricles immediately after systole ANS: D
Rapid filling of the ventricle that is already distended causes an S3 sound. REF: pg. 95
OBJ: 28
65. What clinical problem is associated with a gallop rhythm? a. Left-to-right shunt b. Myocardial infarction sequelae c. Overdistention of the ventricles d. All of the above ANS: D
A gallop rhythm refers to the addition of S3 and/or S4 to the heart sound pattern. This is produced most often with overdistention of the ventricles, as occurs following MI and with shunting in the heart. REF: pg. 95
OBJ: 29
66. Which of the following clinical conditions may cause diminished heart sounds? a. Bronchitis b. Pneumonia c. Emphysema d. Pulmonary embolism ANS: C
Emphysema causes severe hyperinflation of the lungs and surrounds the heart with air, which is a poor transmitter of sounds and causes the heart sounds to sound distant. REF: pg. 94
OBJ: 30
67. What change in the heart sounds is associated with cor pulmonale? a. Loud A2 b. Loud M1 c. Loud T2 d. Loud P2 ANS: D
Cor pulmonale refers to chronic right heart failure due to pulmonary hypertension. Pulmonary hypertension causes a loud P2 sound.
REF: pg. 95
OBJ: 31
68. Which of the following types of murmurs is associated with mitral valve stenosis? a. Systolic b. Diastolic c. Pansystolic d. None of the above ANS: B
A narrowed mitral valve will cause a murmur when blood tries to pass through during diastole. REF: pgs. 95-96
OBJ: 32
69. Which of the following is responsible for the production of murmurs? a. Stenotic AV valves b. Incompetent AV valves c. Incompetent semilunar valves d. All of the above ANS: D
Murmurs are produced by rapid blood flow through a narrowed opening from any cause. REF: pg. 95
OBJ: 32
70. What is a common cause of hepatomegaly? a. Chronic right heart failure b. Pneumothorax c. Pleural effusion d. Acute asthma attack ANS: A
An enlarged liver is commonly produced by chronic right heart failure that causes venous congestion and an elevated central venous pressure (CVP). This reduces the ability of the liver to function properly, and it becomes enlarged. REF: pg. 96
OBJ: 33
71. What is the normal distance that the liver spans in the right upper quadrant of the abdomen? a. 6 cm b. 8 cm c. 10 cm d. 14 cm ANS: C
The normal liver in the adult spans 10 cm or less. REF: pg. 96
OBJ: 33
72. Which of the following terms is used to describe an abnormal collection of fluid in the
peritoneal cavity? a. Ascites
b. Pericarditis c. Pancreatitis d. Hepatomegaly ANS: A
Fluid buildup in the abdomen is known as ascites. REF: pg. 96
OBJ: 33
73. Digital clubbing is associated with: a. COPD. b. Cystic fibrosis. c. Bronchogenic carcinoma. d. All of the above. ANS: D
Digital clubbing is associated with numerous chronic lung diseases. REF: pg. 96
OBJ: 34
74. Which of the following findings is consistent with reduced perfusion? a. Peripheral cyanosis b. Hepatomegaly c. Loud P2 sound d. Jugular vein distention (JVD) ANS: A
Extremities become cool to the touch and turn blue when circulation is poor.
TESTBANKSELLER.COM
REF: pg. 97
OBJ: 34
75. Which of the following clinical conditions is associated with the onset of pedal edema? a. Liver failure b. Right heart failure c. Status asthmaticus d. Tension pneumothorax ANS: B
Right heart failure causes a backup of venous blood flow and increases the hydrostatic pressure of fluids in the dependent regions. This leads to pedal edema. REF: pg. 97
OBJ: 34
76. The length of normal capillary refill is less than a. 1 b. 3 c. 5 d. 10
second(s).
ANS: B
When circulation is good, capillary refill is rapid, taking less than 3 seconds. REF: pg. 96
OBJ: 34
Chapter 6: Neurologic Assessment Test Bank MULTIPLE CHOICE 1. Which of the following is not an important factor in determining the needed frequency of the
neurologic assessment? a. Patient’s diagnosis b. Acuity of the condition c. How rapidly changes are occurring or are expected to occur d. Shift change ANS: D
Once a thorough evaluation is done on admission or at the beginning of each shift, subsequent assessments should be tailored to the patient’s condition. The frequency of these later assessments will depend on the patient’s diagnosis, the acuity of the condition, and how rapidly changes are occurring or are expected to occur. REF: pg. 103
OBJ: 1
2. Which of the following is not part of the brain? a. Cerebrum b. Cranial nerves c. Brainstem d. Cerebellum ANS: B
The brain consists of three parts: the cerebrum, which contains two hemispheres; the brainstem (midbrain, pons, and medulla); and the cerebellum. REF: pg. 103
OBJ: 2
3. Where in the nervous system is breathing regulated? a. Brain cortex b. Medulla c. Cerebellum d. Brainstem ANS: D
Many neurologic functions of particular importance to the respiratory therapist (RT), such as regulation of heart rate, blood pressure, and breathing, are located in the brainstem. REF: pg. 104
OBJ: 4
4. Which of the following is the most common cause of a nerve root pathologic condition due to
compression? a. Herniated vertebral disc b. Spinal tumor c. Spinal cord injury d. Infection
ANS: A
A herniated vertebral disc is the most common nerve root pathologic condition that results in compression on the nerve roots. REF: pg. 106
OBJ: 4
5. Which of the following diseases may be suggested by the involvement of multiple nerve
roots? a. Myasthenia gravis b. Guillain-Barré syndrome c. Brain tumor d. Intracranial hypertension ANS: B
The involvement of multiple nerve roots suggests the presence of inflammatory processes (Guillain-Barré syndrome, amyloidosis, vasculitis), neoplastic processes (carcinomatous meningitis), or infectious processes (syphilis, Lyme disease). REF: pg. 122
OBJ: 4
6. Injury to the cervical spine roots C3 to C5 is associated with which of the following
abnormalities? a. Absence of deep tendon reflexes b. Babinski sign c. Paralysis of the diaphragm d. Doll’s-eye maneuver ANS: C
The phrenic nerves arise from the cervical spine roots of C3 to C5. Damage to this portion of the spinal cord or to both phrenic nerves can result in complete paralysis of the diaphragm and can make the patient dependent on a ventilator for life. REF: pg. 107
OBJ: 8
7. Which of the following is the most important part of the neurologic examination? a. Sensory examination b. Motor examination c. Level of consciousness d. Gait examination ANS: C
Evaluation of the level of consciousness (LOC) and documentation are the most important parts of the neurologic examination. REF: pg. 108
OBJ: 7
8. What instrument is used most widely to quantify neurologic impairment? a. Merck gait evaluation b. Glasgow coma scale c. APACHE d. Mini-Mental State Examination
ANS: B
The Glasgow coma scale (GCS) was published in 1974 by Graham Teasdale and Bryan J. Bennett to assess the LOC after head injury. It is the most widely used instrument for quantifying neurologic impairment. REF: pg. 110
OBJ: 6
9. Which Glasgow coma scale score is typically an indication for endotracheal intubation? a. <9 b. <10 c. <12 d. <14 ANS: A
Patients with GCS scores less than 9 are in a severe coma and typically require endotracheal intubation. REF: pg. 110
OBJ: 6
10. Which of the following cranial nerves are evaluated with the gag reflex? a. II and III b. V and VII c. I and II d. IX and X ANS: D
Other functions such as the pupillary response (CN II and III), the corneal reflex (CN V and VII), and the gag reflex (CT NEIS XTaB ndAX peEnR de.nC tO onMmore than one cranial nerve. N)KaSreEdLeL REF: pgs. 111-112
OBJ: 8
11. Which of the following diseases is characterized by abnormal deep tendon reflexes due to
abnormalities of the neuromuscular junction? a. Syphilis b. Myasthenia gravis c. Cerebritis d. Multiple sclerosis ANS: B
Myasthenia gravis and botulism are diseases that are characterized by abnormal deep tendon reflexes due to abnormalities of the neuromuscular junction that impair normal impulse transmission. REF: pg. 114
OBJ: 9
12. The presence of dorsiflexion of the great toe with fanning of the remaining toes during testing
of the plantar reflex is known as the: a. Babinski sign. b. “Jerk” reflex. c. Patellar reflex. d. Quadriceps reflex.
ANS: A
Dorsiflexion of the great toe with fanning of the remaining toes—the Babinski sign—is abnormal, except in children up to 12 to 18 months of age. REF: pg. 115 | pg. 117
OBJ: 9
13. Which of the following cranial nerves is intact if cough is present while the airway is
suctioned? a. I b. IX c. X d. XII ANS: C
The ability to cough with suctioning can be tested in an intubated patient and implies an intact CN X. REF: pg. 116
OBJ: 10
14. Which of the following does not describe the acronym PERRLA? a. Positioned b. Accommodation c. Round d. Reactive ANS: A
The acronym PERRLA is used very commonly to refer to normal pupils that are equal, round, and reactive to light and acTcoEm mL ovLeE mRen.tC ). OM Sm ToBdAatNioKnS(E REF: pg. 116
OBJ: 11
15. Which of the following anatomic areas is compromised when the patient has dysmetria? a. Cerebrum b. Pons c. Cerebellum d. Brainstem ANS: C
Cerebellar dysfunction results in decomposition of movement and undershooting and overshooting of goal-directed movement (dysmetria). REF: pg. 117
OBJ: 4
16. Which of the following respiratory patterns consists of phases of hyperpnea that regularly
alternate with episodes of apnea? a. Biot breathing b. Ataxic breathing c. Apneustic breathing d. Cheyne-Stokes respiration ANS: D
The most common abnormal respiratory pattern seen in patients with neurologic disorders is Cheyne-Stokes respiration, which consists of phases of hyperpnea that regularly alternate with episodes of apnea. REF: pg. 120
OBJ: 10
17. Which of the following is the most critical parameter to keep in mind when managing a
patient with intracranial hypertension? a. Mean arterial pressure b. Cerebral perfusion pressure c. Intracranial pressure d. Pulse pressure ANS: B
It is extremely important for the RT to remember that although hyperventilation is associated with lower intracranial pressure (ICP) values caused by vasoconstriction of brain blood vessels associated with hypocapnia, cerebral perfusion pressure (CPP) is the most critical element to monitor. REF: pg. 123
OBJ: 12
18. Hyperventilation would be observed in a patient who has damaged which portion of the
brainstem? a. High-midbrain b. Medulla c. Mid-midbrain d. Pons ANS: D
Sequence of Loss of Brainstem Function Breathing
Highest Level of Brainstem Function
Oculocephalic Reflex
Pupils
Pain Response
High midbrain
Normal
Normal
Normal
Mid-midbrain
Cheyne-Stokes
Present
Small but reactive Decorticate
Pons
Hyperventilation
Minimal
Mid-fixed
Decerebrate
Medulla
Ataxic
None
Mid-fixed
None
REF: Table 6-8, pg. 123
Withdrawal
OBJ: 3
19. The main goal of assessing mental status is to determine which of the following? a. Consciousness b. Awareness c. Thinking d. CNS dysfunction ANS: A
The initial goals of the examination of a patient with altered mental status are to determine whether the patient is conscious and then to determine awareness.
REF: pg. 109
OBJ: 5
20. A patient who is alert and oriented times three would know all of the following things
correctly except? a. First and last name b. Today’s date c. City in which he or she lives d. President or head of state’s name ANS: D
The patient will often be asked to state his or her name, the current date, and the present location. A fully conscious patient will be expected to answer those questions in detail, for example, first and last name, month, day, and year, and the ward and name of the hospital. Such patients can be described as alert and oriented times three (one score for each name, date, and location). REF: pg. 109
OBJ: 5
21. The brain death determination test involving disconnecting a patient from the ventilator,
providing 100% oxygen, and monitoring closely for spontaneous breathing is the . a. apnea test. b. metabolic test. c. tendon reflex test. d. deep pain test. ANS: A
After documentation of normal or near-normal PaO2 and PaCO2, the patient is disconnected from the ventilator, and 100% oxygen is administered through an endotracheal tube. The patient is closely monitored during the apnea test, with the emphasis on spontaneous breathing, oxygenation, and cardiovascular status. REF: pg. 123
OBJ: 13
Chapter 7: Clinical Laboratory Studies Test Bank MULTIPLE CHOICE 1. Which of the following terms is used for red blood cells? a. Leukocytes b. Hemocytes c. Monocytes d. Erythrocytes ANS: D
Red blood cells are also known as erythrocytes. REF: pg. 128
OBJ: 2
2. Which of the following is a formed element found in circulating blood? a. Sodium b. Platelets c. Potassium d. Bicarbonate ANS: B
Formed elements in the blood include white blood cells, red blood cells, and platelets. REF: pg. 128
OBJ: 2
3. Which of the following terms is used for white blood cells? a. Erythrocytes b. Thrombocytes c. Leukocytes d. Hemocytes ANS: C
White blood cells are also called leukocytes. REF: pg. 128
OBJ: 2
4. Which of the following terms is used for blood platelets? a. Erythrocytes b. Thrombocytes c. Leukocytes d. Hemocytes ANS: B
Blood platelets are also called thrombocytes. REF: pg. 128
OBJ: 2
5. Which of the following white blood cells is not a granulocyte? a. Basophil
b. Eosinophil c. Neutrophil d. Lymphocyte ANS: D
Lymphocytes do not have granular matter in their cytoplasm. REF: pg. 129
OBJ: 2
6. What type of white blood cell typically is most prevalent in the circulating blood? a. Monocyte b. Eosinophil c. Neutrophil d. Lymphocyte ANS: C
The neutrophil is produced by the bone marrow in large quantities. REF: pg. 129
OBJ: 2
7. What percentage of circulating white blood cells is made up of neutrophils? a. 10% to 30% b. 25% to 50% c. 40% to 75% d. 60% to 85% ANS: C
Neutrophils make up 50% to 70% of circulating white blood cells.
TESTBANKSELLER.COM
REF: pg. 130
OBJ: 2 | 7
8. Which of the following terms describes the immature neutrophil? a. Band b. Monocyte c. Erythrocyte d. Reticulocyte ANS: A
Immature neutrophils are known as bands because of the shape of their nuclei. REF: pg. 130
OBJ: 4 | 7
9. What percentage of normal circulating white blood cells are immature and nonsegmented? a. 0% to 5% b. 5% to 10% c. 10% to 15% d. 15% to 22% ANS: A
Normally, a very small portion of circulating white blood cells is nonsegmented. REF: pg. 130
OBJ: 4
10. Which of the following types of white blood cells fights bacterial infection? a. Basophil b. Monocyte c. Neutrophil d. Lymphocyte ANS: C
The neutrophil is best known for its ability to fight infection. REF: pg. 130
OBJ: 3 | 7
11. What is the normal lifespan of the neutrophil? a. 10 days b. 21 days c. 60 days d. 90 days ANS: A
The normal neutrophil lives about 10 days. REF: pg. 130
OBJ: 2 | 7
12. Which of the following terms describes an increase in white blood cell count associated with a
shift of cells from the marginated pool to the circulating pool? a. Pseudoneutrophilia b. Pseudoneutropenia c. Neutropenia d. Lymphocytosis ANS: A
When marginated neutrophils shift back into the circulating pool, this is known as pseudoneutrophilia. REF: pg. 131
OBJ: 6 | 7
13. Which of the following types of white blood cells is useful against allergic reactions and
defends against parasitic infestations? a. Monocyte b. Neutrophil c. Eosinophil d. Lymphocyte ANS: C
The eosinophil is known to react in situations caused by exposure to allergens. REF: pg. 130
OBJ: 6 | 7
14. Which of the following types of white blood cells exists as two types—the B cell and the T
cell? a. Basophil b. Monocyte c. Neutrophil
d. Lymphocyte ANS: D
Lymphocytes can be either T cells or B cells. REF: pg. 130
OBJ: 6 | 7
15. What is the normal percentage of lymphocytes in circulating white blood cells? a. 5% to 10% b. 15% to 25% c. 20% to 45% d. 50% to 70% ANS: C
Normally, 20% to 45% of circulating white blood cells are lymphocytes. REF: pg. 130
OBJ: 6 | 7
16. Which of the following types of white blood cells is physically the largest? a. Basophil b. Monocyte c. Neutrophil d. Lymphocyte ANS: B
As viewed through the microscope, the monocyte is physically the largest white blood cell. REF: pg. 130
OBJ: 7
17. Which of the following types of white blood cells is responsible for phagocytosis? a. Basophil b. Monocyte c. Neutrophil d. Lymphocyte ANS: B
The monocyte has the ability to engulf bacteria and foreign dust particles and eliminate them from the lung. REF: pg. 130
OBJ: 9
18. Which of the following terms describes an abnormal increase in the proportion of circulating
immature neutrophils? a. Neutrophilia b. Left shift c. Leukopenia d. Leukocytosis ANS: B
When the number of bands circulating in the blood is higher than normal, this is referred to as a left shift. REF: pg. 130
OBJ: 7
19. Which of the following terms describes the change in white blood cell count that often occurs
in patients with tuberculosis? a. Lymphocytosis b. Monocytosis c. Eosinophilia d. Basophilia ANS: B
Patients with tuberculosis usually develop monocytosis. REF: pg. 130
OBJ: 7 | 9
20. Which of the following terms describes an overall abnormal decrease in white blood cell
count? a. Anemia b. Leukopenia c. Leukocytosis d. Polycythemia ANS: B
An abnormal drop in the number of circulating white blood cells is known as leukopenia. REF: pg. 130
OBJ: 8
21. Which of the following types of leukocytosis is typically seen in patients with viral infection? a. Basophilia b. Neutrophilia c. Monocytosis d. Lymphocytosis ANS: D
Viral infections often cause lymphocytosis. REF: pg. 130
OBJ: 7 | 10
22. What is believed to cause neutropenia in patients with viral infection? a. Phagocytosis b. Increased margination of neutrophils c. Increased neutrophil lysis d. Increased loss of neutrophils to gastrointestinal waste ANS: B
Viral infections cause neutrophils to marginate in larger numbers, and marginated neutrophils are not counted in the white blood cell count. REF: pg. 130
OBJ: 7 | 9
23. What cells proliferate in patients with leukemia? a. Reticulocytes b. Segmented neutrophils c. Monocytes
d. Blasts ANS: D
Leukemia is a disease of the bone marrow. It causes the bone marrow to release very immature cells such as blasts. REF: pg. 132
OBJ: 8
24. What is the typical lifespan of a red blood cell? a. 30 days b. 60 days c. 120 days d. None of the above ANS: C
The typical red blood cell will live for 120 days. REF: pg. 132
OBJ: 7
25. What can be said about the nucleus of the mature circulating red blood cell? a. It is round. b. It is absent. c. It is large and red. d. All of the above can be said. ANS: B
The mature red blood cell has no nucleus. REF: pg. 132
OBJ: T9ESTBANKSELLER.COM
26. What substance is responsible for carrying red blood cell oxygen on to the tissues? a. Thrombin b. Cytokines c. Hemoglobin d. Prothrombin ANS: C
Hemoglobin is the major substance found in red blood cells. Its main function is to carry oxygen. REF: pg. 132
OBJ: 10
27. What substance is a vital constituent of the heme portion of hemoglobin? a. Iron b. Protein c. Sodium d. Calcium ANS: A
Iron is vitally important to the function of hemoglobin. REF: pg. 132
OBJ: 10
28. Which of the following terms describes an abnormal decrease in red blood cell count? a. Anemia b. Cytothemia c. Leukopenia d. Polycythemia ANS: A
A low red blood cell count is known as anemia. REF: pg. 133
OBJ: 8
29. Which of the following terms describes red blood cells that are larger than normal? a. Macrocytic b. Hypercytotic c. Hyperchromic d. None of the above ANS: A
Red blood cells that are larger than normal are termed macrocytic. REF: pg. 133
OBJ: 8
30. The most common type of anemia worldwide is the result of: a. Hemorrhage. b. Gastrointestinal bleeding. c. Iron deficiency. d. Vitamin B12 deficiency. ANS: C
Iron deficiency is a very common cause of anemia because it reduces the presence of hemoglobin in the red blood cells. REF: pg. 133
OBJ: 8
31. Which of the following terms describes anemia due to low hemoglobin levels? a. Microcytic b. Leukopenic c. Hypocytotic d. Hypochromic ANS: D
Chromic refers to color, and hemoglobin is known for giving the red blood cell its bright red color. Hypochromic refers to red cells that lack hemoglobin. REF: pg. 133
OBJ: 8
32. In which of the following conditions would secondary polycythemia not be expected? a. Pulmonary fibrosis b. Pulmonary embolism c. Living at high altitude d. Congenital heart disease ANS: B
Secondary polycythemia occurs with chronic hypoxemia. Pulmonary embolism may cause acute hypoxemia. REF: pg. 134
OBJ: 9
33. Heavy smokers are prone to what change in the red blood cell count? a. Secondary polycythemia b. Microcytic anemia c. Hypochromic anemia d. Macrocytic anemia ANS: A
Heavy smokers have significant carbon monoxide in their blood. This is perceived by the bone marrow as hypoxia. Thus, the bone marrow increases its production of red blood cells. REF: pg. 134
OBJ: 9
34. In which of the following conditions would spurious polycythemia most likely be seen? a. Lung cancer b. Dehydration c. Fever d. Hypertension ANS: B
Dehydration causes an increase in the concentration of red blood cells in the blood and causes false polycythemia. REF: pg. 134
OBJ: T9ESTBANKSELLER.COM
35. A patient with a platelet count of about 80,000/mm3 is likely to have what condition? a. Infection b. Renal failure c. Excessive bleeding d. None of the above, because this is a normal value ANS: C
A platelet count of 80,000/mm3 is too low. The patient will have problems with bleeding. REF: pg. 133
OBJ: 9 | 10
36. What problem is associated with an abnormal international normalized ratio (INR) value? a. Kidney disease b. Liver failure c. Bleeding d. Pulmonary edema ANS: C
An abnormal INR value suggests problems with blood clotting. REF: pg. 134
OBJ: 10
37. Which of the following is considered the major cation of the extracellular fluid?
a. b. c. d.
Sodium Calcium Chloride Potassium
ANS: A
Sodium is considered the major cation of the extracellular fluid. REF: pg. 136
OBJ: 12
38. Which of the following symptoms is commonly seen with hyponatremia? a. Confusion b. Headache c. Chest pain d. Shortness of breath ANS: A
A low sodium concentration often causes acute confusion in the patient. REF: pg. 136
OBJ: 12
39. What electrolyte abnormality is associated with irregular pulse, muscle weakness, and
abdominal distention? a. Hyperkalemia b. Hypokalemia c. Hyponatremia d. Hypernatremia ANS: B
Hypokalemia (reduced potassium concentration) causes an irregular pulse and muscle weakness. REF: pg. 136
OBJ: 12
40. Which of the following terms is used to describe an abnormally low potassium level? a. Hyponatremia b. Hypokalemia c. Hypochloremia d. Hypocalcemia ANS: B
A low potassium level is known as hypokalemia. REF: pg. 136
OBJ: 12
41. Elevation of blood urea nitrogen (BUN) and creatinine levels in the blood suggests what type
of problem? a. Renal failure b. Liver failure c. Heart disease d. Gallbladder disease ANS: A
BUN and creatinine are waste products of metabolism that usually are excreted by the kidneys. If they are found to be elevated in the blood, kidney failure is possible. REF: pg. 139
OBJ: 13
42. What disease is associated with elevations in sodium and chloride in sweat? a. Pneumonia b. Emphysema c. Renal disease d. Cystic fibrosis ANS: D
Patients with cystic fibrosis have elevated sodium and chloride levels in their sweat. REF: pg. 137
OBJ: 12
43. During evaluation of the enzymes, what disorder is associated with elevation of the CK-MB
on electrophoresis? a. Hepatitis b. Renal failure c. Metabolic acidosis d. Myocardial infarction ANS: D
Elevation of the creatine kinase MB (CK-MB) fraction indicates damage to the heart muscle. REF: pg. 141
OBJ: 14 | 15
44. The highest levels of aspartate aminotransferase (AST) are seen in acute: a. pulmonary infarction. b. hepatitis. c. myocardial infarction. d. renal failure. ANS: B
AST is an enzyme that is found primarily in the liver. Hepatitis causes damage to the liver and an abnormal increase in AST. REF: pg. 140
OBJ: 13
45. Elevation of troponin is associated with what disorder? a. Acute myocardial infarction b. Hepatitis c. Pulmonary embolism d. Trauma ANS: A
Troponin is an enzyme that is elevated by damage to the myocardium. REF: pg. 141
OBJ: 14 | 15
46. Which of the following disorders is associated with hyperglycemia?
a. b. c. d.
Hepatitis Diabetes Renal failure Myocardial infarction
ANS: B
Hyperglycemia, which refers to an increase in blood glucose level, is associated most often with diabetes. REF: pg. 138
OBJ: 13
47. Hypoalbuminemia may result in what pulmonary problem? a. Asthma b. Emphysema c. Diaphragm paralysis d. Pulmonary edema ANS: D
Hypoalbuminemia refers to a low blood protein level. This causes the osmotic pressure of the blood to decrease, which leads to increased water in the lung. REF: pg. 139
OBJ: 13
48. Which of the following terms is used to describe the microorganisms present in the saliva of a
healthy person? a. Normal flora b. Acid-fast bacteria c. Gram-negative pathogeTnE s STBANKSELLER.COM d. Gram-positive pathogens ANS: A
The normal flora refers to the microorganisms normally present in the saliva. REF: pg. 143
OBJ: 17
49. The purpose of performing the sensitivity test on sputum samples is to: a. determine the best antibiotic to prescribe. b. determine the viscosity of the sputum. c. determine the quality of the sputum sample. d. identify the general type of organism responsible for the infection. ANS: A
The sensitivity determines what antibiotics will be most effective against a microorganism that is causing an infection. REF: pg. 143
OBJ: 18
50. Which of the following organisms is responsible for most cases of bacterial pneumonia? a. Pseudomonas b. Klebsiella pneumoniae c. Staphylococcus aureus d. Streptococcus pneumoniae
ANS: D
Most community-acquired pneumonias are caused by the Streptococcus organism. REF: pg. 142
OBJ: 17 | 18
51. What procedure is performed by a pulmonologist to obtain organisms that may be present
deep in the lung in patients with pneumonia? a. Sputum induction with hypotonic nebulized solution b. Bronchoalveolar lavage (BAL) c. Tracheal intubation with lavage d. All of the above ANS: B
BAL is done to flush out organisms deep in the lung that cannot be obtained for culture through less invasive procedures. REF: pg. 144
OBJ: 19
52. The classic dividing line between transudates and exudates is a protein level of a. 3 b. 6 c. 9 d. None of the above is correct.
g/dL.
ANS: A
Pleural fluid that has a protein concentration of more than 3 g/dL is known as an exudate. Pleural fluid with a lower concentration of proteins is known as a transudate. REF: pg. 144
OBJ: 24
53. Which of the following conditions is not known to cause anergy? a. Chronic hypertension b. Sarcoidosis c. Lung cancer d. Aging ANS: A
Chronic hypertension does not cause anergy. REF: pg. 145
OBJ: 25
54. For which of the following disorders does a skin test use purified protein derivative (PPD)? a. Sarcoidosis b. Tuberculosis c. Coccidioidomycosis d. Histoplasmosis ANS: B
PPD is injected under the skin when testing for tuberculosis. REF: pg. 145
OBJ: 28
55. If an infection is being effectively treated by an antibiotic, which of the following would be
true? a. The Gram-stained smear will no longer demonstrate the pathogen. b. The patient’s white blood count will increase. c. The patient’s platelet count will rapidly increase. d. The patient will present with foul-smelling sputum. ANS: A
After a patient has been on a regimen of antimicrobial therapy, the Gram stain and culture results can change. If the antibiotic is effective against the pathogen causing the infection, the Gram stain and culture will no longer demonstrate the pathogen, but instead will reveal organisms consistent with normal flora. REF: pg. 143
OBJ: 18
56. A patient presents with a D-dimer of 210 ng/mL. Which of the following would not be a
suspected disease process or condition? a. b. c. d.
Streptococcal pneumonia Pleural effusion Pulmonary embolism Tuberculosis
ANS: C
The D-dimer test has high sensitivity; a level below the upper limit of the reference range can help rule out both systemic thrombosis, such as disseminated intravascular coagulation, and local thrombosis, such as pulmonary embolism and deep vein thrombosis. REF: pg. 136
OBJ: 11
Chapter 8: Interpretation of Blood Gases Test Bank MULTIPLE CHOICE 1. Before an arterial blood gas (ABG) value is obtained, the patient’s clotting parameters should
be evaluated because: a. they may affect the patient’s PaO2. b. if reduced, they may hinder filling of the syringe with blood during the draw. c. bleeding time may be prolonged if they are abnormal. d. they may affect the accuracy of the sample pH. ANS: C
Before arterial puncture, the clinician may find it helpful to review the patient’s chart for clinical laboratory studies that reflect the patient’s blood clotting ability (see Chapter 8). Abnormalities such as low platelet count or increased bleeding time may indicate that postpuncture bleeding will pose a problem. In such cases, the puncture site should be pressurized longer than usual to prevent hemorrhage. REF: pg. 154
OBJ: 4
2. Which of the following is the most common site for arterial puncture? a. Radial artery b. Ulnar artery c. Temporal artery d. Jugular artery ANS: A
The radial artery is the preferred puncture site, because it is accessible and easy to stabilize after the puncture. Adequate collateral circulation can be evaluated and is usually available if the radial artery becomes obstructed during the puncture procedure. REF: pg. 154
OBJ: 4
3. Which of the following tests is performed to check the collateral circulation of the radial
artery before puncture? a. Wilson’s test b. Perfusion scan c. Sack’s test d. Allen’s test ANS: D
The degree of collateral circulation to the hand is evaluated by a modified Allen’s test before the radial artery is punctured. REF: pg. 155
OBJ: 4
4. For the test of collateral circulation in an Allen’s Test, “pinking up” of the hand is normal if it
occurs within a. 10 to 15
seconds.
b. 15 to 20 c. 20 to 25 d. 25 to 30 ANS: A
If collateral flow is adequate, the patient’s hand will “pink up” within 10 to 15 seconds; this constitutes a positive Allen’s test. REF: pg. 155
OBJ: 4
5. An arterial puncture site normally should be compressed for a minimum of
minutes after
the puncture. a. 1 to 2 b. 3 to 5 c. 5 to 7 d. 8 to 10 ANS: B
After the needle is withdrawn, the wound should be pressurized for a period of at least 3 to 5 minutes, or longer if clotting problems exist. REF: pg. 154
OBJ: 4
6. What is the primary method of transporting oxygen in the blood? a. Dissolved in plasma b. Bound to plasma proteins c. In the form of HCO3d. Bound to hemoglobin ANS: D
Oxygen in the blood is transported primarily bound to hemoglobin. REF: pg. 156
OBJ: 5
7. What is the normal value for PaO2 in the adult patient? a. 50-60 mm Hg b. 60-70 mm Hg c. 70-80 mm Hg d. 80-100 mm Hg ANS: D
The normal value is approximately 80-100 mm Hg in room air. REF: pg. 158
OBJ: 1 | 2
8. Which of the following is a true statement regarding the term hypoxemia? a. It occurs when the patient’s PaO2 is lower than predicted. b. It indicates a condition in which tissue oxygenation is inadequate. c. It is always considered a severe disorder. d. It is independent of the age of the patient. ANS: A
When the measured PaO2 is below the predicted range for a patient breathing room air, regardless of the actual FIO2, this is called hypoxemia. REF: pg. 159
OBJ: 1 | 5
9. What is a decrease in tissue oxygenation called? a. Hypoxia b. Hypoxemia c. Hypercapnia d. Acidosis ANS: A
Hypoxia is a term that often is associated with hypoxemia; it indicates a condition in which tissue oxygenation is inadequate. REF: pg. 162
OBJ: 5
10. What is the most common physiologic cause of hypoxemia in patients with lung disease? a. Hypoventilation b. mismatch c. Shunt d. Diffusion defect ANS: B
mismatching is the most common cause of hypoxemia in patients with lung disease. REF: pg. 159
OBJ: 5 | 6
11. Which of the following shifts the oxyhemoglobin dissociation curve to the left? a. Increased PaCO2 b. Decreased pH c. Increased body temperature d. Fetal hemoglobin ANS: D
Alkalosis, hypocapnia, hypothermia, fetal hemoglobin, and carboxyhemoglobin shift the curve to the left, resulting in higher SaO2 values at the same PaO2. REF: pg. 160
OBJ: 5 | 6
12. A shift to the right in the oxyhemoglobin dissociation curve has what effect on the affinity of
hemoglobin for oxygen? a. Increased b. Decreased c. No change d. Cannot predict the change ANS: B
Shifts to the left cause oxygen to be more tightly bound to hemoglobin and make unloading of oxygen at the tissues more difficult, whereas shifts to the right have the opposite effect, resulting in decreased oxygen affinity for hemoglobin and allowing easier unloading of oxygen at the tissues.
REF: pg. 160
OBJ: 5 | 6
13. What is the normal value for CaO2? a. 16-20 vol% b. 12-16 vol% c. 8-12 vol% d. 4-8 vol% ANS: A
The normal value is 16-20 mL/dL blood. REF: pg. 161
OBJ: 6
14. Which of the following would be best used to assess arterial oxygenation? a. PaO2 b. SaO2 c. CaO2 d. P(A – a)O2 ANS: C
CaO2, because it is the sum of oxygen bound to hemoglobin and that dissolved in plasma. It is thus one of the most important blood gas measurements, because it significantly influences tissue oxygenation. REF: pg. 161
OBJ: 6
15. Which of the following woTuE ldSbTeBthAeNbK esSt E inL diL caEtoRr.oC fO tisMsue oxygenation? a. PaO2 b. SaO2 c. CaO2 d. P O2 ANS: D
One dimension capable of indicating the tissue oxygenation status in most cases is the P O2. REF: pg. 158
OBJ: 6
16. A 32-year-old man comes to the emergency department after a traffic accident with the
following data: pulse, 118 beats/min; respiratory rate, 27 breaths/min; blood pressure, 100/68 mm Hg; paradoxical chest movement on the left side; breath sounds decreased on the left; and ABG on room air (21%) as follows: pH, 7.32; PaCO2, 70 mm Hg; PaO2, 57 mm Hg; HCO3-, 23 mEq/L; base excess, 0; SaO2, 86%, CaO2, 15.2 vol%; Hb, 13.0 g/dL; P(A – a)O2, 18 mm Hg. Based on this information, what is the primary cause of the patient’s hypoxemia? a. Overall hypoventilation b. mismatch c. Diffusion defect d. Shunt ANS: A
If hypoxemia exists at an FIO2 of 0.21 and the sum of the PaO2 and the PaCO2 is 110 to 130 mm Hg, the cause of the hypoxemia is hypoventilation.
REF: pg. 159
OBJ: 5 | 6
17. What is often the first clinical sign that suggests the presence of hypoxemia? a. Hypotension b. Confusion c. Decreased level of consciousness d. Exertional dyspnea ANS: D
Clinical recognition of hypoxemia often is first suggested when the patient complains of shortness of breath, especially with exertion. REF: pg. 159
OBJ: 5 | 6
18. What could cause tissue hypoxia with normal arterial oxygenation? a. Inadequate cardiac output b. Polycythemia c. Leukocytosis d. Bradypnea ANS: A
Tissue hypoxia may occur with normal arterial oxygenation when inadequate perfusion exists. Clinical signs of an inadequate cardiac output include hypotension, cool extremities, weak or absent peripheral pulses, reduced urine output, and coma. REF: pg. 162
OBJ: 5 | 6
19. What is the normal value for arterial pH? a. 7.20 to 7.30 b. 7.35 to 7.45 c. 7.45 to 7.50 d. Greater than 7.50 ANS: B
The normal value is 7.40, with a range of 7.35 to 7.45. REF: pg. 162
OBJ: 1 | 7
20. The negative log of the hydrogen ion concentration is defined as which of the following? a. PaCO2 b. Base excess c. Standard HCO3d. pH ANS: D
The actual concentration of H+ in the arterial blood serum is low (0.00004 mEq/L) and is a cumbersome number with which to work. To solve this problem, the proposal has been made to convert the H+ term to pH by taking the negative logarithm (base 10) of the hydrogen ion concentration to attain more manageable numbers. REF: pg. 162
OBJ: 7 | 8
21. What defines acidosis in a human? a. pH <7.0 b. pH <7.25 c. pH <7.35 d. pH <7.40 ANS: C
As H+ increases from the addition of acids, pH decreases. Therefore pH values below 7.35 represent increases in the H+ concentration, and the blood is acidotic or acidemic. REF: pg. 162
OBJ: 8
22. Which of the following parameters is the respiratory component of acid-base status? a. PaCO2 b. HCO3c. PaO2 d. Base excess ANS: A
PaCO2 is a reflection of the respiratory component of acid-base status. REF: pg. 163
OBJ: 8
23. What is the direct effect of an increased PaCO2 on HCO3 ? a. Causes it to increase b. Causes it to decrease c. Does not have any effeT ctEoSnTitBANKSELLER.COM d. Complicated because it has an unpredictable effect on bicarbonate
ANS: A
Increases in PaCO2 shift the reaction to the right and result in immediate but small increases in plasma HCO3-. REF: pg. 163
OBJ: 8 | 9
24. Which of the following is the best indicator of metabolic acid-base status? a. Plasma HCO3b. Base excess c. Standard HCO3d. T40 HCO3ANS: B
Base excess provides a more complete analysis of metabolic buffering capabilities. REF: pg. 163
OBJ: 8 | 9
25. Which of the following is a correct representation of the Henderson-Hasselbalch equation? a. pK = pH – log (PaCO2 0.03)/ HCO3b. pK = pH – log HCO3- /(PaCO2 0.03) c. pH = pK + log HCO3- /(PaCO2 0.03) d. –pH = pK – log HCO3- /(PaCO2 0.03)
ANS: C
The Henderson-Hasselbalch equation is written as follows: pH = pK + log REF: pg. 163
OBJ: 7
26. Which of the following ratios of HCO3- /PaCO2 results in a pH of 7.40? a. 24:1 b. 20:1 c. 15:1 d. 6:0.03 ANS: B
The equation demonstrates that arterial blood pH is determined by the ratio of HCO3- to PaCO2. This ratio normally is 20:1. Changes in one component disrupt the 20:1 ratio, resulting in an abnormal pH. REF: pg. 163
OBJ: 9 | 10
27. A 17-year-old man is brought into the emergency department. Vitals are as follows: pulse,
100 beats/min; respiratory rate, 4 breaths/min; and blood pressure, 100/65 mm Hg. The patient was at a party, where he was discovered by his friends to be slumped in a chair and unresponsive. ABG results are as follows: pH 7.29, PaCO2 68 mm Hg, HCO3- 29 mEq/L, BE + 1 mEq/L The patient’s acid-base staT tuE sS isTcB laA ssN ifK ieS dE asLw thM e following? LhEicRh.oCf O a. Uncompensated respiratory acidosis b. Partially compensated respiratory acidosis c. Compensated respiratory alkalosis d. Uncompensated metabolic acidosis ANS: A
Uncompensated respiratory acidosis is identified by an elevated PaCO2, a decreased pH, and a normal plasma HCO3- and base excess. The HCO3- is mildly raised here, but note that the BE is normal. REF: pgs. 163-164
OBJ: 8
28. An acute increase in PaCO2 of 10 to 15 mm Hg causes a corresponding increase in plasma
HCO3- of how many mEq/L? a. 1 b. 2 c. 3 d. 4 ANS: A
For acute respiratory acidosis, the plasma HCO3- increases by 1 mEq/L for each 10 to 15 mm Hg that the PaCO2 increases. REF: pg. 164
OBJ: 8
29. Which of the following is true regarding respiratory alkalosis? a. The PaCO2 is less than 35 mm Hg. b. An increase in HCO3- compensates for respiratory alkalosis. c. It is called completely compensated if the pH is 7.52. d. It is called partially compensated if the pH is in the normal range. ANS: A
Respiratory alkalosis, which is identified by a PaCO2 below the expected level (35-45 mm Hg), indicates that ventilation is exceeding the normal level. Hyperventilation usually is the result of an increased stimulus or drive to breathe. This occurs with pain, hypoxemia (PaO2 <60 mm Hg), acidosis, and anxiety. REF: pg. 165
OBJ: 8 | 9
30. A 35-year-old, 54-kg woman with congestive heart failure enters the emergency department
short of breath. An ABG shows the following results: pH 7.50, PaCO2 30 mm Hg, HCO3- 23 mEq/L, base excess +2 mEq/L The patient’s ABG results indicate which of the following? a. Uncompensated respiratory alkalosis b. Compensated respiratory acidosis c. Uncompensated metabolic alkalosis d. Uncompensated metabolic acidosis ANS: A
Respiratory alkalosis, which is identified by a PaCO2 below the expected level, indicates that ventilation is exceeding the normal level. The kidneys compensate for respiratory alkalosis by TE SaEH LCLOE3R .iCthOaMlow PaCO2 and increased pH is excreting plasma HCO3- . A noSrT mB alApN laK sm -w called uncompensated respiratory alkalosis. REF: pg. 165
OBJ: 8
31. Given the following ABG results, interpret the acid-base status:
pH 7.44, PaCO2 25 mm Hg, HCO3- 17 mEq/L, base excess –6 mEq/L a. Compensated metabolic acidosis b. Uncompensated respiratory alkalosis c. Uncompensated respiratory acidosis d. Compensated respiratory alkalosis ANS: D
Compensation for respiratory alkalosis occurs as follows. Partial compensation occurs when the plasma HCO3- falls below normal but the pH is still above 7.45. Full compensation occurs when the plasma HCO3- decreases enough to return the pH to within normal range. REF: pg. 165
OBJ: 8 | 9
32. A patient has the following ABG results:
pH 7.25, PaCO2 32 mm Hg, HCO3- 16 mEq/L, base excess –10 mEq/L Based on these findings, the patient has which of the following? a. Compensated metabolic acidosis b. Uncompensated respiratory acidosis
c. Uncompensated metabolic acidosis d. Compensated respiratory acidosis ANS: C
Metabolic acidosis is identified when the plasma HCO3- or base excess falls below normal. REF: pg. 165
OBJ: 8
33. Which of the following could cause metabolic acidosis? a. Cardiovascular disease b. Renal disease c. Hypokalemia d. Vomiting ANS: B
Following is an outline of the causes of metabolic acidosis: Loss of HCO3- may be seen with diarrhea or renal disease, and increased metabolic acid production is seen with ketoacidosis, lactic acidosis, ingestion of certain toxins (e.g., methanol), and posthypocapnia disorder. REF: pg. 165
OBJ: 8 | 9
34. Answer the following questions based on the following blood gas data:
pH PaCO2 PaO2 SaO2 Hb CaO2 Bicarbonate Base excess Respiratory rate FIO2
7.28 67 mm Hg 49 mm Hg 76% 10.1 g/dL 10.4TvEoS l%TBANKSELLER.COM 26 mEq/L –2 mEq/L 25 beats/min 0.70
Which of the following is true regarding the PaO2? a. It is adequate. b. It shows mild hypoxemia. c. It shows moderate hypoxemia. d. It shows severe hypoxemia. ANS: C
Defining Levels of Hypoxemia PaO2 (mm Hg) Severity of hypoxemia 60-79 Mild 40-59 Moderate <40 Severe Patients younger than 60 years breathing room air REF: pg. 159
OBJ: 5
35. Which of the following is true regarding oxygen-carrying capacity?
a. b. c. d.
It is normal. It is increased. It is decreased. Cannot be determined with the given data.
ANS: C
Reduced because hemoglobin (Hb) is only 10.1 g/dL. Because most oxygen (99%) is carried in the blood bound to hemoglobin, CaO2 cannot be normal without adequate circulating red blood cells containing hemoglobin. REF: pgs. 159-160
OBJ: 5
36. The acid-base status is classified as which of the following? a. Uncompensated metabolic alkalosis b. Partially compensated metabolic alkalosis c. Uncompensated respiratory acidosis d. Partially compensated respiratory acidosis ANS: C
The pH is reduced, indicating that an acidosis is present. The elevated PCO2 tells us it is respiratory acidosis. The bicarb is within normal limits, indicating that the respiratory acidosis is uncompensated. REF: pg. 164
OBJ: 9
37. Which of the following could cause this patient’s problem? a. Anxiety and fear b. Acute airway obstructiT onESTBANKSELLER.COM c. Lactic acid production d. Drug overdose ANS: B
Respiratory acidosis may result from a variety of respiratory and nonrespiratory abnormalities. Examples include: acute upper airway obstruction, severe diffuse airway obstruction (acute or chronic), and massive pulmonary edema. REF: pg. 164
OBJ: 9
38. Which of the following statements is true regarding the accuracy of the reported results? a. The oxygen is inaccurate. b. The pH is reported considerably higher than it should be. c. The blood gas results are all accurate. d. The reported pH is considerably lower than the given PaCO2 and HCO3- would
indicate. ANS: B
Use the Henderson-Hasselbalch equation to determine the pH. Instead of the 7.29 stated in the case above, based on the PaCO2 and the HCO3-, this value should be 7.21. REF: pg. 163
OBJ: 11 | 12
Chapter 9: Pulmonary Function Testing Test Bank MULTIPLE CHOICE 1. Which of the following is not a general purpose of performing pulmonary function tests
(PFTs)? a. The functional status of the lungs b. The diffusion characteristics of the alveolar-capillary membrane c. How much and how fast gas can be moved into and out of the lungs d. The specific diagnosis of pulmonary disease ANS: D
Pulmonary function testing is employed to (1) evaluate the cause of pulmonary symptoms such as dyspnea, cough, wheezing, sputum, exercise intolerance, and chest pain; (2) evaluate abnormalities seen on chest x-ray and/or computed tomography (CT) scan; (3) follow the course of a disease and the response to treatment; (4) evaluate a person’s risk for perioperative pulmonary complications; (5) rule out significant pulmonary pathologic conditions in people with a high risk for pulmonary dysfunction, such as smokers, firemen, and those exposed to asbestos; and (6) evaluate disability. REF: pgs. 180-181
OBJ: 1
2. Which of the following is n s b.eCenOM studied to establish normal values for ToEt Sa TpaBraAmNeKteSr EthLatLhEaR
PFTs? a. Gender b. Height c. Race d. Weight ANS: D
The range of normal values for pulmonary function testing results for individuals of varying height, age, gender, and race has been studied. REF: pg. 180
OBJ: 2
3. Which of the following factors has the greatest influence on lung size and predicted PFT
values? a. Gender b. Height c. Race d. Weight ANS: B
Height is the most important factor influencing lung size and predicted values. Generally, the taller the person, the larger the predicted lung volumes, flow rates, and diffusing capacity. REF: pg. 180
OBJ: 2
4. Which of the following is the primary instrument used in performing PFTs? a. Spirometer b. Flowmeter c. Thermistor d. Spirograph ANS: A
The instrument used primarily in pulmonary function testing has been the spirometer. REF: pg. 181
OBJ: 3
5. Which of the following associations’ standards should be met before PFT equipment is
purchased and used? a. The American Lung Association b. The American College of Chest Physicians c. The Joint Commission d. The American Thoracic Society ANS: D
It is recommended that PFT technicians make sure their equipment meets American Thoracic Society (ATS) standards before purchase and use. REF: pg. 184
OBJ: 3 | 9
6. Which of the following is not standard equipment in a pulmonary function laboratory? a. Spirometer b. Gas analyzer c. Oxygen source d. Body plethysmograph ANS: C
Pulmonary function laboratories that test a large number of patients often will have the following equipment: a spirometer (for measuring routine flows and volume), a body plethysmograph (for total lung capacity and airway resistance studies), a diffusion system (for measuring lung diffusion), gas analysis equipment (carbon dioxide, carbon monoxide, helium, nitrogen, and oxygen), nebulizer equipment for albuterol and methacholine, an arterial blood gas analyzer, and a treadmill or bicycle for exercise evaluation. REF: pg. 181 | pg. 190
OBJ: 3
7. The volume of air inhaled and exhaled during quiet breathing is known as the: a. resting volume. b. tidal volume. c. inspiratory volume. d. inspiratory capacity. ANS: B
The tidal volume is the volume of air moved during quiet breathing. REF: pg. 181
OBJ: 1
8. What is the normal minute ventilation of an average person at rest? a. <5 L/min b. 5 to 10 L/min c. 5 to 15 L/min d. >15 L/min ANS: B
The normal REF: pg. 181
E values range from 4 to 12 L/min at rest, depending on the size of the patient. OBJ: 1 | 3
9. Which of the following is the most common cause of erroneous forced vital capacity (FVC)
test results? a. Maximal inspiratory effort b. Age of the patient c. Age of the equipment d. Suboptimal coaching ANS: D
Proper coaching of the patient during the FVC maneuver is extremely important because the most common cause of erroneous FVC test results is suboptimal coaching. REF: pg. 185
OBJ: 5
10. What may cause the FVC to decrease in a patient with a restrictive lung disorder? a. A decrease in lung size b. An increase in the expiT raEtoSryTB reA seNrvKeSvE olLuL mE eR (E.RCVO)M c. A decrease in expiratory muscle mass d. An increase in airway resistance caused by the disease ANS: A
Restrictive lung disorders reduce the FVC by shrinking the lung. REF: pg. 187
OBJ: 5
11. Which of the following may decrease the FVC in a patient with an obstructive lung disorder? a. A decrease in lung size b. An increase in residual volume (RV) c. A decrease in expiratory muscle mass d. An increase in airway resistance caused by the disease ANS: B
Obstructive lung dysfunction produces a decrease in the FVC by causing air trapping, which results in a large residual volume. REF: pg. 187
OBJ: 4 | 5
12. The presence of an FVC that is significantly smaller than the slow vital capacity (SVC) is
consistent with: a. poor inspiratory effort. b. use of a bronchodilator.
c. air trapping. d. forced exhalation. ANS: C
If the measured FVC is significantly smaller than the SVC, air trapping is occurring. REF: pg. 188
OBJ: 4 | 5
13. Which level of vital capacity (VC) is associated with an increased risk of postoperative
respiratory complications? a. <50 mL/kg b. <40 mL/kg c. <20 mL/kg d. <15 mL/kg ANS: C
A significantly reduced VC (<20 mL/kg of ideal body weight) indicates that the patient is at increased risk for postoperative respiratory complications. REF: pg. 181
OBJ: 4 | 5
14. Which level of VC is associated with an increased risk of respiratory failure? a. <50 mL/kg b. <40 mL/kg c. <20 mL/kg d. <15 mL/kg ANS: D
A VC of less than 15 mL/kg indicates that the adult patient’s ventilatory reserve is decreased enough to cause respiratory failure. REF: pg. 181
OBJ: 4 | 5
15. The total lung capacity (TLC) is a function of all of the following except: a. age. b. birth weight. c. gender. d. size. ANS: B
It is a function of the person’s size, age, race, and gender, so it is reported as the measured volume and as a percentage predicted. REF: pg. 180
OBJ: 2
16. The body plethysmograph uses which of the following gas laws? a. Boyle’s law b. Charles’ law c. Gay-Lussac’s law d. Combined gas law ANS: A
The body plethysmograph takes advantage of Boyle’s law, which says that the pressure and volume of a gas vary inversely if the temperature is constant. REF: pg. 190
OBJ: 2 | 3
17. In which of the following diseases will the open-circuit nitrogen washout method tend to
underestimate the total lung volume? a. Asthma b. Emphysema c. Cystic fibrosis d. Chronic bronchitis ANS: B
If there is significant air trapping, which is common with emphysema, the nitrogen washout technique would underestimate the total intrathoracic gas volume. REF: pgs. 189-190
OBJ: 18
18. The closed-system helium dilution method is used to measure: a. TLC. b. FVC. c. FRC. d. TV. ANS: C
The functional residual capacity (FRC) can be calculated with the following formula: FRC = (Initial helium – Final helium) V BTPS TESTBA FiN naKl S heElL iuL mER.COM REF: pg. 189
OBJ: 16
19. The ERV is limited in clinical usefulness. It may be reduced in all of the following cases
except: a. obesity. b. restrictive disorders. c. obstructive disorders. d. patients making a poor effort during testing. ANS: C
Obesity, poor effort, and restrictive lung disorders reduce the ERV. REF: pg. 182
OBJ: 7
20. A loss of elasticity of the lung tissue will cause the FRC to: a. increase. b. decrease. c. stay the same. d. None of the above. ANS: A
Any dysfunction that causes a loss of lung tissue elasticity (e.g., emphysema) increases the FRC because the forces (ribs and respiratory muscles) that expand the chest wall have a decreasing amount of opposition. REF: pg. 189
OBJ: 16
21. How would the inspiratory reserve volume and the inspiratory reserve capacity be described
in a patient with a mild to moderate restrictive disorder? a. Normal b. Abnormally increased c. Abnormally decreased d. Not measured ANS: A
Inspiratory reserve volume and inspiratory capacity are measured by the routine spirogram. These values are not used widely in evaluation of pulmonary dysfunction, however. Both these measurements usually are reduced with restrictive lung disease but can be normal. They usually are normal in mild to moderate obstructive lung disease. REF: pg. 187
OBJ: 13
22. The forced expiratory volume in 1 second (FEV1) usually is expressed as a percentage of the
observed: a. PEF. b. TLC. c. FEV3. d. FVC. ANS: D
People with normal lungs will exhale approximately 75% of VC in 1 second. REF: pg. 182
OBJ: 6
23. The normal FEV1 to FVC ratio is approximately: a. 50%. b. 66%. c. 75%. d. 90%. ANS: C
People with normal lungs will exhale approximately 75% of VC in 1 second. REF: pg. 182
OBJ: 12
24. The FEV3 measurement is an indicator of flow in which airways? a. Large b. Medium c. Small d. Medium and small ANS: C
Restrictive lung disease often will cause a high ratio; obstructive lung disease often will lead to a low ratio unless severe air trapping is present. REF: pg. 182
OBJ: 7
25. The normal FEV3 to FVC ratio is approximately: a. 50%. b. 66%. c. 75%. d. 95%. ANS: D
The FEV3 is the 3-second point of the expiratory curve. It is not as reproducible as the FEV1, so the range of normal is much greater than that of the FEV1. It often is reported as a percentage of the FVC; a normal ratio is approximately 95%. REF: pg. 182
OBJ: 12
26. Which of the following tests is more sensitive than the FEV1 in detecting airway obstruction? a. FVC b. FEV1/FVC c. FEV3 d. FEF25% to 75% ANS: D
The FEF25% to 75% measures the average flow rate during the middle half of the expiratory curve. It is not as reproducible as the FEV1, so the normal range is 65% to 100% of predicted. It may be abnormal when tT hE eS FETVB1AisNnKoS rm oR it.isCm EaLl,LsE OoMre sensitive to airway obstruction than is the FEV1. REF: pg. 182
OBJ: 7
27. The maximal voluntary ventilation (MVV) test usually is performed over a period of: a. 5 to 10 breaths. b. 12 to 15 seconds. c. 1 to 2 minutes. d. 5 to 10 minutes. ANS: B
The MVV measurement requires the patient to breathe as rapidly and deeply as possible for 12 to 15 seconds. REF: pg. 183
OBJ: 8
28. The MVV reflects all of the following except: a. airway resistance. b. condition of respiratory muscles. c. condition of lung tissue. d. compliance of the lungs and thorax. ANS: C
The MVV reflects the status of the respiratory muscles, compliance of the thorax–lung complex, and airway resistance (Raw); normal values vary widely. REF: pg. 183
OBJ: 8
29. Which of the following tests is often used preoperatively to evaluate whether the patient may
have postoperative complications? a. ABG analysis b. IC measurement c. MVV test d. PEF test ANS: C
The MVV test is a good measure of patient motivation and ability to move air. It often is used as part of a preoperative pulmonary assessment because motivation and ability are both important in minimizing postoperative pulmonary complications. REF: pg. 183
OBJ: 8
30. To determine whether there is a positive response to bronchodilator therapy, improvement
must be seen in all of the following values except: a. FVC. b. FEV1. c. FEF25% to 75%. d. FEF75% to 85%. ANS: D
The amount of change requTiE reS dT toBqAuN alK ifS yE asLiL mE prRo. veCmOeM nt is as follows: FVC: increase greater than 10%; FEV1: increase of 200 mL or 15% over the baseline FEV1; and FEF25% to 75%: 20% to 30% increase. REF: pg. 188
OBJ: 7
31. The DLCO is a test that is used to measure the: a. amount of carbon monoxide in the blood. b. thickness of the alveolar-capillary membrane. c. diffusion coefficient of carbon monoxide in the lungs. d. ability of a gas to move across the alveolar-capillary membrane. ANS: D
The ability of gas to diffuse across the alveolar-capillary membrane can be measured and is known as the diffusion capacity (DL). REF: pg. 192
OBJ: 18
32. Which of the following conditions is not a determinant of gas exchange across the membrane? a. Amount of oxygen in the blood b. Thickness of the alveolar-capillary membrane c. Diffusion coefficient of carbon monoxide in the lungs d. Surface area of the membrane ANS: A
The determinants of gas exchange across the membrane include the following: the diffusion coefficient of the gas used in testing; the surface area of the membrane; the thickness (diffusion characteristics) of the membrane; the hemoglobin and blood flow in the pulmonary capillaries; and the distribution of inspired gas (matching of ventilation to perfusion). REF: pg. 192
OBJ: 18 | 20
33. Which of the following diseases is not associated with a reduced DLCO? a. Emphysema b. Chronic bronchitis c. Pulmonary embolus d. Pulmonary fibrosis ANS: B
Diseases that reduce the actual surface area or the effective surface area of the alveolar-capillary membrane, such as emphysema, pulmonary fibrosis, or pulmonary embolus, can reduce the DL. REF: pg. 192
OBJ: 18 | 20
34. The concentration of carbon monoxide used in a DLCO study is approximately: a. 0.1%. b. 0.4%. c. 1.4%. d. 10%. ANS: B
Diffusion is measured mosTt E coSmTmBoAnN lyKbS yEhL avLinEgRth.eCpOatMient inhale a single breath of gas containing 0.4% carbon monoxide. REF: pg. 192
OBJ: 18 | 19
35. The reason why carbon monoxide is used in DLCO studies instead of oxygen is because
carbon monoxide is: a. more readily available than oxygen. b. neither diffusion nor perfusion limited. c. perfusion limited and not diffusion limited. d. diffusion limited and not perfusion limited. ANS: D
Carbon monoxide binds with hemoglobin at 200 times the strength of oxygen binding, so its diffusion is limited only by the surface area of the pulmonary-capillary membrane. It is not limited by capillary blood flow or thickness of the alveolar-capillary membrane. REF: pg. 192
OBJ: 18 | 19
36. In a patient with an exacerbation of asthma, airway resistance will be: a. normal. b. increased. c. decreased. d. impossible to measure.
ANS: B
Airway resistance increases with any pathologic process that narrows the airways, such as asthma, bronchitis, or emphysema. REF: pg. 191
OBJ: 21
37. What will happen to the static lung compliance of a postoperative heart patient with left lower
lobe atelectasis? a. It will be normal. b. It will be less than normal. c. It will be greater than normal. d. It will be equal to the dynamic lung compliance. ANS: B
Static and dynamic compliance values decrease when the lung becomes stiffer, as with atelectasis, pneumonia, and pulmonary fibrosis. REF: pg. 195
OBJ: 22
38. Which of the following tests would be used to determine whether there is gross
misdistribution of ventilation in the lungs of a patient given a diagnosis of emphysema? a. DLCO b. Helium dilution c. Closing volume d. Nitrogen washout ANS: D
Nitrogen washout is used tT oE deSteTrB mA inNeKthSeEdL isL triEbR ut. ioC nOoM f ventilation. REF: pg. 189
OBJ: 20
39. The closing-volume (CV) test is used for early diagnosis of obstruction of airways of which of
the following sizes? a. Large b. Medium c. Small d. Large and medium ANS: C
The CV test is a special form of nitrogen washout that is used for early diagnosis of obstruction in the small airways. REF: pg. 189
OBJ: 20
40. Which of the following tests is not considered for exercise testing? a. Stress ECG b. Ventilatory capacity c. Anaerobic threshold d. DLCO ANS: D
Some of the uses for exercise testing include: stress electrocardiography; ventilator capacity; blood gases; exercise bronchial provocation; anaerobic threshold; maximal oxygen uptake ( O2max); and exercise ability. REF: pg. 199
OBJ: 12 | 29
41. Which of the following tests would be used to determine at what point oxygen demand
exceeds oxygen consumption during hard exercise? a. Exercise challenge b. Ventilatory capacity c. Anaerobic threshold d. Maximal oxygen uptake ( O2max) ANS: C
Anaerobic threshold: When the body is stressed by exercise, there is a point at which the oxygen need exceeds availability, and a backup mode of metabolism called anaerobic (without oxygen) metabolism is brought into action. Exercise testing is useful for identifying this point for athletes in training and for patients with heart disease who may need an individualized exercise program that is safe for them. Proper exercise training will increase the anaerobic threshold. REF: pg. 200
OBJ: 25
42. Which of the following tests would be used to determine at what point oxygen consumption
reaches its peak? a. Exercise challenge b. Ventilatory capacity c. Anaerobic threshold d. Maximal oxygen uptake ( O2max) ANS: D
Maximal oxygen uptake is designed to identify the level of exercise that causes the patient to reach maximum oxygen consumption. REF: pg. 199
OBJ: 29
43. Which of the following medications is used most commonly during bronchoprovocation
testing? a. Methacholine b. Isoproterenol c. Methylene blue d. Metaproterenol ANS: A
Inhaled histamine or methacholine is used most commonly to challenge the patient. REF: pg. 196
OBJ: 23
44. When bronchoprovocation testing is used, the FEV1 needs to decrease by what percentage
before a diagnosis of hyperactive airways is made? a. 10%
b. 20% c. 50% d. 66% ANS: B
A 20% decrease in the FEV1 indicates that the patient has reactive airways. REF: pg. 196
OBJ: 23
45. Obstructive lung disorders are characterized by a reduction in: a. Lung capacities. b. Lung volumes. c. Expiratory flows. d. Inspiratory flows. ANS: C
Obstructive disease is present if expiratory flow is below normal (<80% predicted). REF: pg. 187
OBJ: 13
46. A restrictive defect is present when lung volumes are reduced to less than what percentage of
predicted levels? a. 60% b. 70% c. 80% d. 90% ANS: C
Restrictive disease is present if the lung volume is below normal (<80% predicted, or smaller than 1 standard deviation [SD] below the mean). REF: pg. 187
OBJ: 13
47. Which of the following conditions are not common causes of restrictive lung disease? a. Atelectasis and obesity b. Interstitial lung disease and lung scarring c. Chest wall dysfunction and neurologic disease d. Asthma and emphysema ANS: D
This category of disease includes chest wall dysfunction, neurologic disease resulting in paralysis of the muscles of inspiration, dysfunction of the diaphragm, absent lung tissue, interstitial lung disease, atelectasis, and obesity. REF: pg. 187
OBJ: 13
48. Which of the following conditions is associated with a low FVC, low FEV1 but normal
FEV1/FVC, small TLC, low DLCO, and no response to a bronchodilator? a. Emphysema b. Chronic bronchitis c. Pulmonary fibrosis d. Pulmonary embolus
ANS: C
Pulmonary fibrosis usually is associated with a low FVC, low FEV1 but normal FEV1/FVC, small TLC, and low DLCO, and no response to a bronchodilator. REF: pgs. 187-188
OBJ: 13
Chapter 10: Chest Imaging Test Bank MULTIPLE CHOICE 1. Electrons coming off the cathode in an x-ray machine are focused to hit a small area of the
anode called the: a. tube. b. target. c. transformer. d. focusing plate. ANS: B
Electrons coming off the cathode wire are focused to hit a small area on the anode. This area is called the target. REF: pg. 208
OBJ: 1
2. The ability of x-rays to penetrate matter is inversely proportional to which of the following
physical properties of matter? a. Viscosity b. Density c. Weight d. Diffusion ANS: B
X-rays are not reflected asTliE ghStTraByA sN arK e,SbE utLtL heEyRp. enCeO trM ate most matter. Their ability to penetrate matter is dependent on the density of the matter. Dense objects such as bone absorb more x-rays (allow less penetration) than do air-filled objects such as lung tissue. REF: pg. 209
OBJ: 1
3. Which of the following tissues will look radiolucent on a chest radiograph? a. Blood b. Bone c. Lungs d. Liver ANS: C
X-rays that pass through low-density (air-filled) tissue strike the film in great numbers and turn it black (radiolucent). REF: pg. 209
OBJ: 3
4. Which of the following terms defines high-density tissues on the x-ray film? a. Radiolucent b. Radiopaque c. Radiographic d. Tomographic ANS: B
Radiolucent areas on the chest radiograph are seen as dark shadows. X-rays that strike bone are partially absorbed; therefore, fewer x-rays strike the film and less darkening of corresponding areas on the x-ray film is seen (radiopaque). Radiopaque areas are seen as white shadows on the film. REF: pg. 209
OBJ: 2
5. Which of the following tissues represent the four distinct densities recognized in the x-ray
(from higher to lower density)? a. Bone, water, air, fat b. Fat, bone, water, air c. Bone, water, fat, air d. Air, fat, water, bone ANS: C
Four distinct densities recognized on x-ray include bone (which is very dense), water (which is less dense), fat (which is mildly radiolucent), and air (which is very radiolucent). Most tissues in the body have a characteristic density based on the mixture within these materials. REF: pg. 210
OBJ: 3
6. As a patient who is being positioned for a chest x-ray is moved closer to the source of the
x-rays, what happens to the shadows of anatomic structures seen on the chest film? a. They get larger. b. They get smaller. c. They get clearer. d. They stay the same. ANS: A
X-rays leave the x-ray tube from a single point and scatter, so that they cover the whole x-ray film. This leads to enhanced magnification of shadows on the x-ray film if the patient is close to the x-ray tube, and less magnification if the patient is not close to the x-ray source. REF: pg. 209
OBJ: 4
7. To minimize the magnification associated with portable films, how far should the patient and
the film be from the source of the x-rays in a conventional chest x-ray? a. 6 feet b. 8 feet c. 10 feet d. 12 feet ANS: A
The patient and the film are positioned approximately 6 feet from the x-ray source for the conventional chest x-ray examination. At this distance, the magnification effect is minimized. Portable films are positioned approximately 4 feet from the x-ray source. This results in enhanced magnification, especially for structures in the body farthest from the x-ray film. REF: pg. 210
OBJ: 4
8. Which of the following statements is not considered an indication for obtaining a radiograph? a. Determine the appropriate therapy.
b. Evaluate the effectiveness of treatment. c. Detect alterations of the lung caused by pathologic processes. d. Diagnose specific diseases. ANS: D
Production of the chest radiograph has become one of the most popular and important procedures performed in the hospital. It can be used in the following ways: detecting alterations of the lung caused by pathologic processes; determining appropriate therapy; evaluating the effectiveness of treatment; determining the positioning of tubes and catheters; and observing the progression of lung disease. REF: pg. 210
OBJ: 4
9. What breathing instruction is a patient given immediately before an anteroposterior (AP) or
posteroanterior (PA) chest radiograph? a. Breathe normally. b. Exhale completely. c. Take a deep breath. d. Do not worry about breathing; it will not affect this x-ray. ANS: C
The patient is instructed to take a deep breath and hold it just before the x-ray is taken. REF: pg. 210
OBJ: 6
10. A male patient who is standing upright with his back to the x-ray tube, his anterior thorax
pressed against a metal cassette containing the film, and his arms positioned out of the way is positioned for what x-ray vTieEwS?TBANKSELLER.COM a. PA view b. AP view c. Apical lordotic view d. Lateral decubitus view ANS: A
Standard chest radiographs are taken in two directions. First, with the patient standing upright with his or her back to the x-ray tube, the anterior thorax is pressed against a metal cassette containing the film, and his or her arms are positioned out of the way. The patient is instructed to take a deep breath and hold it just before the x-ray is taken. The x-ray beam leaves the source, strikes the patient’s posterior chest, moves through the chest, exits through the front (anterior), and then strikes the film. Because the beam moves from posterior to anterior, this is called a posteroanterior (PA) view. REF: pg. 210
OBJ: 5
11. Which of the following views provides less cardiac magnification and a sharper view of the
left lower lobe? a. PA view b. AP view c. Left lateral view d. Lateral decubitus view ANS: C
Generally, a left lateral view (left side against the cassette) is preferred. The left lateral view provides less cardiac magnification than the right lateral view. REF: pg. 210
OBJ: 5
12. Which of the following views is used to see whether free fluid (pleural effusion or blood) is
present in the chest in the left pleural region? a. AP view b. Left lateral view c. Right lateral view d. Lateral decubitus view ANS: D
Other views are sometimes obtained to elucidate special problems. A lateral decubitus view is taken with the patient lying on the right or left side, to see whether free fluid (pleural fluid) is present in the chest. REF: pgs. 210-211
OBJ: 5
13. A lateral decubitus view is able to detect as little as a. 200 to 300 b. 100 to 200 c. 50 to 100 d. 25 to 50
mL of the pleural fluid.
ANS: D
As little as 25 to 50 mL of pleural fluid can be detected with the lateral decubitus view. REF: pg. 211
OBJ: 5
14. Which of the following views is most helpful in identifying a pneumothorax? a. Expiratory view b. AP view c. Left lateral view d. Lateral decubitus view ANS: A
The expiratory view can be helpful in the identification of a small pneumothorax. REF: pg. 211
OBJ: 5
15. AP portable films are obtained to evaluate all of the following except: a. Lung status. b. Lung tumor growth. c. Line and tubing positions. d. Results of invasive therapeutic procedures. ANS: B
AP portable films are obtained to evaluate lung status, to gain information on how well lines and tubing are positioned, and to see the results of invasive therapeutic maneuvers. REF: pg. 212
OBJ: 5
16. A portable film is ordered immediately after a patient is intubated in the emergency
department. Which of the following distances confirms proper placement of the endotracheal tube? a. 1 to 2 cm below the cricoid cartilage b. 5 to 7 cm above the carina c. 3 cm below the thyroid cartilage d. 3 to 5 cm above the carina ANS: D
The AP chest film often is used to evaluate the position of the endotracheal tube, to ensure that the inferior tip comes to rest appropriately 3 to 5 cm above the carina after intubation. REF: pg. 222
OBJ: 10
17. A portable film is ordered immediately after a central venous pressure (CVP) catheter is
placed via the subclavian vein. Which of the following is the correct placement site for the catheter? a. 1 to 2 cm above the right atrium b. Between the left and right ventricles c. At the confluence of the superior vena cava and the right atrium d. Anywhere in the superior vena cava ANS: C
The catheter is placed into the right or left subclavian or jugular vein and should come to rest just above the confluence of the superior vena cava and the right atrium of the heart. REF: pg. 222
OBJ: T1E 1 STBANKSELLER.COM
18. The presence of the characteristic “signet sign” in a computed tomography (CT) scan is
consistent with which of the following diseases? a. Asthma b. Lung tumor c. Emphysema d. Bronchiectasis ANS: D
Bronchiectasis shows up on a CT scan as a very characteristic “signet ring” pattern, with the blood vessel appearing to be a small stone set against the much larger ring of bronchial tissue. REF: pg. 227
OBJ: 9
19. In which of the following areas has magnetic resonance imaging (MRI) demonstrated
superiority over CT scanning? a. Evaluation of the hila b. Evaluation of diffuse, interstitial lung disease c. Evaluation of chest wall invasion by lung cancer d. Both a and b ANS: A
The only area in which MRI may be a little better than CT scanning is in the evaluation of mediastinal and hilar masses.
REF: pg. 226
OBJ: 13 | 14
20. Which of the following forms of radiologic assessment is most useful for studying the
distribution of ventilation and perfusion and the effects that diseases may have on these two important functions? a. MRI b. CT scanning c. Lung scanning d. Pulmonary angiography ANS: C
Lung scans are obtained by measuring gamma radiation emitted from the chest after radiopharmaceuticals are injected into the bloodstream and inhaled into the lung. Lung scanning is useful for studying the distribution of ventilation and perfusion and the effects that disease may have on these two important functions. REF: pg. 228
OBJ: 15
21. Which of the following studies is gradually replacing
scanning for the diagnosis of
pulmonary embolus? a. MRI b. CT scanning c. Lung scanning d. CT angiography ANS: D
CT angiography is gradualT lyErS epTlaBcA inN g KSELsL caEnR ni. ngCO foM r the diagnosis of pulmonary embolus. REF: pg. 230
OBJ: 15
22. The presence of “hot spots” in a positron emission tomography (PET) scan is indicative of: a. metabolically active tumor or infection. b. pneumothorax. c. pulmonary embolism. d. atelectasis. ANS: A
Tumors and areas of infection increase the local rate of metabolic activity and cause “hot” spots. This allows the clinician to determine whether a lung mass is metabolically active. REF: pg. 229
OBJ: 15
23. On a PA film, what number of posterior ribs visible above the diaphragm indicates a good
inspiratory effort? a. 10 b. 8 c. 6 d. 5 ANS: A
Finally, the degree of the patient’s inspiratory effort is evaluated by counting the posterior ribs visible above the diaphragm. On a PA film, 10 ribs indicates a good inspiratory effort. REF: pg. 213
OBJ: 5
24. The casting of a white shadow on a film is consistent with which of the following situations? a. Less exposed film b. More exposed film c. More radiolucent film d. More penetrated film ANS: A
The greater the density, the less the penetration. X-rays that do not penetrate fully are absorbed, resulting in less exposure of the film and the casting of a white shadow on the film. REF: pg. 212
OBJ: 2
25. What is the significance of the silhouette sign? a. It allows differentiation between alveolar and interstitial infiltrates. b. It can aid in detection of pleural effusion. c. It helps determine whether an infiltrate is in contact with a heart border. d. It allows differentiation between alveolar and interstitial opacities. ANS: C
The silhouette sign is useful primarily in determining whether a pulmonary infiltrate is in anatomic contact with a heart border or the diaphragm. REF: pg. 214
OBJ: T2ESTBANKSELLER.COM
26. Which of the following radiologic terms confirm the presence of intrapulmonary disease? a. Sulcus sign b. Compression atelectasis c. Air bronchogram d. Platelike atelectasis ANS: C
The presence of an air bronchogram confirms intrapulmonary disease, and the absence of an air bronchogram does not rule out intrapulmonary disease. REF: pg. 214
OBJ: 7
27. Which of the following radiologic findings is not consistent with lobar atelectasis? a. Collapse of lung tissue b. Shift in hilar structures toward the area of atelectasis c. Hemidiaphragm elevation d. Shift in hilar structures away from the area of atelectasis ANS: D
The chest radiograph in most cases demonstrates the loss of lung volume caused by atelectasis. Collapse of entire segments or lobes produces characteristic densities that show on the chest x-ray. Shift of the trachea, heart, and major thoracic vessels toward the affected side may be seen with lobar atelectasis.
REF: pg. 216
OBJ: 9
28. Which of the following clinical or chest x-ray findings is consistent with hyperinflation
caused by obstructive lung disease? a. Large retrosternal space b. Prolonged inspiratory phase c. Decreased resonance to percussion d. Decreased functional residual capacity (FRC) ANS: A
The most common cause of hyperinflation is obstructive lung disease. This is seen on chest x-rays as an increased AP diameter, a large retrosternal air space, and flattening of the diaphragm. REF: pg. 217
OBJ: 9
29. Which of the following would suggest that a patient has congestive heart failure (CHF)? a. Low and flat diaphragm b. Increased cardiothoracic ratio c. Tracheal deviation d. Increased retrosternal air space on a lateral film ANS: B
The ratio of the width of the heart at its greatest span to the width of the thorax is the cardiothoracic (C/T) ratio. Normally, this ratio does not exceed 0.5. In CHF, the heart enlarges and its width exceeds one half the width of the thoracic cage. REF: pg. 219
OBJ: 9
30. All of the following are typical findings of a small pleural effusion except: a. blunting of the costophrenic angle. b. a small meniscus sign. c. a partially obscured diaphragm. d. complete whiteout of the affected side. ANS: D
Small-volume findings include (1) blunting of the otherwise sharp angle between the chest wall and the point at which the diaphragm touches the chest wall laterally (the costophrenic [CP] angle) and (2) a small meniscus sign, a sign seen whenever fluid starts to fill the space between the lung and the chest wall, forming an opaque white crescent (meniscus) next to the chest wall. REF: pg. 221
OBJ: 9
31. Radiographic signs of consolidation include all of the following except: a. lobar distribution. b. minimal loss of volume. c. homogenous density late in the process. d. a low shift of the diaphragm. ANS: D
Radiographic signs of consolidation include the following: minimal loss of volume; lobar or segmental distribution (usually); homogenous density; and an air bronchogram if the airway leading to the consolidated area is open. REF: pg. 222
OBJ: 9
Chapter 11: Interpretation of Electrocardiogram Tracings Test Bank MULTIPLE CHOICE 1. Why is it important for a respiratory therapist (RT) to be able to identify cardiac
dysrhythmias? a. It is likely that the RT will observe the onset of the initial event b. The RT is primarily responsible for management of the dysrhythmia. c. The RT must be able to confirm the physician’s initial diagnosis. d. RTs are always the first responders for complaints of a cardiac nature. ANS: A
Given the hands-on nature of respiratory care, the likelihood that an RT may observe a patient during the acute onset of an ischemic cardiac event or a lethal dysrhythmia is high. Thus, it is vital for RTs to have basic knowledge in electrocardiogram (ECG) interpretation. REF: pg. 235
OBJ: 1
2. Why is an ECG tracing so useful? a. It reflects the heart’s pumping ability. b. It can identify structural abnormalities in the heart. c. It can be used to diagnose malfunctioning valves. d. It can aid in the diagnosis of cardiac tissue ischemia. ANS: D
If a patient presents with dT ysEpS neTaBaA ndNcKhS esEt L diL scEoR m.foCrtO, M an ECG can aid in the diagnosis of an ischemic cardiac event. It is important to note that the ECG tracing does not measure the pumping ability of the heart. It is not unusual for a patient with low cardiac output to have a normal ECG tracing. This is because the ECG does not directly depict abnormalities in cardiac structure such as defects in the heart valves or the interventricular septum. REF: pg. 235
OBJ: 1 | 2
3. Which of the following symptoms is least suggestive of the need for an ECG? a. Fever b. Orthopnea c. Chest pain d. Fainting spells ANS: A
Orthopnea, chest pain, and fainting spells all are clinical findings that suggest the need for an ECG; fever is not (see Box 11-1, pg. 236). REF: pg. 236
OBJ: 3
4. Which chamber of the heart initially receives deoxygenated blood from the vena cava? a. Left atrium b. Left ventricle c. Right atrium
d. Right ventricle ANS: C
The right atrium receives deoxygenated blood from the vena cava and directs the blood into the right ventricle. REF: pg. 236
OBJ: 3
5. Which chamber is responsible for pumping blood into the pulmonary circulation? a. Left atrium b. Left ventricle c. Right atrium d. Right ventricle ANS: D
Right ventricular contraction ejects blood into the pulmonary artery, which carries blood to the lungs for oxygenation. REF: pg. 236
OBJ: 3
6. Which chamber normally has the largest muscle mass? a. Left atrium b. Left ventricle c. Right atrium d. Right ventricle ANS: B
Because the left side of the heart pumps blood throughout the entire body, it normally has a significantly larger muscle mass than the right side. REF: pg. 236
OBJ: 3
7. Where does the normal electrical impulse originate for each heartbeat? a. Atrioventricular (AV) node b. Sinoatrial (SA) node c. Bundle of His d. Right bundle branch ANS: B
Normally, the electrical activity of the heart is initiated in the sinus or sinoatrial (SA) node, which is located in the right atrium (see Figure 11-2, pg. 236). REF: pg. 236
OBJ: 5
8. What term describes heart cells that have the ability to spontaneously depolarize? a. Systole b. Purkinje c. Automaticity d. Myocardiocity ANS: C
Cells that have the ability to generate electrical activity spontaneously are said to exhibit automaticity.
REF: pg. 236
OBJ: 5
9. Why is the electrical signal delayed slightly at the AV node?
1. To allow better filling of the ventricles 2. To protect the ventricles against excessively rapid atrial rates 3. To provide time for atrial contraction prior to ventricular systole 4. To prevent premature atrial beats from reaching the ventricles a. 1, 3 b. 2, 4 c. 1, 2 d. 1, 2, 3, 4 ANS: C
Once the electrical impulse reaches the AV node, it is delayed for approximately 0.1 second before it passes on into the bundle of His. The delay is believed to serve the purpose of allowing more complete filling of the ventricles before ventricular contraction, which occurs as the result of atrial contraction. In addition, the AV node can protect the ventricles from excessively rapid atrial rates that the ventricles could not tolerate. REF: pg. 237
OBJ: 5
10. What is the ventricular heart rate when the AV node paces the heart? a. 20 to 40 beats/min b. 40 to 60 beats/min c. 60 to 80 beats/min d. 80 to 100 beats/min ANS: B
If the SA node fails to function properly and does not pace the heart, the AV junction can serve as the pacemaker for the ventricles. When this occurs, the ventricular rate is usually between 40 and 60 beats/min and the ECG reveals a distinct pattern, as is described later in this chapter (see Figure 11-3, pg. 237). REF: pg. 237
OBJ: 5
11. What term applies to any heartbeat that originates outside the SA node? a. Ectopic impulse b. Aberrant impulse c. Eccentric impulse d. Recurrent impulse ANS: A
Any impulse that originates outside the SA node is called an ectopic impulse, and the site from which the ectopic impulse originates is called the focus. REF: pg. 239
OBJ: 5
12. Which of the following physiologic effects is associated with myocardial infarction? a. Arterial hypertension b. Increased cardiac output c. Good blood flow to the brain
d. Backup of blood into the lungs ANS: D
Infarction of a major portion of the left ventricle is likely to cause significant arterial hypotension, an abnormal sensorium, and a backup of blood into the pulmonary circulation. REF: pg. 239
OBJ: 11
13. Which of the following are associated with the onset of dysrhythmias?
1. Hypoxia 2. Electrolyte imbalances 3. Sympathetic stimulation 4. Pulmonary hypertension a. 1, 2 b. 3, 4 c. 2, 4 d. 1, 2, 3 ANS: D
Dysrhythmias can be caused by hypoxia, ischemia, sympathetic stimulation, drugs, electrolyte imbalances, abnormal heart rate, and abnormally stretched or dilated cardiac chambers. REF: pg. 239
OBJ: 9
14. What term describes the sudden loss of the negative charge inside the myocardial cells? a. Repolarization b. Depolarization c. Automaticity d. Conductivity ANS: B
Depolarization occurs when a polarized cell is stimulated. Polarized cells carry an electrical charge on their surface, the inside of the cell being more negatively charged than the outside of the cell. The sudden loss of the negative charge within the cell is called depolarization. REF: pg. 240
OBJ: 4
15. What is represented by the P wave on the ECG tracing? a. Repolarization of the atria b. Depolarization of the atria c. Repolarization of the ventricles d. Depolarization of the ventricles ANS: B
Depolarization of the atria creates the initial wave of electrical activity detected on the ECG tracing, known as the P wave (see Figure 11-5, pg. 240). Because the atria usually are small, they generate less voltage than the ventricles, and the resulting P wave is small. REF: pg. 240
OBJ: 6
16. What does the QRS complex represent? a. Depolarization of the atria
b. Repolarization of the atria c. Depolarization of the ventricles d. Repolarization of the ventricles ANS: C
Depolarization of the ventricles is represented by the QRS complex. Because the ventricular muscle mass is larger than the atria and produces more voltage during depolarization, the QRS complex normally is taller than the P wave in most cases (see Figure 11-5, pg. 241). REF: pg. 241
OBJ: 6
17. At what speed does the paper travel through the ECG machine? a. 25 mm/sec b. 30 mm/sec c. 40 mm/sec d. 50 mm/sec ANS: A
Time is measured on the horizontal axis of the ECG paper. The ECG paper moves through the electrocardiograph at a speed of 25 mm/sec. REF: pg. 242
OBJ: 8
18. How much time is represented on the horizontal axis of the ECG paper by five large boxes? a. 0.5 second b. 1 second c. 3 seconds d. 5 seconds ANS: B
Each small square (1 mm) represents 0.04 second, and each larger square (5 mm) represents 0.2 second. Five large boxes represent 1 second. REF: pg. 242
OBJ: 6
19. What is the upper limit in height of the normal P wave? a. 1 mm b. 2.5 mm c. 3.5 mm d. 5 mm ANS: B
The normal P wave is less than 2.5 mm in height and is not more than 0.10 second in length. REF: pg. 242
OBJ: 6
20. What is the normal range for the PR interval? a. 0.05 to 0.1 second b. 0.12 to 0.2 second c. 0.2 to 0.35 second d. 0.25 to 0.5 second ANS: B
The normal PR interval is between 0.12 and 0.2 second (three to five small boxes). REF: pg. 242
OBJ: 6
21. The normal QRS complex does not exceed what time on the horizontal axis? a. 0.05 second b. 0.10 second c. 0.5 second d. 1 second ANS: B
Normally, the QRS interval does not exceed 0.12 second (three small boxes). REF: pg. 248
OBJ: 7
22. What is the heart rate if the RR interval is five large boxes apart? a. 40 beats/min b. 60 beats/min c. 90 beats/min d. 120 beats/min ANS: B
The RR interval is useful in identifying the rate and regularity of ventricular contraction. The distance in millimeters is determined from one R wave to the next in successive QRS complexes. This is done for several different RR intervals. The average of the measurements is determined and converted to time. Remember that each large box is equal to 0.2 second and five large boxes equal 1 second. If the RR interval is 1 second, the heart rate is 60 beats/min. REF: pg. 243
OBJ: 8
23. In the terminology for three of the limb leads—aVR, aVL, and aVF—what does the letter “A”
stand for? a. Atrial b. Assisted c. Ambient d. Augmented ANS: D
The three limb leads—aVR, aVL, and aVF—are called augmented leads because the ECG machine must amplify the tracings to get an adequate recording. REF: pg. 244
OBJ: 8
24. What part of the heart is best viewed by the chest leads V5 and V6? a. Left atrium b. Right atrium c. Left ventricle d. Right ventricle ANS: C
V5 and V6 lie over the left ventricle.
REF: pg. 245
OBJ: 7
25. What is considered to be the normal position of the axis in a healthy patient’s heart? a. 0 to –90 degrees b. 0 to 90 degrees c. 60 to 120 degrees d. 120 to 180 degrees ANS: B
Normally, the mean QRS axis (vector) points leftward (patient’s left) and downward, somewhere between 0 and +90 degrees in the frontal plane previously described (see Figure 11-12, pg. 245). REF: pg. 245
OBJ: 8
26. What axis change is common in patients with chronic pulmonary hypertension? a. Left axis deviation b. Right axis deviation c. Inverted axis deviation d. Extreme left axis deviation ANS: B
Right axis deviation is important to recognize early in the care of the patient because it often indicates significant chronic pulmonary hypertension. This most often is related to chronic hypoxemia from chronic obstructive pulmonary disease. REF: pg. 246
OBJ: 10
27. What problem is indicated by elevation of the ST segment? a. Heart block b. Myocardial ischemia c. Elevation of serum potassium d. Depletion of serum potassium ANS: B
Inspect the ST segment in all leads. ST segment elevation may indicate myocardial injury, whereas ST segment depression may indicate myocardial ischemia. REF: pg. 258
OBJ: 9
28. What is the upper limit of normal for the width of a normal Q wave? a. 0.08 second b. 0.04 second c. 1.2 seconds d. 1.4 seconds ANS: B
Evaluate the Q wave. A Q wave is considered normal (or physiologic) if it is less than 0.04 second (40 msec) wide and less than one-third the amplitude of the R wave. Q waves that exceed either of these values are considered pathologic and indicate a new or possibly old infarction.
REF: pg. 248
OBJ: 8
29. What is indicated by high-voltage R waves in the QRS complex? a. Axis deviation b. Atrial enlargement c. Myocardial ischemia d. Ventricular enlargement ANS: D
Look for signs of chamber enlargement. High-voltage R waves in the precordial leads indicate ventricular hypertrophy. REF: pg. 248
OBJ: 8
30. Which of the following are common causes of tachycardia?
1. Pain 2. Fever 3. Hypoxemia 4. Hypothyroidism a. 1, 3 b. 2, 4 c. 1, 2, 3 d. 1, 2, 3, 4 ANS: C
Fever, pain, hypoxemia, hypovolemia, hypotension, sepsis, and heart failure are causes of sinus tachycardia. REF: pg. 249
OBJ: 9
31. What dysrhythmia is characterized by a sawtooth pattern of waves between normal QRS
complexes on the ECG tracing? a. Atrial flutter b. Atrial fibrillation c. Ventricular flutter d. Ventricular fibrillation ANS: A
Atrial flutter is a dysrhythmia that produces a very distinctive ECG pattern, usually caused by a rapidly firing ectopic site in the atria that presents as a characteristic sawtooth pattern between normal-appearing QRS complexes. REF: pg. 250
OBJ: 9
32. Which of the following would describe the ECG of a patient with atrial fibrillation? a. Regular ventricular response b. Large bizarre QRS complexes c. Chaotic baseline between QRS complexes d. No identifiable QRS complexes ANS: C
In atrial fibrillation, the electrical activity of the atria is completely chaotic and lacks coordination because it is arising from multiple ectopic sites within the atria. The ECG tracing shows a chaotic baseline between QRS complexes, with no regular pattern or organization. REF: pg. 251
OBJ: 9
33. Which of the following is a hallmark of a premature ventricular contraction (PVC)? a. The QRS complex is early but normal in appearance. b. The QRS complex is wider than normal. c. There is a P wave in front of the QRS complex. d. The T wave moves in the same direction as the QRS complex. ANS: B
No P wave is associated with a PVC. The QRS complex comes early, exceeds 0.12 second in width, and has abnormal configuration. The T wave following the PVC is deflected in a direction opposite to that of the QRS complex. REF: pg. 253
OBJ: 9
34. What dysrhythmia often follows sustained ventricular tachycardia? a. Heart block b. Atrial fibrillation c. Elevated ST segments d. Ventricular fibrillation ANS: D
Sustained ventricular tachycardia may lead to ventricular fibrillation, particularly if untreated. REF: pg. 253
OBJ: 9
35. Which of the following dysrhythmias is associated with the lowest cardiac output? a. AV block b. Atrial fibrillation c. Ventricular flutter d. Ventricular fibrillation ANS: D
In ventricular fibrillation, the heart cannot pump blood, the cardiac output drops to zero, and the patient becomes unconscious immediately. REF: pg. 253
OBJ: 9
36. Which of the following dysrhythmias represents a dissociation of the mechanical and
electrical activities of the heart? a. Asystole b. AV block c. Bradycardia d. Pulseless electrical activity (PEA) ANS: D
PEA is not a discrete dysrhythmia but rather an electromechanical condition that can be diagnosed clinically. As the name implies, there is a dissociation of the electrical and mechanical activity of the heart. In other words, the pattern that appears on the ECG monitor does not generate a pulse. REF: pg. 254
OBJ: 9
37. Which of the following is a characteristic of third-degree heart block? a. There is a lengthening PR interval. b. There is often a 2:1 ratio between P waves and QRS complexes. c. The atrial rate is the same as the ventricular rate. d. There is no relationship between the P waves and the QRS complexes. ANS: D
This block does not allow any conduction of stimuli from the atria to the ventricles. In this situation, the ventricles and the atria beat independently of one another. Thus no pattern is distinguishable between the atria and the ventricles. REF: pg. 256
OBJ: 9
38. What is the normal progression of abnormalities seen on the ECG for a patient who is having
a myocardial infarction? a. ST elevation followed by large Q waves b. ST depression followed by large Q waves c. Large Q waves, ST elevation, followed by ST depression d. ST depression, large Q waves, and then ST segment elevation ANS: A
The typical pattern of acute myocardial injury is ST segment elevation in the leads that reflect electrical activity of the corresponding injured heart tissue (see Figure 11-32, pg. 258). In general, the degree of damage to the heart caused by the ischemia determines the degree of ST segment elevation. The ST segment abnormality usually resolves when perfusion is restored. At some point after myocardial infarction, significant Q waves (0.04 second in length) are seen on the ECG in the corresponding leads. Q waves may develop within hours of an infarction but may not evolve for several days in some patients. They persist for the remainder of the patient’s life. REF: pg. 258
OBJ: 11
39. Which of the following would be a typical ECG finding in a patient with emphysema? a. Reduced voltage in the limb leads b. Left axis deviation c. Smaller-than-normal P waves d. Prominent QRS complexes, particularly in leads V5 and V6 ANS: A
Hyperinflation of the lungs, as occurs with emphysema, impairs passage of the electrical activity of the heart through the lung. This causes reduced voltage to be seen in the limb leads. REF: pg. 259
OBJ: 10
40. A patient who has atrial fibrillation is at risk for which of the following pulmonary
conditions? a. Chronic obstructive pulmonary disease (COPD) b. Pneumothorax c. Pleural effusion d. Pulmonary emboli ANS: D
Similar to atrial flutter, patients with atrial fibrillation are at greater risk for mural thrombi formation and embolization due to blood stagnation in the atria. As a result, RT's and other clinicians should be on the alert for pulmonary emboli and stroke in such patients. REF: pg. 251
OBJ: 10
41. A patient complaining of chest pain that has not been relieved by nitroglycerin should be
treated with all of the following except: a. oxygen therapy b. thrombolytic therapy c. stress testing d. surgical intervention ANS: C
The AHA recommends that for chest pain or associated symptoms not relieved by nitroglycerin, a myocardial infarction should be suspected until proven otherwise. Remember that "time is muscle," and treatment interventions, such as thrombolytic therapy or surgical intervention, should be implemented quickly to minimize damage to the myocardium. REF: pg. 259
OBJ: T1E 2 STBANKSELLER.COM
Chapter 12: Neonatal and Pediatric Assessment Test Bank MULTIPLE CHOICE 1. What range of gestational weeks is considered term? a. 32 to 36 b. 34 to 40 c. 37 to 42 d. 38 to 44 ANS: C
Term infants are born at between 37 and 42 weeks of gestational age. REF: pg. 264
OBJ: 1
2. Which of the following conditions is associated with an infant born early in gestation? a. Meconium aspiration b. Respiratory distress syndrome (RDS) c. Persistent pulmonary hypertension d. Perinatal asphyxia ANS: B
A disease of prematurity is RDS. REF: pg. 265
OBJ: 3
3. You are looking in the chart of your patient, and in the maternal history, you see the
following: G3, P2, Ab0. Which of the following statements is true? a. The mother is in her third pregnancy. b. The mother is in her second pregnancy. c. The mother has delivered three healthy children. d. The mother has had one abortion. ANS: A
Gravida is a pregnant woman, para is a woman who delivers a live infant, and abortion is the delivering of a dead infant. These terms most likely will be abbreviated and followed by numbers (e.g., G2, P1, Ab0, meaning that this woman is in her second pregnancy, has delivered a living infant, and has not had any abortions). REF: pg. 265
OBJ: 1
4. Which of the following is part of the biophysical evaluation of a fetus performed by
ultrasound? a. Reactive heart rate b. Fetal brain development c. Fetal gastrointestinal function d. Body size ANS: A
The biophysical profile reflects an ultrasound evaluation of fetal breathing, body movement, tone, reactive heart rate, and amniotic fluid volume. REF: pg. 267
OBJ: 3
5. What do the L/S ratio and phosphatidylinositol (PI) and phosphatidylglycerol (PG) levels
assess? a. Cardiovascular maturation b. Lung maturation c. Renal system maturation d. Overall level of fetal maturation ANS: B
The L/S ratio is the ratio of two phospholipids: lecithin and sphingomyelin. Increasing levels of lecithin indicate improving maturation of the lung’s surfactant system. Similar to lecithin, PI and PG become present with advancing maturation of the lung. Their presence usually is indicative of lung maturation. REF: pg. 267
OBJ: 1
6. What Apgar parameter usually deteriorates first in a hypoxic infant? a. Respiratory effort b. Heart rate c. Muscle tone d. Skin color ANS: D
If an infant is becoming asT phEyS xiTaB teA d,NthKeSsE igL nsLm EeRa.suCreOdMby the Apgar score generally decline in a particular order (color, reflex irritability, muscle tone, respiratory effort, and heart rate). REF: pg. 266
OBJ: 2
7. Infants who need extensive medical resuscitation at birth will have Apgar scores in the range
of: a. b. c. d.
9 to 10. 6 to 8. 4 to 6. 0 to 3.
ANS: D
Moderately depressed infants have 1-minute scores of 4 to 6. They need more than routine care and often need an increased FIO2 (fraction of inspired oxygen) with bag-and-mask ventilation. Most infants respond well to this therapy and improve in a few minutes. Infants who have 1-minute scores of 0 to 3 are severely depressed and need extensive medical care such as intubation and mechanical ventilation. REF: pg. 266
OBJ: 2
8. What does the Ballard examination of a newborn infant assess? a. Lung maturation b. Transition from fetal to extrauterine circulation c. Gestational age
d. Cerebral function ANS: C
If an infant is small or was born prematurely, or if there is uncertainty about gestational age, an assessment to determine gestational age should be performed. Most nurseries currently use a Ballard examination for this assessment. REF: pg. 269
OBJ: 1 | 2
9. Which of the following is a common cause of hypothermia in an infant? a. Infection b. Heart failure c. Atelectasis d. Liver disease ANS: A
Hypothermia is a more common and significantly more serious sign of infection in a newborn than in an older child or adult. Hypothermia probably occurs because the infant is unable to maintain normal heat production during an acute infection. REF: pg. 270
OBJ: 3
10. What is the upper limit of the normal range for the heart rate in the newborn? a. 120 beats/min b. 140 beats/min c. 160 beats/min d. 180 beats/min ANS: C
The normal pulse rate for infants is age and size dependent and usually is between 100 and 160 beats/min; it is a function of the developmental age of the infant. REF: pg. 270
OBJ: 1
11. Which of the following are typical causes of tachypnea in the newborn?
1. Hypothermia 2. Hypoxemia 3. Respiratory acidosis 4. Pain a. 2, 3, 4 b. 2, 4 c. 1, 3 d. 1, 2, 3, 4 ANS: A
In newborns, tachypnea can be caused by hypoxemia, metabolic and respiratory acidosis, congenital heart disease, anxiety, pain, hyperthermia, and crying. REF: pg. 271
OBJ: 3
12. What is the upper limit for normal systolic blood pressure in the term newborn? a. 60 mm Hg
b. 70 mm Hg c. 80 mm Hg d. 90 mm Hg ANS: B
Usually, a term infant’s systolic blood pressure should be no higher than 70 mm Hg, with diastolic pressure no higher than 50 mm Hg. REF: pg. 272
OBJ: 3
13. What is indicated by the presence of retractions in the newborn? a. A compliant chest wall b. Stiff lungs c. Reduced airway resistance d. Heart failure ANS: B
Retractions (sinking inward of the skin around the chest wall during inspiration) occur when the lung’s compliance is less than the compliance of the chest wall or when airway obstruction is significant. Thus, retractions are a sign of an increase in the work of breathing. REF: pg. 273
OBJ: 4
14. What is the significance of “grunting” in an infant? a. A low functional residual capacity (FRC) b. Increased airway resistance c. Significant secretion retention d. High compliance ANS: A
Infants who have a disease that decreases the FRC attempt to increase their lung volume by holding their tidal volume at end-inspiration. The infant accomplishes this by occluding the airway with glottic closure and actively exhaling against the closed glottis after the end of inspiration. The grunting sound is produced when the infant suddenly opens the glottis and quickly exhales, inhales, and again closes the glottis. REF: pg. 274
OBJ: 4
15. What is indicated by a capillary refill greater than 3 seconds in the infant? a. Normal cardiopulmonary function b. Respiratory failure c. Circulatory failure d. Renal failure ANS: C
To check skin perfusion, or capillary refill, the examiner should blanch the infant’s skin with a finger and note how long it takes for the blanched skin to recover its color. Capillary refill is checked on the trunk and extremities. Capillary refill should be less than 3 seconds and will be greater than 3 seconds if the infant has a low cardiac output. REF: pg. 274
OBJ: 4
16. What effect does abdominal distention have on respiration? a. Impedes diaphragm movement b. Decreases the work of breathing c. Does not have any impact on respiration d. Improves expiration and lung stabilization ANS: A
An infant’s abdomen and abdominal organs move significantly with respiration because the diaphragm is the major source of power for respiration and the abdominal wall musculature is relatively weak. Anything that impedes the motion of the abdomen or its organs hinders the infant’s respiration. REF: pg. 274
OBJ: 3
17. An infant’s entire right hemithorax “lights up” during transillumination. What does this
signify? a. Normal lung tissue b. Significant pleural effusion c. Significant pneumothorax d. Intensity of the light is set too high ANS: C
Transillumination is a technique that often is used in examining infants because their chest wall is thin enough to shine a light through. Normally, this produces a lighted halo around the point of contact with the skin. In the presence of a pneumothorax or a pneumomediastinum, the entire hemithorax lights up. REF: pg. 275
OBJ: T8ESTBANKSELLER.COM
18. What is the normal leukocyte count for newborns? a. 5000 to 15,000/mm3 b. 9000 to 30,000/mm3 c. 15,000 to 40,000/mm3 d. 21,000 to 35,000/mm3 ANS: B
The white blood cell (WBC) count in infants tends to be higher than in older patients. Normal values vary with the chronologic age of the infant. At birth, this ranges from 9000 to 30,000 mm3. REF: pg. 276
OBJ: 9
19. What is indicated by leukopenia in an infant? a. Overwhelming infection b. Chronic infection c. Acute infection d. Local infection ANS: A
Leukopenia, particularly neutropenia (an absolute neutrophil count of less than 2000 neutrophils/mm3), is an ominous sign. Usually, neutropenia indicates an infection and implies that the infant is being overwhelmed.
REF: pg. 277
OBJ: 9
20. Why is it so important to quickly identify a low blood glucose level in a newborn? a. Because of worry about impaired pulmonary function b. Because it is detrimental to brain development c. Because it results in rapid renal deterioration d. Because it can result in blindness through retinal detachment ANS: B
The blood glucose level probably is the most frequent blood chemistry determination made in newborns. This simple test is of tremendous importance because hypoglycemia, or low serum glucose, is as detrimental to the developing newborn’s brain as hypoxia. Because the exact levels and the length of time necessary to cause damage to the central nervous system have not been determined, most physicians treat an infant with a glucose level of less than 40 mg/dL. REF: pg. 278
OBJ: 11
21. What clinical problem is associated with low serum levels of calcium and phosphorus? a. Acute hypoxia b. Liver failure c. Poor nutrition d. Renal failure ANS: C
Calcium and phosphorus levels are of indirect importance in the evaluation of a newborn with chronic lung disease. InfanTtsEwSiT thBbA roNnK chSoE puLlL mE onRa. ryCdOyM splasia or other chronic lung diseases have increased work of breathing and increased metabolic and nutritional needs. The metabolism of calcium and phosphorus provides a valuable clue to the nutritional status of the chronically ill infant. A chronically ill infant with poor nutrition has low levels of calcium and phosphorus and an increased risk of developing rickets. REF: pg. 279
OBJ: 11
22. What is the normal range for PaO2 at birth? a. 40 to 60 mm Hg b. 50 to 70 mm Hg c. 60 to 80 mm Hg d. 70 to 90 mm Hg ANS: B
Arterial blood samples are the most reliable source for blood gas analysis in newborns. Normal values in newborns depend on the age of the infant when the blood is drawn. A normal newborn PaO2 is 50 to 70 mm Hg. REF: pg. 279
OBJ: 14
23. What parameter demonstrates the largest difference when capillary blood is compared with
arterial blood? a. PO2 b. PCO2
c. pH d. HCO3– ANS: A
When the values of capillary carbon dioxide tension (PcCO2) and pH are compared with those obtained by arterial sample, the carbon dioxide level is 2 to 5 mm Hg higher and the pH is 0.01 to 0.03 unit lower. These small differences are inconsequential in most clinical situations. However, when the values for capillary oxygen tension (PcO2) are compared with those obtained by arterial samples, the differences are not so slight. Unfortunately, there is no fixed ratio for PaO2/PcO2. An infant with a PcO2 of 50 mm Hg may have a PaO2 of 50 to 90 mm Hg or higher. The only statement that can be made about PaO2 when only PcO2 is known is that PaO2 is no lower than PcO2. Bicarbonate is calculated in accordance with the Henderson-Hasselbalch equation. REF: pg. 280
OBJ: 14
24. Which of the following conditions is most likely to cause a falsely low tcPO2 reading? a. Alkalosis b. Hypertension c. Hypovolemia d. Hyperthermia ANS: C
Any condition that decreases blood flow under the electrode, such as acidosis, shock, hypovolemia, or hypoglycemia, can cause tcPO2 to be falsely lower than PaO2. REF: pg. 281
OBJ: 15
25. Which lung volumes can be measured easily in newborns?
1. Thoracic gas volume 2. Residual volume 3. Functional residual capacity 4. Crying vital capacity a. 1, 3, 4 b. 2, 44 c. 1, 3 d. 1, 2, 3, 4 ANS: A
Three volumes can be measured easily in newborns independent of their cooperation: the FRC, thoracic gas volume (TGV), and crying vital capacity (CVC). REF: pg. 282
OBJ: 17
26. Which of the following clinical findings suggests the need for a chest radiograph in the infant?
1. Cyanosis 2. Unexplained tachypnea 3. Abnormal breath sounds 4. Worsening clinical status a. 1, 3, 4 b. 2, 4
c. 1, 3 d. 1, 2, 3, 4 ANS: D
Chest radiographs should be done in infants who have unexplained tachypnea, cyanosis, abnormal breath sounds, malformation of the chest or airway, or a sick appearance. In addition, any infant who has a significant worsening of his or her clinical status should have a chest radiograph. REF: pg. 284
OBJ: 19
27. What differentiates pneumonia from transient tachypnea of the newborn (TTNB) on chest
radiograph? a. Rapid resolution of TTNB b. TTNB will have a ground-glass appearance c. Silhouette sign d. Diffuse streakiness and fluid in the major and minor fissures ANS: A
The characteristic chest radiograph shortly after birth shows diffuse streakiness and fluid in the major and minor fissures. This is impossible to distinguish from the chest radiograph of pneumonia. The characteristic of TTNB results from rapid resolution of the disease. By 24 hours of age, the newborn’s chest radiograph is typically normal. REF: pg. 284
OBJ: 19
28. A child presents at the emergency department with difficulty breathing and a harsh, barking
cough. The history revealsTaEruSnTnB yA noNsK eS ovEeL rL thE eR pa.sC t fO ew M days. What is the most likely diagnosis? a. Croup b. Epiglottitis c. Asthma d. Bronchopulmonary dysplasia (BPD) ANS: A
Croup usually is a viral disease that affects the trachea and small airways. The child may have cold symptoms such as a stuffy or runny nose for a few days. He or she also may have a fever. These symptoms progress to a loud, seal-like barking cough and rapid and/or difficult respirations. REF: pg. 288
OBJ: 20
29. A 4-year-old presents to the emergency room with difficulty breathing, a dry cough, and
audible inspiratory and expiratory wheezing. The parents report the child was playing outside with siblings prior to the difficulty breathing. The symptoms have been progressively worsening. What is the most likely diagnosis? a. Epiglottitis b. Asthma c. Croup d. Pneumonia ANS: B
Asthma is a chronic airway disease that is caused by airway inflammation and hyper-responsiveness to irritants. The typical symptoms include intermittent dry cough and expiratory wheezing. Young children may report non-focal chest pain. Older children may report shortness of breath and chest tightness. Severe exacerbations may include airflow obstruction that can be life threatening. REF: pg. 289
OBJ: 20
30. Which of the following conditions is a result of a bacterial disease? a. Asthma b. Croup c. Epiglottitis d. Respiratory Distress Syndrome (RDS) ANS: C
Epiglottitis is usually a bacterial disease that causes significant edema and inflammation of the epiglottis. REF: pg. 289
OBJ: 20
Chapter 13: Older Patient Assessment Test Bank MULTIPLE CHOICE 1. Ageism in the healthcare setting can lead to: a. practitioners spending more time with older patients. b. practitioners listening to older patients less carefully. c. practitioners treating older patients with compassion. d. better communication with elderly patients. ANS: B
The connection of aging with chronic illness and death may lead to ageism, or discrimination against old people. Ageism in the healthcare setting can cause practitioners to listen to older patients less carefully, spend less time addressing their concerns, and treat them mechanically, without compassion. REF: pg. 297
OBJ: 1
2. Which of the following best describes the manner in which a clinician should address the
older adult patient? a. Use the patient’s first name to promote an atmosphere of equality. b. Address the patient by his or her last name with the appropriate title. c. Approach the patient in a matter-of-fact, business-like manner. d. Avoid physical contact with the elderly. ANS: B
It is always important to approach older adults in a caring manner. A smile, a pat on the hand, and a kind word while making eye contact usually are successful forms of nonverbal communication. When addressing the patient, use his or her last name preceded by the appropriate title (e.g., Mrs., Miss, Mr., Dr., Father). Using the patient’s last name shows respect and promotes an atmosphere of equality. REF: pg. 297
OBJ: 1
3. In the 65- to 75-year-old age group, about what percentage of adults experience hearing loss? a. 23% b. 32% c. 37% d. 40% ANS: A
Presbycusis, an age-related, progressive, bilateral hearing loss, is the most common cause of auditory impairment in the United States. This condition affects about 23% of adults between the ages of 65 and 75 years. REF: pg. 298
OBJ: 2
4. By age 79 years, about what percentage of older patients have hearing impairment? a. 30%
b. 40% c. 50% d. 60% ANS: C
In the 70- to 80-year-old age group, as many as 50% of older adults have hearing impairment that actually affects their communication skills. REF: pg. 298
OBJ: 2
5. Ototoxicity has a damaging effect on which cranial nerve? a. Fifth cranial nerve b. Sixth cranial nerve c. Seventh cranial nerve d. Eighth cranial nerve ANS: D
Ototoxicity is defined as a damaging effect on the eighth cranial nerve or on the organs of hearing or balance. REF: pg. 298
OBJ: 2
6. Age is the major risk factor for developing which of the following eye disorders? a. Cataract b. Glaucoma c. Vascular disorders d. Diabetic retinopathy ANS: A
Age is the major risk factor for developing cataract, an opacity of the lens that reduces visual acuity. Patients who live to an advanced age are likely to develop cataract. REF: pg. 299
OBJ: 3
7. Which heart chamber typically thickens with aging? a. All four chambers b. The left atrium c. The left ventricle d. The right ventricle ANS: C
Many older adults do have some measure of cardiac hypertrophy, but it generally is accepted that in the absence of disease, alteration in the size of the heart is minimal. Because of decreased contractile properties in the heart and blood vessels, an age-related increase in systolic blood pressure occurs, along with an elevated left ventricular afterload, which results in left ventricular wall thickening. REF: pg. 300
OBJ: 4
8. What fraction of patients older than 70 years have calcium deposits in their aortic or mitral
valves? a. One-fourth
b. One-third c. One-half d. Two-thirds ANS: B
Calcification of heart valves is more prevalent in the elderly. In elderly patients, the predominant causes of valvular heart disease include degenerative calcification, myxomatous degeneration, papillary muscle dysfunction, and infective endocarditis. One-third of patients older than 70 years of age have calcium deposits in the aortic or mitral valves. REF: pg. 300
OBJ: 4
9. How much does the incidence of congestive heart failure (CHF) increase with each decade of
life between 45 and 75 years? a. Doubles b. Triples c. Quadruples d. Quintuples ANS: A
CHF may result from valvular disease, hypertension, cardiomyopathy, or ischemic heart disease. The incidence of CHF doubles for each decade of life between 45 and 75 years. REF: pg. 300
OBJ: 4
10. Which of the following statements is true about the effects of aging on the pulmonary system?
1. Alveolar septa gradually deteriorate. raE duSaTllB yA deNcK reSasEeL s.LER.COM 2. The number of alveoli gT 3. Loss of alveolar walls reduces the surface area for gas exchange. 4. Smooth muscle fibers in the lungs are progressively replaced with fibrous connective tissue. a. 1, 2, 3 b. 2, 4 c. 1, 3 d. 1, 3, 4 ANS: D
A number of age-related physiologic changes are seen in the pulmonary system. Smooth muscle fibers in the lungs are progressively replaced with fibrous connective tissue. Alveolar septa deteriorate gradually. Although the number of alveoli does not change, loss of alveolar walls increases the size of the alveoli and reduces surface area for gas exchange. REF: pg. 300
OBJ: 4
11. What is the physiologic effect of aging on pulmonary function? a. Decreased residual volume (RV) b. Increased functional residual capacity (FRC) c. Increased tidal volume (VT) d. Decreased total lung capacity (TLC) ANS: B
The stiffer chest wall and the reduction in elastic recoil are factors in an age-related increase in FRC and RV.
REF: pg. 300
OBJ: 4
12. At approximately what age do respiratory muscles begin to weaken? a. 55 years b. 65 years c. 70 years d. 75 years ANS: A
At about 55 years of age, respiratory muscle strength begins to weaken. The diaphragm may be reduced to about 75% of the strength that is normal for healthy young adults. REF: pg. 301
OBJ: 4
13. Which of the following factors contribute to a reduction in an elderly person’s mucociliary
clearance? 1. Decrease in ciliary activity 2. Degenerative changes in the epithelial cells of the mucous membrane 3. Decrease in the phagocytic activity of macrophages in the mucous membrane 4. Decrease in the cough reflex a. 1, 2, 3 b. 2, 4 c. 1, 3 d. 1, 2, 3, 4 ANS: D
With increasing age, the epTitEhS elT iaB l cAeN llsKoSfEthLeLmEuRco.uCs O mMembrane that line the tracheobronchial tree show degenerative changes. Ciliary activity slows down. The phagocytic activity of macrophages in the mucous membrane is decreased. Combined, these changes lead to a reduction in mucociliary clearance. In addition, the loss of an effective cough reflex contributes to the increased susceptibility of older patients to lung infection. REF: pg. 301
OBJ: 4
14. What is the impact of aging on the body’s immune system? a. A higher white blood cell (WBC) count b. Higher fevers in response to bacterial infections c. Diminished response to vaccines d. Decreased occurrence of reactivation tuberculosis (TB) ANS: C
This age-related decline in cellular immunity correlates with an increased frequency of secondary (reactivation) tuberculosis in elderly patients. The acute antibody response to extrinsic antigens, such as pneumococcal and influenza vaccines, also is reduced in old age. Immunizations, although not as effective in older adults, still are strongly recommended. REF: pg. 301
OBJ: 5
15. What is the most surprising clinical finding in older patients with pneumonia that disguises its
presence? a. Lack of fever
b. Cough c. Diminished breath sounds. d. Purulent sputum production. ANS: A
In a patient with pneumonia, typical presentations include cough, fever, and purulent sputum production. These signs can be deceptively subtle in older adults, particularly the lack of an elevated temperature. With a lower base temperature and a reduced ability to mount a febrile response, older adults with pneumonia may be quite ill before the cause is detected. Some older patients with an infection simply may complain of a poor appetite, fatigue, lack of ability to perform daily activities, and generalized weakness. REF: pg. 301
OBJ: 5
16. Which of the following is the most common complaint from a patient suffering from an acute
myocardial infarction? a. Chest pain b. Throat pain c. Shoulder pain d. Abdominal pain ANS: A
Unfortunately, clinical presentation of heart attack may be altered by advanced age. The most common symptom is chest pain, but the pain may be referred. Complaints of shoulder, throat, and abdominal pain are common. REF: pg. 302
OBJ: 6
17. Why is asthma thought to be so underdiagnosed in the aged? a. They do not wheeze during an asthma attack. b. Their symptoms more closely match those of chronic obstructive pulmonary
disease (COPD). c. They do not wheeze when they have asthma. d. Historically, asthma has been considered a childhood disorder. ANS: D
Perhaps it is because historically, asthma has been considered a disease that is common to children and young adults. Other potential confounding factors may be associated with normal age-related pulmonary changes and blunted perception of symptoms. It is possible that when asthma and COPD occur simultaneously, only the COPD is diagnosed. REF: pg. 302
OBJ: 6
18. In healthy elderly men, what is the expected supine resting heart rate? a. 50 to 60 beats/min b. 60 to 100 beats/min c. 90 to 110 beats/min d. >110 beats/min ANS: B
In healthy men, the supine resting heart rate does not change with age.
REF: pg. 303
OBJ: 6
19. What would be a normal pulse pressure in the older adult? a. 20 b. 70 c. 110 d. 130 ANS: B
The pulse pressure (the difference between systolic and diastolic pressures) may widen. A normal pulse pressure is anywhere between 50 and 100 mm Hg in older adults. REF: pg. 303
OBJ: 6
20. What percentage of adults older than 65 years have elevated systolic or diastolic pressure? a. 20% b. 30% c. 40% d. 60% ANS: D
It is estimated that 60% of older adults have elevated systolic or diastolic blood pressure, or both. REF: pg. 303
OBJ: 6
21. The normal resting respiratory rate in elderly patients is in what range? a. 10 to 18 breaths/min b. 12 to 21 breaths/min c. 14 to 23 breaths/min d. 16 to 25 breaths/min ANS: D
The normal respiratory rate in elderly patients ranges from 16 to 25 breaths/min. REF: pg. 304
OBJ: 6
22. Which of the following disorders would be characterized by Cheyne-Stokes breathing? a. COPD b. Asthma c. CHF d. Raynaud phenomenon ANS: C
The more common causative factors of Cheyne-Stokes breathing include CHF, uremia, decreased blood flow to the respiratory depression, and brain damage. REF: pg. 304
OBJ: 6
23. Which of the following disorders would be characterized by intermittent cutaneous pallor or
cyanosis? a. COPD
b. Asthma c. Acute renal failure d. Raynaud phenomenon ANS: D
Raynaud phenomenon can present with intermittent cutaneous pallor or cyanosis. REF: pg. 305
OBJ: 6 | 7
24. Which clinical symptom that is indicative of dehydration in younger adults loses its
significance in the elderly because of diminished skin turgor? a. Tenting b. Pitting edema c. Cyanosis d. Clubbing ANS: A
Pinching the skin to check for tenting used to be a quick check for the presence of dehydration. REF: pg. 305
OBJ: 7
25. Which of the following would explain the presence of “clubbing” in the aged? a. COPD b. Simple aging process c. CHF d. Connective tissue disorders ANS: D
Abnormal enlargement of the distal phalanges, most easily noted in the fingers, is not age related. However, the conditions that lead to clubbing are chronic, and older adults have a higher incidence of chronic disease. COPD alone, even in the presence of hypoxemia, does not lead to clubbing. If older adult patients with COPD have clubbing, something other than obstructive lung disease is occurring, such as pulmonary manifestations of connective tissue disease. REF: pg. 305
OBJ: 6
26. An elderly patient who gains more than how many pound(s) in a week may be suffering from
CHF? a. 1 lb b. 3 lb c. 5 lb d. 7 lb ANS: C
A gain of more than 2 lb in a day or more than 5 lb in a week may indicate fluid retention and could signal CHF. REF: pg. 305
OBJ: 6
27. Why is pulmonary auscultation more difficult to assess in an elderly patient?
a. b. c. d.
Pulmonary abnormalities do not result in abnormal breath sounds in this age group. Coarse breath sounds are common even over normal lung tissue. Fine inspiratory crackles may indicate the presence of asthma. Patients often cannot sustain deep breathing of good quality.
ANS: D
Older adult patients may not be able to sustain deep breathing of good quality. Besides diminished breath sounds over healthy lung tissue, these patients otherwise will produce typical adventitious breath sounds in response to pathologic changes to the lung. REF: pg. 305
OBJ: 7
28. Which of the following pulmonary function measurements increases with age? a. Vital capacity b. Closing capacity c. Diffusing capacity d. Inspiratory capacity ANS: B
Closing capacity is simply closing volume added to residual volume. The closing capacity increases from about 30% of the total lung capacity at age 20 years to about 50% of the total lung capacity at age 70 years. REF: pg. 308
OBJ: 7
29. Which of the following PaO2 values would be normal for an 80-year-old individual who is
breathing room air? a. 60 mm Hg b. 65 mm Hg c. 50 mm Hg d. 85 mm Hg ANS: B
This will vary depending on the source, but according to Table 13-1, pg. 307, the expected level of partial pressure of oxygen in an 80-year-old would be 65 mm Hg. REF: pg. 307
OBJ: 7
30. Which of the following pulmonary function measurements decreases with age? a. Tidal volume b. Residual volume c. Total lung capacity d. Diffusing capacity ANS: D
After the age of 25 years, pulmonary function in adults starts to decline gradually. The total lung capacity and tidal volume remain relatively stable throughout the adult life span. The big change is seen in the residual volume, which almost doubles with advanced age. Diffusing capacity of the lung for carbon monoxide (DLCO) peaks in persons in their early 20s and then gradually decreases. REF: pg. 307
OBJ: 7
31. Which of the following laboratory values increases with age? a. Albumin b. Serum glucose c. Serum creatinine d. Creatinine clearance ANS: B
See Box 13-3, pg. 308. REF: pg. 308
OBJ: 7
32. Which of the following laboratory values decreases with age? a. Albumin b. Hemoglobin c. Serum glucose d. Serum cholesterol ANS: A
See Box 13-3, pg. 308. REF: pg. 308
OBJ: 7
Chapter 14: Respiratory Monitoring in Critical Care Test Bank MULTIPLE CHOICE 1. All the following activities are included in respiratory monitoring except: a. alarm setting. b. pulmonary consults. c. physical examinations. d. measurements and calculations. ANS: B
These goals are accomplished through the use of physical examination, monitoring equipment, measurements, calculations, and alarms. REF: pg. 315
OBJ: 1
2. It is important to monitor ventilatory parameters in addition to arterial blood gases because: a. it is easier and more cost-effective to monitor ventilatory parameters. b. changes in ventilatory parameters will occur before they are seen in arterial blood
gases. c. monitoring of ventilatory parameters does not require specialized equipment and
can be done more quickly. d. monitoring of ventilatory parameters can be done without a physician’s order,
whereas arterial blood gas monitoring requires a physician’s order. ANS: B
Changes in the patient’s metabolism, lung mechanics, ventilatory efficiency, and equipment function will occur before changes in blood gases are seen. REF: pg. 315
OBJ: 1
3. Ventilatory measurements routinely monitored at the bedside include all of the following
except: a. airway pressures. b. lung volumes and flows. c. fractional gas concentrations. d. oxygen consumption and carbon dioxide production. ANS: D
Ventilatory measurements that can be monitored at the bedside in the intensive care unit (ICU) routinely include lung volumes and flows, airway pressures, and fractional gas concentrations. REF: pg. 315
OBJ: 1
4. It is important to monitor lung volumes in ICU patients because changes in lung volumes
reflect all of the following changes except: a. changes in gas exchange in the lung. b. changes in the patient’s clinical status.
c. a response to therapy and any problems that may arise. d. they influence the selection of antibiotic therapy in the ICU. ANS: D
Lung volumes are important to the clinician for four reasons: (1) they affect gas exchange in the lung; (2) they reflect changes (improvement of deterioration) in the patient’s clinical status; (3) they indicate the response to therapy; and (4) they signal problems with patient/ventilator interface (i.e., circuitry, ventilator settings). REF: pg. 316
OBJ: 1
5. A nonintubated patient should be monitored for lung volumes in the presence of the following
clinical conditions except: a. deteriorating blood gases. b. receipt of noninvasive positive-pressure ventilation. c. respiratory rate less than 30 breaths/min. d. central nervous system depression. ANS: C
All the following represent conditions associated with the need for monitoring lung volumes in nonintubated patients: (1) preoperative evaluation (especially upper abdominal and thoracic surgery); (2) adult patients with respiratory rates greater than 30 breaths/min; (3) patients with neuromuscular disease; (4) patients with central nervous system (CNS) depression; (5) patients with deteriorating blood gases; and (6) patients receiving noninvasive positive-pressure ventilation. REF: pg. 316
OBJ: 1
6. Which of the following statements about the tidal volume is true? a. It usually is 10 to 15 mL/kg of ideal body weight. b. It usually is about 25% to 30% of total lung capacity. c. It is made up of two components: alveolar volume and dead space volume. d. It has an inversely proportional relationship with minute ventilation. ANS: C
VT has two components: alveolar volume (VA), or the portion of VT that effectively exchanges with alveolar-capillary blood, and dead space volume (VD), or the portion of VT that does not exchange with capillary blood. REF: pg. 316
OBJ: 1
7. In healthy, spontaneously breathing patients, an occasional increase in tidal volume to three or
four times the normal level, which normally occurs about six to ten times each hour, is the definition of a: a. sigh. b. cough. c. sneeze. d. forced vital capacity. ANS: A
In healthy, spontaneously breathing people, the VT occasionally increases to three or four times normal levels. These larger tidal breaths are known as sighs and normally occur about six to ten times each hour. REF: pg. 316
OBJ: 1
8. If intubated and mechanically ventilated patients are given shallow tidal volumes without
sighs, which of the following is most likely to occur? a. Coughing b. Atelectasis c. Respiratory arrest d. Increase in secretion production ANS: B
If shallow breathing without occasional sighing is maintained for prolonged periods, atelectasis and pneumonia may result, especially in patients who are breathing high oxygen concentrations or who are having compromised mucociliary clearance. REF: pg. 316
OBJ: 1
9. All of the following are likely to cause a decrease in a patient’s tidal volume except: a. pulmonary edema. b. metabolic acidosis. c. acute respiratory distress syndrome. d. the postoperative period after coronary artery bypass surgery. ANS: B
Conditions that may causeTthEeSVTTBtoAbNeKrS edEuL ceLdEinRc. luCdO eM pneumonia, atelectasis, the postoperative period following chest and abdominal surgery, chest trauma, acute exacerbation of chronic obstructive pulmonary disease (COPD), congestive heart failure (CHF), pulmonary edema, acute restrictive diseases such as acute respiratory distress syndrome (ARDS), neuromuscular diseases, and CNS depression (especially of the respiratory centers). REF: pg. 316
OBJ: 1
10. Which of the following is least likely to cause an increase in a patient’s tidal volume? a. Metabolic acidosis b. Sepsis c. Metabolic alkalosis d. Severe neurologic injury ANS: C
Larger-than-normal VT may be seen with metabolic acidosis, sepsis, or severe neurologic injury. REF: pg. 316
OBJ: 1
11. In mechanically ventilated patients, the addition of positive end-expiratory pressure (PEEP) to
normal tidal volumes is associated with the following effects except a(n): a. decrease in intrapulmonary shunting. b. increase in functional residual capacity. c. increase in partial pressure of arterial oxygen (PaO2).
d. increase in residual volume. ANS: D
When normal spontaneous VT is used during continuous mandatory ventilation (CMV) without positive end-expiratory pressure (PEEP), the following occur: a reduction in functional residual capacity (FRC), an increase in intrapulmonary shunt, and a fall in partial pressure of arterial oxygen (PaO2). REF: pg. 316
OBJ: 1d
12. Patients who are ventilated with excessively large tidal volumes are at risk for: a. air trapping. b. volutrauma. c. emphysema. d. hyperventilation. ANS: B
The use of high-VT ventilation may predispose patients to volutrauma, a lung injury that results from overdistention of terminal respiratory units. REF: pg. 317
OBJ: 1a
13. Which of the following conditions explains why lung damage persists after recovery from a
severe protracted episode of acute respiratory distress syndrome (ARDS)? a. Volutrauma b. Severity of the episode of ARDS c. Use of excessive amounts of PEEP during the episode d. Condition of the lungs T beEfS orT eB thAeNoK nsSetEoLf L dE iseRa. seCOM ANS: A
Volutrauma often develops in nondependent lung regions and is the main reason why lung damage persists after recovery from severe protracted ARDS. REF: pg. 317
OBJ: 1a
14. Volutrauma is most likely to develop in: a. the larger airways. b. dependent lung regions. c. nondependent lung regions. d. small airways. ANS: C
Volutrauma often develops in nondependent lung regions and is the main reason why lung damage persists after recovery from severe protracted ARDS. REF: pg. 317
OBJ: 1a
15. In a sedated, mechanically ventilated patient, inspiratory tidal volumes are consistently larger
than expiratory tidal volumes. If it is assumed that there is no leak in the circuit, which of the following provides the best explanation for this discrepancy? a. Ventilator malfunction b. The compressibility factor of the ventilator circuit
c. An increase in airway resistance that causes air trapping d. Different pressure profiles ANS: B
Differences often are caused by the compressible volume of the ventilator circuit or by environmental factors at different locations of the inspiratory and expiratory flow sensors (i.e., heated, humidified gases, differing flow profiles). REF: pg. 317
OBJ: 1 | 2
16. The “stacking” of breaths, which often is seen in mechanically ventilated patients with severe
airway obstruction, can be caused by: a. ventilator malfunction. b. a low-measured tidal volume. c. insufficient expiratory time. d. a respiratory rate that is too low. ANS: C
If not enough time is allowed for exhalation before the next breath is initiated by the ventilator, the subsequent VT will “stack” on top of the previous breath. REF: pg. 317
OBJ: 1
17. Which of the following ventilator changes will have to be made if the problem of “breath
stacking” is to be resolved? a. Increase in tidal volume b. Increase in respiratory rate c. Increase in inspiratory T flE ow STBANKSELLER.COM d. Increase in both respiratory rate and inspiratory flow ANS: C
Expiratory time is increased when the ventilator rate is reduced (if inspiratory time remains the same), thereby increasing the inspiratory flow rate or decreasing inspiratory time. REF: pg. 317
OBJ: 1
18. All of the following changes can decrease “breath stacking” in a mechanically ventilated
patient except: a. a decreased ventilator rate. b. an increased inspiratory flow. c. a decreased expiratory time. d. a decreased mechanical tidal volume. ANS: C
Expiratory time is increased by reducing the ventilator rate (if inspiratory time remains the same), thereby increasing the inspiratory flow rate or decreasing the inspiratory time. REF: pg. 317
OBJ: 1
19. An important advantage of monitoring mechanical tidal volumes proximally is that: a. it decreases circuit resistance and dead space. b. it decreases the respiratory work for the patient.
c. the measuring device is less susceptible to condensation. d. it eliminates the compressible volume factor of tubing circuits. ANS: D
Proximal volume monitoring eliminates the loss of compressible volume to the circuit and may reflect a more accurately delivered VT than does expiratory limb monitoring. REF: pg. 317
OBJ: 1a
20. Weaning failure during a spontaneous breathing trial may be predicted when the spontaneous
respiratory rate is greater than: a. 20 breaths/min. b. 25 breaths/min. c. 30 breaths/min. d. 35 breaths/min. ANS: D
The criteria used to define failure in this breathing trial include the following: a 20% increase or decrease in blood pressure or heart rate, SpO2 less than or equal to 85% to 90%, respiratory rate greater than 35 breaths/min, a change in mental status, accessory muscle use, and the onset of diaphoresis. REF: pg. 318, Box 14-1
OBJ: 1
21. In adult patients, spontaneous tidal volumes should be at least what value if weaning is to be
successful? a. 200 mL b. 300 mL c. 400 mL d. 500 mL ANS: B
VT also may decrease if the patient fatigues during the spontaneous breathing trial (SBT) and should be at least 300 mL or greater than 4 mL/kg. REF: pg. 318
OBJ: 1
22. The ratio of respiratory frequency to tidal volume that is used to predict the likelihood of
success in weaning the patient from mechanical ventilation is also known as: a. a spontaneous breathing test. b. the successful weaning test. c. the spontaneous ventilation index. d. the rapid-shallow breathing index. ANS: D
The rapid-shallow breathing index (RSBI) incorporates this spontaneous breath rate change and measures the ratio of respiratory frequency (f) to VT. RSBI = f (breaths/min)/VT (liters). If measured with a Wright spirometer and T-piece, RSBI values greater than 105 have been reported to be strong prognostic indicators of weaning failure. REF: pg. 318
OBJ: 1b
23. Which of the following values for the ratio of respiratory frequency to tidal volume would
predict that the patient will be successfully weaned off the ventilator? a. 50 b. 100 c. 150 d. 200 ANS: A
If measured using a Wright spirometer and T-piece, RSBI values greater than 105 have been reported to be strong prognostic indicators of weaning failure. REF: pg. 318
OBJ: 1b
24. Which of the following RSBI values would predict that the patient is least likely to be
successfully weaned from mechanical ventilation? a. 25 b. 50 c. 75 d. 100 ANS: D
If measured using a Wright spirometer and T-piece, RSBI values greater than 105 have been reported to be strong prognostic indicators of weaning failure. REF: pg. 318
OBJ: 1b
25. For weaning to be successful, the patient’s spontaneous minute ventilation needs to be less
than a. 10 b. 20 c. 30 d. 40
L/min.
ANS: A
If a E greater than 10 L/min is needed for a mechanically ventilated patient to maintain a normal PaCO2, weaning is not likely to be successful. REF: pg. 318
OBJ: 1
26. The highest incidence of postoperative morbidity is associated with which of the following
surgery sites? a. Transsternal b. Upper abdominal c. Lower abdominal d. Thoracoabdominal ANS: D
Although many factors can contribute to a reduction in vital capacity (VC) postoperatively, one of the most important is the incision site. Thoracic and abdominal surgery produce a significant fall in VC postoperatively, and this reduction may persist for a week or longer. REF: pg. 319
OBJ: 1 | 2
27. Which of the following values of VC is more consistent with impending respiratory failure? a. 10 mL/kg b. 30 mL/kg c. 50 mL/kg d. 70 mL/kg ANS: A
Values of less than 10 mL/kg usually are associated with impending respiratory failure. REF: pg. 319
OBJ: 1c
28. Which of the following ventilator changes is most likely to increase the functional reserve
capacity (FRC) and reduce the extent of acute lung injury? a. Increased FIO2 b. Increased flow c. Increased PEEP d. Decreased inspiratory time ANS: C
The application of PEEP prevents alveolar collapse and may reduce the extent of acute lung injury. REF: pg. 319
OBJ: 1d
29. The amount of force needed to overcome opposition to air flow in the lungs during
mechanical ventilation is known as: a. peak pressure. b. airway pressure. c. maximal airway resistance. d. positive end-expiratory pressure. ANS: A
Peak inspiratory pressure (PIP) is the maximum pressure attained during the inspiratory phase of mechanical ventilation (see Figure 14-1). It reflects the amount of force needed to overcome opposition to air flow into the lungs. REF: pg. 319
OBJ: 2
30. The amount of force needed to maintain a mechanical tidal volume breath in the patient’s
lungs is known as pressure. a. peak b. static c. positive end-expiratory d. continuous positive airway ANS: B
The plateau pressure (also referred to as static pressure) is the pressure required to maintain delivered VT in a patient’s lungs during a period of no gas flow. REF: pg. 320
OBJ: 2
31. All of the following are likely to increase the mean airway pressure (MAP) except an increase
in: a. b. c. d.
flow rate. PEEP levels. peak pressure. expiratory time.
ANS: D
Ventilator measurements that affect include CPAP and PEEP levels, inspiratory time (flow rate, flow patterns), peak pressure, and rate. A simple method of calculating is as follows: = [1/2 (PIP – PEEP) (Inspiratory time/Total cycle time)] + PEEP REF: pg. 320
OBJ: 2
32. If auto-PEEP is present, it is most likely to be detected if the expiratory limb of the patient
circuit is occluded at what point in the cycle? a. At the end of exhalation b. In the middle of inhalation c. In maximal inhalation d. At the middle of exhalation ANS: A
When the flow is stopped at end-exhalation, pressure equilibrates throughout the closed system and registers on the ventilator manometer. REF: pg. 322
OBJ:
T3ESTBANKSELLER.COM
33. Which of the following waveforms most accurately allows the clinician the recognition of
auto-PEEP? a. Pressure-volume b. Pressure-time c. Flow-time d. Flow-volume ANS: C
The flow-time waveform can be examined to determine most accurately whether expiratory flow returns to baseline before the subsequent inhalation. REF: pg. 322
OBJ: 3
34. Compliance is defined as: a. elasticity. b. pressure change per unit of volume. c. volume change per unit of pressure change. d. volume change per unit of flow. ANS: C
Compliance is defined as volume change per unit of pressure change, or the amount of lung volume achieved per unit of pressure.
REF: pg. 322
OBJ: 2
35. Acute respiratory distress syndrome (ARDS), pneumonia, and pulmonary edema are likely to
cause a decrease in lung compliance. This is evidenced in a mechanically ventilated patient by an increase in: a. static pressure. b. expiratory time. c. inspiratory time. d. dynamic pressure. ANS: A
Lung diseases such as pulmonary edema, pneumothorax, pneumonia, and ARDS increase lung recoil and observed static pressure. As a result, static compliance is reduced in these situations. REF: pg. 322
OBJ: 2
36. In the intensive care unit (ICU), the airway resistance (Raw) of a mechanically ventilated
patient can be estimated easily by using which of the following formulas? a. (Flow – Inspiratory time)/60 b. (Peak pressure – Static pressure)/2 c. (Peak pressure – Static pressure)/Flow d. (ET tube diameter – Static pressure)/Peak airway pressure ANS: C
REF: pg. 323
OBJ: 2
37. When a patient’s mechanical ventilator has a graphic display screen, which of the following
waveforms could be used to determine whether there is any leak in the system and the amount of the leak? a. Flow-time b. Volume-time c. Pressure-time d. Pressure-flow ANS: B
The volume-time waveform is used most often to compare inspiratory and expiratory delivered volumes. This can be particularly useful in checking for leaks within the ventilator circuit system, or in determining the amount of leak around the endotracheal tube or through a chest tube. REF: pg. 326
OBJ: 4
38. Which of the following is normally used when PEEP levels are titrated to determine optimal
PEEP? a. Flow-time curve b. Volume-time curve c. Pressure-time curve
d. Pressure-volume curve ANS: D
Some investigators have advocated using a static pressure-volume curve to titrate PEEP and tidal volume. REF: pg. 328
OBJ: 4
39. When carbon dioxide elimination is monitored, the highest levels are obtained at the: a. end of inhalation. b. end of exhalation. c. middle of inhalation. d. middle of exhalation. ANS: B
Exhaled alveolar carbon dioxide concentrations often are at their highest at the end of exhalation, so the term end-tidal PCO2 (PETCO2) may be used to indicate the highest exhaled carbon dioxide concentrations attained. REF: pg. 331
OBJ: 5
40. Carbon dioxide production is increased by approximately what percentage per 1 C of body
temperature in patients with fever? a. 1% b. 10% c. 25% d. 50% ANS: B
CO2 often is increased in fever (10% increase per 1 C increase), trauma, peritonitis (25% to 50% increase), head trauma, rewarming after hypothermia, and high carbohydrate loading with total parenteral nutrition. REF: pg. 332
OBJ: 5
41. When the dead space–to–tidal volume ratio (VD/VT) is calculated, VD refers to the
dead
space. a. alveolar b. anatomic c. ventilatory d. physiologic ANS: D
VD has two components: anatomic dead space and alveolar dead space. Anatomic dead space is made up of the conducting airways and normally measures about 1 mL/kg of ideal body weight. Alveolar dead space is classically defined as alveoli that are ventilated but are not perfused. The combination of anatomic and alveolar dead space is called physiologic dead space. REF: pg. 333
OBJ: 5 | 6
42. Under normal conditions, the hemoglobin is responsible for carrying approximately what
percentage of the oxygen carried in the blood? a. 66% b. 75% c. 88% d. 99% ANS: D
Under normal conditions, the hemoglobin is responsible for carrying 99% or more of the oxygen in the blood. REF: pg. 333
OBJ: 6
43. Which of the following conditions is the most common cause of inadequate oxygenation of
pulmonary capillary blood? a. Hypovolemia b. mismatch c. Hypoventilation d. Diffusion defect ANS: B
Traditional respiratory care has focused on the physiologic mechanisms that result in inadequate oxygenation of pulmonary capillary blood. These mechanisms include the following: mismatch (most common cause), diffusion block (rare cause), hypoventilation, and shunt (extreme mismatch). REF: pg. 333
OBJ: T6E| 7STBANKSELLER.COM
44. The cardiac output will increase to compensate for a decrease in oxygen tension when the
PaO2 falls to below a. 50 b. 60 c. 70 d. 80
mm Hg.
ANS: A
Cardiac output is not sensitive to moderate changes in oxygen tension (PO2) and usually does not increase until the PaO2 drops to below 50 mm Hg. REF: pg. 334
OBJ: 6 | 7
45. On the oxygen dissociation curve (ODC), the P-50 refers to the: a. partial pressure of oxygen when the hemoglobin is 50% saturated. b. hemoglobin saturation when the fraction of inspired oxygen is 50%. c. partial pressure of oxygen when the fraction of inspired oxygen is 50%. d. hemoglobin saturation when the partial pressure of oxygen is 50 mm Hg. ANS: A
Clinically, the position of the curve is measured by tonometry or is calculated at a point where the oxygen’s partial pressure has saturated 50% of the hemoglobin (P-50).
REF: pgs. 334-335
OBJ: 7
46. The normal compensatory mechanism for a left shift in the oxygen dissociation curve is a(n): a. increase in tidal volume. b. increase in cardiac output. c. increase in respiratory rate. d. decrease in body temperature. ANS: B
The normal compensatory mechanism for a left-shifted curve is an increase in cardiac output. REF: pg. 335
OBJ: 7
47. Under most clinical conditions, the PaO2 should be kept within the range of a. 60 to 80 b. 80 to 100 c. 100 to 120 d. 120 to 150
mm Hg.
ANS: A
Under most clinical conditions, the PaO2 should be kept within a range of 60 to 80 mm Hg. REF: pg. 335
OBJ: 7 | 8
48. Which level of a PaO2/FIO2 ratio is consistent with a definition of acute lung injury (ALI)? a. 50 b. 100 to 200 c. 150 d. 200 to 300 ANS: D
A PaO2/FIO 2 ratio of between 200 and 300 mm Hg is associated with acute lung injury (ALI) and a PaO2/FIO 2 ratio of less than 200 is associated with ARDS. REF: pg. 336
OBJ: 8
49. All of the following will cause an increase in intrapulmonary shunting except: a. ARDS. b. atelectasis. c. pneumonia. d. cystic fibrosis. ANS: D
Clinical states that often produce an increase in include atelectasis, pneumonia, ARDS, pulmonary edema, and, rarely, congenital heart anomalies or arteriovenous anastomosis. REF: pg. 336
OBJ: 8
50. The parameter that indicates oxygen usage throughout the whole body is: a. CaO2. b. .
c. PaCO2. d. P(A – a)O2. ANS: B
is a measure of the partial pressure of oxygen in mixed venous blood and is an indication of oxygen usage by the entire body. REF: pg. 338
OBJ: 8c
51. The portion of delivered oxygen actually consumed and an index of the efficiency of
circulation is the definition for the: a. oxygenation index (OI). b. oxygen extraction ratio (C[a – v]O2/CaO2). c. arterial-mixed venous oxygen content difference (C[a – v]O2). d. arterial-mixed venous oxygen saturation difference (S[a – v]O2). ANS: B
The oxygen extraction ratio identifies the portion of delivered oxygen that actually is consumed and is therefore an index of the efficiency of circulation. REF: pg. 340
OBJ: 10
Chapter 15: Vascular Pressure Monitoring Test Bank MULTIPLE CHOICE 1. Arterial cannulation is indicated for all of the following except: a. determining cardiac output. b. severe hypotension. c. unstable respiratory failure. d. avoiding arterial injury from multiple arterial punctures. ANS: A
Arterial cannulation is not needed to assess cardiac output. REF: pg. 349
OBJ: 1a
2. Which of the following techniques is used most commonly to insert an arterial pressure
monitoring catheter? a. Needle-through-needle technique b. Judkins technique c. Seldinger technique d. Sones technique ANS: C
The Seldinger technique is most popular for placement of an arterial line. REF: pg. 351
OBJ: T1E d STBANKSELLER.COM
3. The most common site for arterial catheterization is the a. radial b. axillary c. femoral d. brachial
artery.
ANS: A
The radial artery is the most popular site for placement of an arterial line because it can be stabilized easily and collateral circulation is available should a clot develop. REF: pg. 349
OBJ: 1b
4. What does the dicrotic notch represent on an arterial pressure waveform? a. Aortic valve closure b. Mean arterial pressure c. A blood pressure increase of 50 to 60 mm Hg d. An inaccurate pressure reading lower than the actual pressure ANS: A
The dicrotic notch represents aortic valve closure, which occurs as the ventricles relax. REF: pg. 351
OBJ: 1e
5. When vasodilators such as sodium nitroprusside are administered, it is important to monitor
the fall in blood pressure because low blood pressure can: a. cause arterial spasm. b. cause a stroke in obese adult patients. c. decrease blood flow to the coronary arteries. d. increase the likelihood that pulmonary edema may develop. ANS: C
Hypotension can lead to poor coronary artery blood flow and cardiac ischemia. REF: pg. 352
OBJ: 1f
6. The pulse pressure is important hemodynamically because it is an indication of: a. central venous pressure (CVP). b. mean arterial pressure (MAP). c. right ventricular stroke volume. d. left ventricular stroke volume. ANS: D
The pulse pressure is determined primarily by the stroke volume of the left ventricle. REF: pg. 354
OBJ: 1f
7. Circulation to the vital organs (kidneys, coronary arteries) may be compromised if the MAP
falls to below a. 60 b. 80 c. 100 d. 120
mm Hg.
ANS: A
The MAP must remain above 60 mm Hg if perfusion to all vital organs is to be maintained. REF: pg. 354
OBJ: 1f
8. What will the MAP be if the systolic blood pressure is 140 mm Hg and the diastolic blood
pressure is 80 mm Hg? a. 60 mm Hg b. 100 mm Hg c. 110 mm Hg d. 120 mm Hg ANS: B
The equation is:
Pulse pressure + Diastolic pressure = MAP
REF: pg. 354
OBJ: 1f
9. Complications of direct arterial monitoring include all the following except: a. infection. b. ischemia. c. hemorrhage. d. hypotension.
ANS: D
Direct arterial cannulation is not likely to cause hypotension. REF: pg. 354
OBJ: 1g
10. The CVP represents: a. the right ventricular preload. b. the pressure of blood in the right atrium. c. the right ventricular end-diastolic pressure. d. all of the above. ANS: D
The CVP represents the right ventricular preload, pressure in the right atrium, and right ventricular end-diastolic pressure. REF: pg. 354
OBJ: 2d
11. Monitoring of the CVP is indicated when there is a need to assess: a. the circulating blood volume. b. the degree of venous return. c. the right ventricular function. d. all of the above. ANS: D
CVP monitoring can provide important information on the amount of circulating blood volume, degree of venous return, and right ventricular function. REF: pg. 354
a STBANKSELLER.COM OBJ: T2E
12. The CVP catheter insertion site that provides both stability after placement and accurate
hemodynamic information is the a. external jugular b. internal jugular c. subclavian d. femoral
vein.
ANS: C
The subclavian vein is the most popular site for placement of a CVP line because of its stability. REF: pg. 355
OBJ: 2b
13. Which of the following is not an advantage of using the internal jugular vein as the insertion
site for the central venous catheter? a. The risk of pneumothorax from pleural puncture is low. b. Visibility is good if hematomas form around the neck. c. Tubing is more stable and is less likely to kink in a patient who is awake. d. It is nearly a straight line from the right internal jugular vein to the right atrium. ANS: C
The internal jugular vein is not a popular site for placement of a CVP line, because the tubing is not stable at this site.
REF: pg. 355
OBJ: 2b
14. Both the CVP and the pulmonary artery wedge pressure waveforms have three waves for each
cycle, namely: a. waves a, c, and v. b. waves q, r, and s. c. waves p, t, and u. d. none of the above. ANS: A
The CVP pressure tracing normally has three waves: a, c, and v. REF: pg. 356
OBJ: 2d
15. The a wave would be absent in which of the following dysrhythmias? a. Sinus bradycardia b. First-degree heart block c. Third-degree heart block d. Atrial fibrillation ANS: D
The a wave represents atrial contraction. In atrial fibrillation, there is no atrial contraction. REF: pg. 356
OBJ: 2d
16. An exaggerated v wave would be found in which of the following conditions? a. Atrial fibrillation b. Tricuspid regurgitation c. Ventricular diastole d. All of the above ANS: B
Tricuspid regurgitation causes blood to leak backward into the right atria, making the v wave more pronounced. REF: pg. 356
OBJ: 2d
17. An increase in CVP during inspiration most likely is caused by: a. a mechanical breath. b. a spontaneous breath. c. the instrument not being zero-balanced correctly. d. the catheter being inserted into the right ventricle. ANS: A
A positive-pressure breath will cause the CVP to increase during inspiration. REF: pg. 356
OBJ: 2e
18. During systole, the CVP v waveform is exaggerated and the CVP increases. This most likely
is due to: a. a kink in the tubing.
b. air bubbles in the catheter. c. an incompetent tricuspid valve. d. incorrect positioning of the measuring instrument with respect to the right atrium. ANS: C
Tricuspid regurgitation causes blood to leak backward into the right atria, making the v wave more pronounced. REF: pg. 356
OBJ: 2d
19. What change is seen on the CVP waveform when the patient takes a spontaneous deep breath? a. The waveform falls to baseline. b. The waveform falls to below baseline. c. The waveform increases to above baseline. d. There is no change in the waveform. ANS: B
A deep spontaneous breath causes intrathoracic pressure to drop significantly. REF: pg. 356
OBJ: 2d | 2e
20. When the water manometer is used to measure the CVP, the reading usually is taken at: a. peak inspiration with the patient in the semi-Fowler position. b. the end of expiration with the patient in the supine position. c. the end of inspiration with the head of the bed elevated 45 degrees. d. any time during inspiration or expiration with the head of the bed elevated 45
degrees. ANS: B
The influence of pleural pressure changes on the CVP is least at end-exhalation. REF: pgs. 356-357
OBJ: 2e
21. All of the following cause an increase in the CVP except: a. volume overload. b. tension pneumothorax. c. spontaneous inspiration. d. increased intrathoracic pressure. ANS: C
Spontaneous inspiration causes the intrathoracic pressure to drop and the CVP reading to decrease. REF: pg. 356
OBJ: 2e
22. All of the following cause an increase in the CVP except: a. peripheral vasodilation. b. right ventricular failure. c. pulmonary hypertension. d. positive-pressure mechanical ventilation. ANS: A
Peripheral vasodilation causes venous return to decrease and CVP to decrease.
REF: pg. 358
OBJ: 2e
23. The CVP can be used to estimate left ventricular filling pressures if there is no left ventricular
dysrhythmia and the ejection fraction is greater than: a. 0.20. b. 0.30. c. 0.40. d. 0.50. ANS: D
A low ejection fraction causes blood from the left side of the heart to back up into the right side of the heart and elevates the CVP. REF: pg. 358
OBJ: 2f
24. The CVP is most likely to correlate well with left ventricular filling pressure in which of the
following patients? a. Male, age 20 years, scheduled for surgery for mitral valve stenosis repair b. Female, age 52 years, scheduled for coronary artery bypass grafting in four arteries c. Male, age 37 years, receiving care in the intensive care unit (ICU) for acute respiratory distress syndrome (ARDS) d. Female, age 25 years, recovering well after abdominal surgery ANS: D
A young person who is recovering from abdominal surgery is not likely to have an elevated pulmonary vascular resistance. REF: pg. 357
OBJ: 2f
25. Complications involving the use of CVP catheters include all of the following except: a. Bleeding. b. Pneumothorax. c. Infection. d. Atrial septal puncture. ANS: D
The CVP catheter does not reach into the atria. REF: pg. 358
OBJ: 2g
26. The pulmonary artery catheter allows assessment of: a. the left ventricular preload. b. the mixed venous oxygen saturation (SvO2). c. the cardiac output. d. all of the above. ANS: D
Placement of a pulmonary artery catheter allows assessment of left ventricular preload, SvO2, and cardiac output (CO). REF: pg. 359
OBJ: 3a
27. Pulmonary artery catheter monitoring is likely to be considered for patients with any of the
following conditions except: a. ARDS. b. pulmonary embolus. c. cardiogenic pulmonary edema. d. septic shock. ANS: B
The patient with a pulmonary embolus does not need a pulmonary artery catheter. A CVP catheter is adequate. REF: pg. 359
OBJ: 3a
28. The purpose of the balloon at the tip of the pulmonary artery catheter is to: a. obtain wedge pressure measurements. b. sense temperature changes in the blood. c. make the catheter more visible on a chest x-ray. d. provide a place for slow-release medication to be stored before it is released. ANS: A
The balloon allows the tip of the catheter to be influenced only by left-sided pressures. REF: pg. 360
OBJ: 3b
29. When the pulmonary artery catheter is inserted, the balloon is inflated in the right atrium
before it is inserted further. This is done to: a. allow the catheter to floTaE t.STBANKSELLER.COM b. decrease the risk of premature ventricular contractions. c. allow the catheter to enter the right ventricle more easily. d. all of the above. ANS: A
The inflated balloon allows the catheter to float through the right side of the heart and into the pulmonary artery. REF: pg. 360
OBJ: 3b | 3c
30. If, when the pulmonary artery catheter is inserted, there is a rapid increase in the height of the
pressure waveforms with the downstroke dropping near zero, the respiratory therapist should: a. continue the insertion because the catheter now has entered the right ventricle. b. deflate the balloon and withdraw the catheter because it is curling within the atrium. c. withdraw the catheter because it has been jammed into the heart wall. d. inflate the balloon to prevent further damage because the ventricular septum has ruptured. ANS: A
Pressure in the right ventricle normally is near zero during diastole. REF: pg. 366
OBJ: 3d
31. The normal range for pulmonary artery systolic pressure is a. 0 to 10 b. 10 to 20 c. 20 to 30 d. 30 to 40
mm Hg.
ANS: C
The right ventricle normally has a small muscle mass and can generate pressures of about 20 to 30 mm Hg. REF: pg. 362
OBJ: 3e
32. With a properly inserted and positioned pulmonary artery catheter, a systolic pressure reading
in the pulmonary artery of 50 mm Hg could be due to: a. pulmonary vasodilation. b. tricuspid valve stenosis. c. pulmonic valve stenosis. d. chronic obstructive pulmonary disease (COPD). ANS: D
COPD causes elevation of the pulmonary vascular resistance and elevation of the pulmonary artery systolic pressure. REF: pg. 362
OBJ: 3e
33. An increase in pulmonary artery systolic pressure is seen in patients with all of the following
conditions except: a. pulmonary embolus. b. mitral valve stenosis. c. right ventricular failure. d. hypoxia and hypoxemia. ANS: C
Right ventricular failure does not increase pulmonary artery pressure. REF: pg. 362
OBJ: 3e
34. The normal range for pulmonary artery diastolic pressure is a. 0 to 8 b. 8 to 15 c. 15 to 28 d. 28 to 35
mm Hg.
ANS: B
Normal pulmonary artery diastolic pressure is about 8 to 15 mm Hg. REF: pg. 363
OBJ: 3e
35. A difference between the pulmonary artery diastolic pressure and the pulmonary capillary
wedge pressure greater than 5 mm Hg is seen in which of the following conditions? a. ARDS b. Sepsis
c. Excessive positive end-expiratory pressure (PEEP) d. All of the above ANS: D
Any condition that causes the pulmonary vascular resistance (PVR) to become elevated causes the pulmonary artery diastolic pressure to be more than 5 mm Hg above wedge pressure. REF: pg. 363
OBJ: 3f
36. The normal pulmonary capillary wedge pressure is a. 0 to 6 b. 4 to 12 c. 10 to 15 d. 12 to 18
mm Hg.
ANS: B
The normal wedge pressure is 4 to 12 mm Hg. REF: pg. 363
OBJ: 3e
37. In left ventricular failure, the pulmonary capillary wedge pressure is expected to: a. fall to zero. b. remain in the normal range. c. increase to above the normal range. d. decrease to below the normal range. ANS: C
Left ventricular failure causes a backup of blood into the pulmonary circulation and a significant elevation of wedge pressure. REF: pg. 364
OBJ: 3e
38. For the pulmonary capillary wedge pressure to reflect pulmonary venous and left atrial
pressures, blood flow must be uninterrupted between the catheter tip and the left heart. This condition exists only in West zone: a. I. b. II. c. III. d. IV. ANS: C
In zone III, pulmonary capillary blood flow is maintained because of the influence of gravity on blood flow. REF: pg. 364
OBJ: 3g
39. According to West, which zone is dominant in the supine position? a. I b. II c. III d. IV
ANS: B
Patients lying in the supine position have zone II most available for positioning of the pulmonary artery catheter. REF: pg. 364
OBJ: 3g
40. A shift to zone I or II may occur if which of the following conditions is present? a. Hypovolemia b. Increased PEEP c. Hemorrhage d. All of the above ANS: D
As blood volume decreases as PEEP is applied, blood flow to dependent regions of the lung may be diminished and zone III may be converted to zone II. REF: pg. 364
OBJ: 3g
41. Large fluctuations in the net distending pressure within the left ventricle, also known as the
transmural pressure, are seen in which of the following situations? a. Coughing b. Valsalva maneuver c. Positive-pressure ventilation d. All of the above ANS: D
Any activity that increases intrathoracic pressure can influence the distending pressure within the left ventricle. REF: pg. 366
OBJ: 3h
42. PEEP levels lower than what value will have a limited effect on intrapleural pressure? a. 5 cm H2O b. 10 cm H2O c. 15 cm H2O d. 20 cm H2O ANS: B
PEEP levels lower than 10 have a limited effect on intrapleural pressure. REF: pg. 366
OBJ: 3i
43. All of the following are potential hazards of pulmonary artery catheter migration except: a. air embolus. b. dysrhythmias. c. pulmonary artery rupture. d. pulmonary infarction. ANS: A
Air embolus is not likely with pulmonary artery catheterization. REF: pg. 359
OBJ: 3j
44. A patient with a newly inserted pulmonary catheter begins to cough up blood-tinged sputum.
The most likely cause for this is: a. pneumothorax. b. pulmonary embolus. c. pulmonary infarction. d. perforation of the heart muscle. ANS: C
Pulmonary infarction can cause hemoptysis. This problem occurs when the balloon is left inflated too long. REF: pg. 359
OBJ: 3j
Chapter 16: Cardiac Output Measurement Test Bank MULTIPLE CHOICE 1. What is the most important factor in assessment of the ability of the cardiovascular system to
meet the body’s metabolic demands? a. Respiratory quotient (RQ) b. Cardiac output c. Oxygen uptake d. Adequacy of perfusion ANS: D
The ability of the blood flow to perfuse all vascular beds is important if the metabolic demands of the body are to be met. REF: pg. 374
OBJ: 1
2. The amount of blood pumped out of the left ventricle in 1 minute is the definition of: a. afterload. b. cardiac output. c. stroke volume. d. ejection fraction. ANS: B
Cardiac output refers to the blood flow in liters per minute pumped by the left ventricle. REF: pg. 374
OBJ: 1
3. The average cardiac output for adult men and women at rest is a. 0.5 b. 5 c. 15 d. 25
L/min.
ANS: B
Normal cardiac output varies with the size of the patient, but the average is 5.0 L/min. REF: pg. 374
OBJ: 1
4. All of the following can affect cardiac output except: a. race. b. body size. c. hematocrit. d. tissue demand for oxygen. ANS: A
Race does not influence cardiac output. The larger the patient’s body mass index (BMI), the higher is the predicted normal cardiac output. A low hematocrit makes the blood viscosity low and increases the cardiac output. The cardiac output increases with increased tissue metabolism.
REF: pg. 374
OBJ: 1
5. To enable the heart of a well-trained athlete to produce a cardiac output of up to 35 L/min,
stimulation of the heart muscle is needed through the: a. vagus nerve. b. phrenic nerve. c. autonomic nervous system. d. sympathetic nervous system. ANS: D
The sympathetic nervous system can increase the rate and contractility of the heart when stimulated. REF: pgs. 374-375
OBJ: 2
6. Blood flow is directed to organs that have the highest metabolic needs when at rest. Which of
the following organs would have the lowest metabolic needs when at rest? a. Heart b. Liver c. Muscles d. Kidneys ANS: A
The heart is very efficient in its use of oxygen. REF: pg. 375
OBJ: 3
7. At rest, approximately what percentage of total blood volume is stored in the venous system? a. 16% b. 32% c. 48% d. 64% ANS: D
The venous system is very large and holds almost two-thirds of the circulating blood volume at rest. REF: pg. 375
OBJ: 4
8. At rest, approximately 64% of the total blood volume is stored in the: a. liver. b. lungs. c. extremities. d. venous system. ANS: D
The venous system is very large and holds almost two-thirds of the circulating blood volume at rest. REF: pg. 375
OBJ: 4
9. Approximately how much blood can be lost before circulatory function and pressures are
affected? a. 5% b. 15% c. 25% d. 35% ANS: C
The circulatory system can tolerate as much as a 25% loss in blood volume before the blood pressure begins to fall. REF: pg. 375
OBJ: 4
10. Cardiac output is a product of which two variables? a. Heart rate and stroke volume b. Heart rate and body surface area c. Cardiac index and blood pressure d. Stroke volume and ejection fraction ANS: A
The rate of ventricular contraction multiplied by the stroke volume (the volume of blood ejected by the left ventricle during each heartbeat) will provide the cardiac output. REF: pg. 375
OBJ: 2
11. To standardize the measurement of cardiac output in individuals of different sizes, the value
for cardiac output is divided by the person’s body surface area. The result is known as: a. cardiac work. b. the cardiac index. c. the standardized cardiac output. d. the cardiac end-systolic volume. ANS: B
The cardiac index takes into account the size of the patient and allows clinicians to identify whether the cardiac output is appropriate for the size of the patient. REF: pg. 375
OBJ: 2
12. All the following are true about the ejection fraction except: a. the normal value is about 65% to 70%. b. it can be calculated from a nomogram. c. the percentage is the end-diastolic volume ejected with each beat. d. it represents the difference between end-diastolic and end-systolic volumes. ANS: B
The ejection fraction cannot be calculated from a nomogram. REF: pgs. 375-377
OBJ: 6b
13. The normal ejection fraction is approximately: a. less than 50%. b. 55% to 60%.
c. 65% to 70%. d. 75% to 80%. ANS: C
Normally, about 65% to 70% of the end-diastolic volume is ejected during systole. REF: pg. 377
OBJ: 6b
14. The cardiac output of the right ventricle is the same as the cardiac output of the left ventricle.
Compared with the left ventricle, the cardiac work index of the right ventricle is: a. less. b. the same. c. somewhat greater. d. much greater. ANS: A
Pulmonary artery pressures normally are much lower than pressures in the arterial system. Therefore, the right ventricular work index is normally much lower than that of the left ventricle. REF: pg. 376
OBJ: 6f
15. In patients with pulmonary hypertension, the right ventricular stroke work index will be: a. less than normal. b. the same as normal. c. greater than normal. d. equal to the left ventricular stroke work index. ANS: C
Pulmonary hypertension makes it more difficult for the right ventricle to eject blood into the pulmonary artery and increases the amount of work needed to maintain appropriate blood flow. REF: pg. 381
OBJ: 6f
16. Which of the following does not have a direct effect on stroke volume? a. Preload b. Heart rate c. Afterload d. Contractility ANS: B
Heart rate has an indirect effect on stroke volume. REF: pg. 377
OBJ: 6c
17. Bradycardia in an adult is defined as a heart rate less than a. 50 b. 60 c. 100 d. 120 ANS: B
beats/min.
Bradycardia is a heart rate less than 60 beats/min. REF: pg. 377
OBJ: 2
18. Tachycardia in an adult is defined as a heart rate greater than a. 50 b. 60 c. 100 d. 120
beats/min.
ANS: C
Tachycardia is a heart rate above 100 beats/min. REF: pg. 377
OBJ: 2
19. Which of the following best describes preload? a. The filling volume of the left ventricle. b. The stretch on the ventricular muscle fibers before contraction. c. The difference between the end-systolic and end-diastolic volumes. d. The volume of blood deposited from the right atrium into the right ventricle. ANS: B
The term preload refers to the degree of stretch on the ventricular muscles before systole. REF: pg. 377
OBJ: 7
20. The strength of contraction of the myocardial muscle fibers is related directly to the amount of
stretch on the fiber before T coEnS traTcB tiA onNiK sS aE deLfiLnE itiR on.C ofO :M a. preload. b. Starling law. c. cardiac contractility. d. ventricular compliance. ANS: B
The Starling law states that the degree of contraction is related directly to the amount of stretch in the left ventricle before systole. REF: pg. 377
OBJ: 7
21. When ventricular function curves are performed on a patient with a suspected failing left
ventricle, how does cardiac output typically change as the pulmonary capillary wedge pressure increases? a. Cardiac output increases at first and then stays flat. b. Cardiac output increases at first and then decreases. c. Cardiac output decreases at first and then stays flat. d. Cardiac output decreases at first and then increases. ANS: B
Initial stretch on the ventricle increases contractility, but too much stretch then causes the stroke volume to decrease. REF: pgs. 377-378
OBJ: 7
22. When ventricular function curves are performed on a patient with a suspected failing right
ventricle, how does cardiac output typically change as central venous pressure increases? a. Cardiac output increases at first and then stays flat. b. Cardiac output increases at first and then decreases. c. Cardiac output decreases at first and then stays flat. d. Cardiac output decreases at first and then increases. ANS: B
Initial stretch on the ventricle increases contractility, but too much stretch then causes stroke volume to decrease. REF: pg. 378
OBJ: 7
23. The filling pressure of the right heart also is known as the a. Starling b. central venous c. ventricular filling d. pulmonary capillary wedge
pressure.
ANS: B
Central venous pressure provides the filling pressure for the right ventricle. REF: pg. 378
OBJ: 7
24. The filling pressure of the left heart also is known as the a. Starling b. central venous c. ventricular filling d. pulmonary capillary wedge
pressure.
ANS: D
The pulmonary capillary wedge pressure is used to measure the left ventricular filling pressure. REF: pg. 378
OBJ: 7
25. All of the following may cause a decrease in ventricular compliance except: a. inotropic drugs. b. vasodilator drugs. c. myocardial infarction. d. positive end-expiratory pressure. ANS: B
Vasodilators do not affect ventricular compliance. REF: pg. 378
OBJ: 8
26. What effect does an increase in intrathoracic pressure have on venous return? a. No effect b. A dampening effect c. An enhancing effect
d. An unknown effect ANS: B
An increase in intrathoracic pressure reduces venous return. REF: pg. 379
OBJ: 7
27. What effect does spontaneous deep inspiration have on venous return? a. No effect b. A dampening effect c. An enhancing effect d. An unknown effect ANS: C
The negative intrathoracic pressure that occurs with deep inspiration increases venous return. REF: pg. 379
OBJ: 7
28. The resistance to ventricular ejection also is known as: a. afterload. b. Forrester’s hemodynamic subset I. c. Forrester’s hemodynamic subset II. d. Forrester’s hemodynamic subset III. ANS: A
Resistance to ventricular ejection during systole is known as afterload. REF: pg. 380
OBJ: 7
29. Intermittent positive-pressure breathing (IPPB) with positive end-expiratory pressure (PEEP)
has what effect on afterload? a. No effect b. A dampening effect c. An enhancing effect d. An unknown effect ANS: C
Increasing intrathoracic pressure with IPPB and PEEP enhances ventricular emptying. REF: pg. 381
OBJ: 5 | 7
30. IPPB with PEEP has what effect on preload? a. No effect b. A dampening effect c. An enhancing effect d. An unknown effect ANS: B
Positive pressure in the thorax reduces venous return and preload. REF: pg. 381
OBJ: 5 | 7
31. All the following make up the peripheral component of afterload except:
a. b. c. d.
the radius of the vessels. the length of the vessels. the viscosity of the blood. the elasticity or compliance of the vessels.
ANS: B
The length of the vessels has little effect on afterload. REF: pg. 381
OBJ: 8
32. The greatest determinant of peripheral resistance to blood flow is: a. the radius of the vessels. b. the viscosity of the blood. c. the compliance of the vessels. d. the distance of the vessels from the heart. ANS: A
The radius of the vessels has the greatest effect on vascular resistance. REF: pg. 381
OBJ: 8
33. Which of the following is used to determine the afterload of the right ventricle? a. Cardiac output b. Pulmonary artery pressure c. Pulmonary vascular resistance d. Pulmonary capillary wedge pressure ANS: C
The pulmonary vascular resistance is used to assess right ventricular afterload. REF: pg. 381
OBJ: 8
34. All of the following can cause an increase in pulmonary vascular resistance except: a. exercise. b. acidosis. c. hypoxemia. d. release of histamine because of allergic response. ANS: A
Exercise has little effect on the pulmonary vascular resistance. REF: pg. 381
OBJ: 8
35. A drug that increases the contractility of the heart is known as a positive: a. inotrope. b. dromotrope. c. chromotrope. d. vasoconstrictor. ANS: A
Positive inotropic drugs increase cardiac contractility. REF: pg. 379
OBJ: 7 | 8
36. To obtain the coronary perfusion pressure, what two values are needed? a. Afterload and preload b. Ejection fraction and heart rate c. Mean arterial pressure and afterload d. Diastolic blood pressure and pulmonary capillary wedge pressure ANS: D
Subtracting the pulmonary capillary wedge pressure from the diastolic blood pressure yields the coronary perfusion pressure. REF: pg. 383
OBJ: 9
37. What method of measuring cardiac output is most popular in intensive care units (ICUs)
around the country? a. Thermodilution b. Fick method c. Pulse contour method d. Intravascular ultrasound ANS: A
The thermodilution technique is most popular because it is accurate and is fairly easy to perform. REF: pg. 385
OBJ: 9b
38. Why has use of the pulmonary artery catheter diminished in many ICUs? a. It is expensive. b. It is difficult to place. c. Patients complain about the discomfort. d. Some experts believe it increases death rates. ANS: D
Although little evidence supports the notion that placement of a pulmonary artery catheter increases death rates, some experts believe that it does. REF: pg. 386
OBJ: 9
39. When the Fick method is used to calculate cardiac output, if C(a – v)O2 is greater than
expected and VO2 is normal, cardiac output probably will be: a. normal. b. less than expected. c. greater than expected. d. unknown—insufficient information is available to answer the question. ANS: B
C(a – v)O2 and cardiac output are inversely related when VO2 remains constant. REF: pg. 383
OBJ: 9a
40. According to the Fick equation, what could cause the C(a – v)O2 to decrease? a. Exercise
b. High cardiac output c. Cardiogenic shock d. High fevers ANS: B
An increase in cardiac output decreases C(a – v)O2 because less oxygen needs to be extracted from each unit of blood that passes. REF: pg. 385
OBJ: 9a
41. Which of the following is not used in continuous cardiac output monitoring? a. Echocardiography b. Doppler ultrasound c. Pulse contour cardiac output monitoring d. Thermal dilution ANS: A
Echocardiography is not used to monitor the cardiac output continuously. REF: pg. 388
OBJ: 9
42. All of the following can be used to assess cardiac output noninvasively except: a. the pulse pressure. b. echocardiography. c. Doppler ultrasound. d. transthoracic electrical bioimpedance. ANS: C
The transtracheal Doppler ultrasound is attached to the end of an endotracheal tube. REF: pg. 389
OBJ: 9d
43. The periodic, noninvasive measurement of cardiac performance that requires only the use of a
sphygmomanometer is called: a. echocardiography. b. electrical bioimpedance. c. pulse contour cardiac output monitoring. d. pulse pressure. ANS: D
The pulse pressure reflects the stroke volume and can be assessed with only the use of a blood pressure cuff. REF: pg. 390
OBJ: 10
Chapter 17: Bronchoscopy Test Bank MULTIPLE CHOICE 1. What is the maximum depth that the standard adult bronchoscope can visualize in some
patients? a. Third-level bronchi b. Fourth-level bronchi c. Fifth-level bronchi d. Sixth-level bronchi ANS: C
The standard flexible bronchoscope has an external diameter of 5.3 mm and a total length of 605 mm (see Figure 17-1). It can pass easily down the trachea (first-order bronchus) and into the right or left mainstem bronchus and can enter most fourth-order bronchi and one-third of all fifth-order bronchi (see Figure 17-2). REF: pg. 398
OBJ: 2
2. What is the smooth-cusped flexible forceps used for during bronchoscopy? a. To clean the suction channel of the bronchoscope b. To obtain microbiologic samples c. To cut into large, bulky airway lesions d. To obtain a biopsy of the lung parenchyma ANS: B
The smooth-cusped flexible forceps can be used if the physician needs to obtain a piece of lung parenchyma during the procedure. If a large, dense (tough) lesion is seen in the larger airways, a serrated forceps capable of cutting tissue is preferred. The forceps is metallic and is easily visualized by a fluoroscope in radiology; thus, it can be visually directed with the use of real-time fluoroscopy for direct biopsy of peripheral lung lesions. REF: pg. 399
OBJ: 2
3. What are double-shielded brushes used for during bronchoscopy? a. To clean the fiberoptics b. To obtain microbiologic samples c. To assist in cleaning the smaller airways of mucous plugs d. To assist in diagnosing malignant tumors ANS: B
The double-shielded sterile brush is used to gather microbiologically important material for diagnosis of infection in the lung. REF: pg. 399
OBJ: 2
4. For what purpose are lasers used during bronchoscopy? a. To obliterate obstructing tumors b. To stop excessive bleeding
c. To open mucus-plugged airways d. To improve visualization of smaller airways ANS: A
Lasers can be used to obliterate tumors that obstruct large airways. REF: pg. 400
OBJ: 2
5. What is the most common indication for the use of a bronchoscope? a. To retrieve inhaled foreign objects b. To obtain microbiologic samples c. To help diagnose abnormalities seen on chest roentgenogram d. To treat hemoptysis ANS: C
The most common indication for flexible bronchoscopy is to diagnose the cause of an abnormality seen on a chest roentgenogram. REF: pg. 401
OBJ: 3 | 4
6. What is the advantage of using fluoroscopy during bronchoscopy for peripheral lesions? a. Less contamination of samples b. Increased diagnostic yield c. Increased sample size d. Less tissue destruction ANS: B
When fluoroscopy is used, the diagnostic yield is much higher.
TESTBANKSELLER.COM
REF: pg. 401
OBJ: 4
7. For what condition is the rigid bronchoscope most likely to be used? a. Pneumonia b. Massive hemoptysis c. Tumor d. Interstitial lung disease ANS: B
Occasionally, massive hemoptysis (>200 mL/24 hours) occurs, making intervention more urgent. Large-bore rigid bronchoscopes (see Figures 17-5 and 17-6) often are used in this setting because they allow more rapid removal of blood than is possible with flexible fiberoptic bronchoscopes. Their use generally requires an operating room, however, necessitating more time in preparation than does bedside flexible fiberoptic bronchoscopy. REF: pg. 401
OBJ: 4 | 7
8. What is the primary purpose of bronchoscopy in a patient with pneumonia? a. To obtain biopsy specimens b. To identify the causative organism c. To control hemoptysis d. To remove secretions ANS: B
The flexible bronchoscope can be useful in identifying the causative organism in difficult-to-diagnose cases of pneumonia. REF: pg. 402
OBJ: 4
9. What is the primary purpose of bronchoscopy in a patient with interstitial lung disease? a. To perform microscopic assessment of the biopsy sample, which often is
diagnostic b. To stop massive hemoptysis c. To identify the causative agent d. To remove excessive secretions ANS: A
Because of the difficulty of diagnosing these diseases, a piece of lung tissue usually must be examined under a microscope. REF: pg. 402
OBJ: 4
10. What complication associated with bronchoscopy is most common? a. A complication associated with medications used in the procedure b. Pneumothorax c. Secondary infection d. Excessive bleeding ANS: A
Few complications are associated with fiberoptic bronchoscopy (see Box 17-2). When they do occur, they are associated most often with medications used in the procedure, not with the procedure itself. REF: pg. 402
OBJ: 7
11. In which patients or situations is hypoxemia most likely to develop? a. Patients who have not been well sedated b. Prolonged bronchoscopic procedures c. Patients with pronounced anxiety d. Patients who experience vagal stimulation during a procedure ANS: B
The longer the procedure lasts, the greater is the risk of hypoxemia and hypercapnia. REF: pg. 404
OBJ: 7
12. What aerosolized medication is administered to most patients before the bronchoscopic
procedure is begun? a. Albuterol b. Racemic epinephrine c. Fentanyl d. Lidolocaine ANS: D
The patient is placed in the Fowler position and is given a medicated nebulizer treatment (unpressurized aerosol) that contains 5 mL of 4% topical lidolocaine over a 20-minute period.
REF: pg. 403
OBJ: 9
13. The respiratory therapist (RT) who is assisting the pulmonologist during bronchoscopy should
have medication cups of xylocaine, normal saline, and procedure. a. acetylcysteine (Mucomyst) b. epinephrine c. sterile water d. midazolam (Versed)
ready before beginning the
ANS: B
Samples usually are obtained quickly, so the RT must have an organized working surface with easily accessible supplies. Four medication cups should be available: 4% topical xylocaine, 2% topical xylocaine, 0.9% sodium chloride (NaCl), and a 1:19 solution of epinephrine. REF: pg. 406
OBJ: 9 | 10
14. What is the most common sign of xylocaine overdose during bronchoscopy? a. Tachycardia b. Bradycardia c. Seizures d. Apnea ANS: C
Seizures are the most common sign of acute xylocaine overdose. REF: pg. 407
OBJ: T7ESTBANKSELLER.COM
15. How are lung brushings handled? a. They are flushed in a saline wash. b. They are secured in gauze and labeled. c. They are preserved in formaldehyde. d. They are smeared onto a glass slide. ANS: D
To obtain a lung brushing, the bronchoscopist gently brushes the abnormal-appearing area and then withdraws the brush from the scope. The RT rubs the brush lightly on the frosted side of a slide, making a smear approximately the size of a dime. REF: pg. 407
OBJ: 5
16. Which of the following criteria is least important before a patient is discharged after
bronchoscopy? a. Gag reflex is present b. Stable vital signs c. No bleeding is present d. Ability to swallow without pain ANS: D
The following criteria should be met before the time of discharge:
The patient is alert and oriented; the gag reflex is present; sensation in the throat has returned to normal; vital signs are stable; and no bleeding is present. REF: pg. 407
OBJ: 11
17. The bronchoscopy technique that flushes 100 mL of normal saline through the scope's suction
channel in four or five increments of 20 to 30 mL to distend the distal bronchioles and fill the alveoli, thereby washing out samples of any microorganisms, is which of the following? a. Rigid bronchoscopy b. Bronchoalveolar lavage c. Bronchial fluoroscopy d. Transbronchial biopsy ANS: B
During the bronchoalveolar lavage (BAL), the bronchoscope is passed to the affected part of the lung, and the tip of the scope is positioned into a fourth-generation bronchus. A total of about 100 mL of normal saline is flushed through the scope's suction channel in four or five increments of 20 to 30 mL to distend the distal bronchioles and fill the alveoli, thereby washing out samples of any microorganisms. REF: pg. 399
OBJ: 5
18. All of the following would prevent a physician from performing bronchoscopy for a patient
except for a/an . a. PaO2 of 45 mm Hg b. international normalized ratio (INR) of 4.5 c. PaCO2 of 38 mm Hg d. current heart rate of 120 beats/min and rhythm of atrial fibrillation. ANS: C
Contraindications for Outpatient Flexible Bronchoscopy • Recent myocardial infarction • Lack of patient cooperation • Unstable severe asthma • Severe hypoxemia and/or severe hypercapnia • Bleeding disorders • Potentially lethal cardiac arrhythmias • Lung abscess • Renal failure • Immunosuppression • Obstruction of the superior vena cava REF: pg. 404
OBJ: 8
Chapter 18: Nutrition Assessment Test Bank MULTIPLE CHOICE 1. What gas is required for optimum production of adenosine triphosphate (ATP)? a. Argon b. Carbon dioxide c. Nitrous oxide d. Oxygen ANS: D
The use of food for energy at the cellular level requires oxygen to support a controlled combustion process that produces energy molecules of ATP, which are used in all body processes for energy (see Figure 18-2). REF: pg. 412
OBJ: 1 | 2 | 8
2. Which of the following is an indicator of the energy requirements of the patient? a. PaO2 to PaO2 ratio b. Minute ventilation c. Oxygen uptake per minute d. Vital capacity ANS: C
The metabolic rates of tissues dictate the amount of oxygen that must be picked up within the ESTBANKSELLER .COM lungs. Oxygen uptake ( OT 2) is a respiratory factor that can be measured in the laboratory or at the bedside. Nutritionally speaking, it is this measure that indicates the patient’s energy requirement. If O2 is measured while a person is in a resting, nonstressed state, the basal metabolic rate (BMR) or basal energy expenditure (BEE) can be calculated. REF: pgs. 413-414
OBJ: 2 | 3
3. Measuring the patient’s energy expenditure with the use of oxygen consumption is referred to
as a. b. c. d.
calorimetry. direct indirect simple complex
ANS: B
Indirect calorimetry measures respiratory parameters ( O2 and CO2) to determine the energy consumed by the body. Because oxygen is not stored in the body, measuring oxygen uptake ( O2) correlates directly with energy (ATP) creation and use. Metabolism resting energy expenditure (REE) then can be measured by oxygen consumption and is related directly to the energy (calories) used. REF: pg. 415
OBJ: 2 | 3
4. What element must be stored in sufficient quantities to meet the metabolic demands of the
diaphragm? a. Arginine b. Glycogen c. 2,3-DPG (diphosphoglycerate) d. Fat ANS: B
Blood sugar levels are maintained from liver glycogen (carbohydrate) stores between meals and during fasting. The liver’s glycogen stores come from the carbohydrates (starches and sugars) that are eaten in the diet. However, liver glycogen will be depleted within 12 to 16 hours unless sufficient carbohydrate is ingested again. REF: pg. 416
OBJ: 4 | 5
5. Which of the following is a pulmonary effect of starvation? a. Increased diffusing capacity of the lung for carbon monoxide (DLCO) b. Increased forced expiratory volume in 1 second (FEV1) c. Increased risk of pneumonia d. Increased functional residual capacity (FRC) ANS: C
Because our immune antibodies are composed of proteins, persistent low calorie and low protein intake will compromise the immune system, thereby limiting the body’s ability to fight pneumonia and other infections. REF: pg. 416
OBJ: 5
6. Which of the following might hinder attempts at nutritional repletion in patients with
respiratory disease? 1. The use of bronchodilators 2. Simple oxygen therapy 3. Intubation 4. Deep breathing and cough exercises a. 1, 2, 3 b. 1, 3 c. 2, 4 d. 1, 2, 3, 4 ANS: A
Nutritional repletion in respiratory patients often is hindered by necessary therapeutic actions. Bronchodilators may produce nausea; oxygen by nasal cannula disturbs the sense of smell and, therefore, taste because 70% of the taste of food is contributed by the sense of smell. Medications that patients are taking may interact with nutrients and render them less available for absorption or even may inhibit specific metabolic enzymes. An intubated patient really complicates the process of eating. REF: pg. 417
OBJ: 6
7. Which of the following is true about anaerobic metabolism?
1. It can cause metabolic acidosis.
2. It results in excess lactate production. 3. It results in excess production of carbon dioxide. 4. It is a very inefficient method of producing ATP. a. 1, 2, 3 b. 1, 3 c. 2, 4 d. 1, 2, 3, 4 ANS: D
In anaerobic metabolism, pyruvate produces lactate (lactic acid) rather than acetyl coenzyme A (CoA). The greater the energy production without sufficient oxygen, the more lactic acid is produced. This can result in lactic acidosis. If this occurs, the acid must be buffered with bicarbonate ( ), and carbon dioxide is produced in the process. This requires increased minute ventilation to eliminate the carbon dioxide generated. This is a short-term solution only. In the absence of oxygen, inefficient anaerobic metabolism will rapidly result in death. REF: pg. 419
OBJ: 7
8. What is the RQ value of a patient eating a pure carbohydrate diet? a. 0.60 b. 0.70 c. 0.85 d. 1.00 ANS: D
Pure fat metabolism has an RQ of 0.7 (more oxygen is used than the carbon dioxide produced), protein has a value of 0.85, and carbohydrate has a value of 1 (an even ratio, where one carbon dioxide (CO2) T mE olS ecTuB leAiN s pKrS odEuL ceLdEfR or.eC acOhMmolecule of oxygen (O2) used). REF: pg. 420
OBJ: 9
9. Of which of the following components should the majority of dietary intake be made up? a. Fat b. Carbohydrate c. Protein d. None of the above ANS: B
The food component that should constitute the largest amount of the dietary intake is carbohydrate. REF: pg. 421
OBJ: 10
10. Which of the following may be more difficult with a high-carbohydrate diet? a. Oxygenation b. Cellular gas exchange c. Oxygen transport d. Weaning from mechanical ventilation ANS: D
The major problem that has arisen with carbohydrate intake in patients with respiratory disorders occurs when glucose has been given in excess. The increased CO2 (a result of both the high RQ of glucose metabolism and its conversion to fat) has induced respiratory difficulty in patients who are being weaned from mechanical ventilators. This has been observed primarily when glucose has been administered by the parenteral (IV) route. REF: pg. 421
OBJ: 10 | 11
11. What dietary adjustment might be best for patients with more severe chronic obstructive
pulmonary disease (COPD)? a. Increased carbohydrates b. Increased fats c. Increased proteins d. Increased salt intake ANS: B
Patients with more severe COPD often do better with a higher fat content in the diet and less carbohydrate because carbon dioxide production is reduced. However, this does not lessen the importance of carbohydrates. Various carbohydrate and dietary fat percentages should be tried until the best combination is found for each patient. REF: pg. 422
OBJ: 6 | 10
12. What is nitrogen balance useful in determining? a. The adequacy of protein intake b. The adequacy of carbohydrate intake c. The need for vitamin sT upEpS leT mB enAtN atK ioS n ELLER.COM d. Fluid and electrolyte balance ANS: A
Nitrogen values often are referred to in discussion of protein requirements because nitrogen is found only in the amino acids that make up proteins. Measuring nitrogen, which is a simple laboratory measure, is the easiest way to measure protein intake or excretion. REF: pg. 422
OBJ: 12
13. Which of the following is associated with a low-protein diet? a. Increased work of breathing b. Increased fixed acid load c. Immune compromise d. Increased stress on the kidneys ANS: C
Too little protein compromises the immune system, promotes edema and ascites, produces generalized wasting of muscle tissue, and retards growth and proper development in children. REF: pg. 422
OBJ: 13
14. Which of the following is not associated with a high-fat diet? a. Increased risk of heart disease b. Decreased tissue oxygenation
c. Decreased DLCO d. Increased carbon dioxide production ANS: D
As the fat content of the diet increases, there is a measurable decrease in patients with COPD, this results in less dyspnea and improved function. REF: pg. 423
CO2. For many
OBJ: 14
15. What mineral plays a very important role in oxygen transport? a. Calcium b. Iron c. Zinc d. Magnesium ANS: B
The role of iron in oxygen transport and its use in hemoglobin and myoglobin and within the respiratory transport chain make it necessary to maintain iron at normal levels. REF: pg. 424
OBJ: 15
16. What method of nutritional delivery is considered the least efficient? a. By mouth b. By nasogastric tube c. By arterial line d. By IV line ANS: D
Total parenteral nutrition (TPN) is the feeding of patients by direct infusion of nutrients into a peripheral or central vein. There is a reluctance to feed patients by TPN because this route is not as efficient as the enteral route, it is expensive, and risks of complications such as infection are increased. REF: pg. 425
OBJ: 16
17. Which of the following are associated with increased metabolism?
1. Severe burns 2. Severe infection 3. Multiple traumas 4. Hyperthyroidism a. 1, 2, 3 b. 1, 3 c. 2, 4 d. 1, 2, 3, 4 ANS: D
Conditions that increase a patient’s metabolic rate or caloric and other nutrient requirements are multiple surgical or nonsurgical traumas; fever; infection; burns; long bone fractures; hyperthyroidism; and prolonged corticosteroid therapy. REF: pg. 426
OBJ: 17
18. Which of the following might indicate poor nutritional status? a. Low BMI b. Positive nitrogen balance c. Positive response to skin antigen testing d. High serum albumin level ANS: A
BMIs below 19 are associated with malnutrition problems and more prevalent pneumonia infection. REF: pg. 427
OBJ: 17
19. What test is most useful for screening a patient for protein malnutrition? a. Serum albumin level b. Triceps skinfold c. Creatinine–height index d. BMI ANS: A
Measurement of serum albumin levels provides a useful screening tool for detecting protein energy malnutrition. REF: pg. 429
OBJ: 18
20. How does the cachectic patient appear physically? a. Central obesity only b. Very thin and malnourished c. Edematous around the T faEceSTBANKSELLER.COM d. Very short of breath ANS: B
One should note the effects of body mass on breathing efficiency. Cachectic (nutritionally depleted) patients have readily outlined bony structures with depression of the intercostal spaces. Accessory muscles of respiration often are visible in these patients. REF: pg. 430
OBJ: 17 | 18
21. Which of the following findings during inspection would suggest that the patient is
malnourished? a. Generalized obesity b. Use of accessory muscles c. Presence of wheezes d. Weak cough ANS: D
During inspection of the patient, the amount of effort that can be generated during coughing should be observed. Muscle weakness accompanies poor nutrition. REF: pg. 430
OBJ: 18
Chapter 19: Sleep and Breathing Assessment Test Bank MULTIPLE CHOICE 1. According to the National Heart Lung and Blood Institute (NHLBI), approximately how many
Americans suffer from some type of sleep disorder? a. 10 million b. 20 million c. 30 million d. 40 million ANS: D
According to the NHLBI, approximately 40 million Americans suffer from some type of sleep disorder. REF: pg. 438
OBJ: 2
2. What test is used to diagnose sleep disorders? a. Overnight oximetry b. Polysomnography c. Electroencephalogram (EEG) d. Leg twitch study ANS: B
Respiratory therapists (RTs) represent a major segment of the workforce responsible for OM conducting a diagnostic sleTeE pS stT udByA , oNrKpS olEyL soLmEnR og.rC am (PSG), in sleep disorder centers (SDCs) throughout the country. REF: pg. 438
OBJ: 8
3. What stage of sleep is characterized by large eye rolls and low-amplitude waves on EEG
recordings? a. REM b. NREM stage 1 c. NREM stage 2 d. NREM stage 3 ANS: B
As they drift to sleep, children and adults normally begin with stage 1 non–rapid eye movement (NREM) sleep. During stage 1 NREM, the eyes roll slowly and low-amplitude waves (i.e., those seen at the vertical height of the EEG tracings) are noted. REF: pg. 438
OBJ: 4
4. Which phase of sleep is characterized by delta or slow waves? a. Stage 1 NREM b. Stage 2 NREM c. Stage 3 NREM d. REM
ANS: C
Stage 3 NREM (formerly stages 3 and 4) is considered the deepest stage of sleep and represents approximately 25% of the sleep period. The EEG demonstrates delta-wave, or slow-wave, sleep, which is characterized by high-amplitude waves. REF: pg. 438
OBJ: 4
5. In which stage of sleep is “restorative sleep” thought to occur? a. Stage 1 NREM b. Stage 2 NREM c. Stage 3 NREM d. REM ANS: C
Stage 3 NREM (formerly stages 3 and 4) is considered the deepest stage of sleep and represents approximately 25% of the sleep period. The EEG demonstrates delta-wave, or slow-wave, sleep, which is characterized by high-amplitude waves. “Delta sleep” is thought to represent restorative sleep. REF: pg. 438, Table 19-4
OBJ: 4
6. In which stage of sleep does the brain show electrical activity similar to wakefulness on EEG
tracings? a. Stage 1 NREM b. Stage 2 NREM c. Stage 3 NREM d. REM ANS: D
During rapid eye movement (REM) sleep, the brain is active and dreaming almost always occurs. EEG tracings demonstrate low-voltage, random, fast waveforms that are very similar to the brain wave activity seen when adults are awake. REF: pg. 439
OBJ: 4
7. A patient comment regarding the lack of dreaming could mean that the patient is not spending
a significant period of sleep in which stage? a. Stage 1 NREM b. Stage 2 NREM c. Stage 3 NREM d. REM ANS: D
The patient is spending less time in REM, as dreaming almost always occurs in REM. REF: pg. 439
OBJ: 5
8. During which stage of sleep are skeletal muscles partially paralyzed, thus greatly affecting
ventilation and maintenance of the upper airway? a. Stage 1 NREM b. Stage 2 NREM c. Stage 3 NREM
d. REM ANS: D
Electromyographic (EMG) monitoring demonstrates skeletal muscle tone at its lowest level during the REM stage, producing a paralyzing effect. This partial paralysis results in a further decrease in minute ventilation in healthy adults and in children, producing associated episodes of hypoxemia and hypercapnia. REF: pg. 439
OBJ: 4
9. What occurs during REM that results in sleep-disordered breathing (SDB)? a. Hypotension b. Rise in heart rate c. Fall in respiratory rate d. Upper airway narrowing ANS: D
For patients with SDB in REM, the soft tissues of the oropharynx relax because of partial paralysis, resulting in upper airway obstruction to air flow. REF: pg. 440
OBJ: 3 | 4
10. Which of the following would be most suggestive of a patient who is liable to have SDB? a. One with a smoking history b. An obese patient c. A very tall and thin patient d. A patient with a history of hypertension ANS: B
Results of the physical examination frequently are nonspecific and unremarkable for the patient with SDB. On inspection, patients most commonly present with obesity or with a normal body habitus, normal skin coloring, a normal respiratory rate, and no discernible features of SDB. REF: pg. 446
OBJ: 3 | 4
11. To meet the definitive criteria for sleep apnea, how many seconds of air flow cessation during
sleep must be documented? a. 4 seconds b. 6 seconds c. 8 seconds d. 10 seconds ANS: D
By far the most common problem found in SDB is sleep apnea. Sleep apnea is defined as the cessation of air flow for at least 10 seconds during sleep caused by an obstruction in the airway. REF: pg. 446
OBJ: 7
12. What is the first sign of SDB? a. Snoring
b. Excessive daytime sleepiness (EDS) c. Hypercarbia d. Cyanosis ANS: A
Snoring, the first sign of an SDB, may be noted in children and in adults. REF: pg. 448
OBJ: 3
13. What is the most common complaint of patients with obstructive sleep apnea? a. Snoring b. Headache c. Bedwetting d. EDS ANS: D
Although patients often may be unaware of snoring, they are aware of the other common symptom associated with sleep apnea and other SDBs, which is EDS. Excessive daytime sleepiness, or excessive daytime somnolence, is defined simply as difficulty in maintaining wakefulness. In contrast to those with sleep apnea, patients with EDS are very conscious of the fact that they have difficulty remaining awake during the day. REF: pg. 441
OBJ: 3 | 4
14. Which of the following will assist the RT in predicting sleep apnea? a. Epworth Sleepiness Scale (ESS) b. Berlin Questionnaire c. Inspection of the chest d. Electrocardiogram (ECG) ANS: A
The ESS is a simple, eight-item questionnaire that is intended to measure daytime sleepiness; it is essential for initial screening of sleep disorders. REF: pg. 441
OBJ: 5
15. Which of the following would not be part of the montage monitored during a polysomnogram
study (PSG)? a. SpO2 b. ECG c. EEG d. EMG of right and left arm ANS: D
The PSG page, or screen, is called a montage. A typical montage includes a minimum of 17 channels of information, or tracings, similar to an ECG tracing (see Figure 19-5). In fact, the ECG channel is always monitored during a PSG study, along with four channels that reflect the EEG tracings to identify NREM or REM sleep stages, two channels that document right and left eye movement, one channel that monitors chin movement, two channels that record right and left leg movement, and seven additional channels that record snoring, air flow, thoracic and abdominal movement, SpO2, heart rate, and body position (see Figure 19-6).
REF: pg. 444
OBJ: 9
16. A patient has an apnea-hypopnea index (AHI) of 15. What would this be considered? a. Normal b. Mild sleep apnea c. Moderate sleep apnea d. Severe sleep apnea ANS: B
An AHI of less than 5 means that the sleep study documented fewer than five hypopneas and/or apneas per hour, which is within the normal range. An AHI of 5 to 20 is termed “mild” sleep apnea, and an AHI of 20 to 40 indicates “moderate” sleep apnea. An AHI of more than 40 denotes “severe” sleep apnea in the adult population. REF: pg. 444
OBJ: 7
17. Which of the following diagnostically defines the presence of moderate sleep apnea? a. AHI of 3 b. AHI of 12 c. AHI of 32 d. AHI of 44 ANS: C
An AHI of less than 5 means that the sleep study documented fewer than five hypopneas and/or apneas per hour, which is within the normal range. An AHI of 5 to 20 is termed “mild” sleep apnea, and an AHI of 20 to 40 indicates “moderate” sleep apnea. An AHI of more than 40 denotes “severe” sleep apnea in the adult population. REF: pg. 444
OBJ: 7
18. How would you diagnose central sleep apnea (CSA)? a. Obesity b. Patient complains of EDS c. Excessive snoring d. 10-second apnea without respiratory effort during sleep ANS: D
CSA consists of 10 seconds of apnea in the absence of thoracic-abdominal effort. REF: pg. 446
OBJ: 7
19. A reduction in air flow of between 4% and 50% of baseline air flow for at least 10 seconds
during sleep defines which of the following? a. Apnea b. Respiratory effort-related arousal (RERA) c. Arousal d. Hypopnea ANS: D
A term that is associated closely with all types of apnea is hypopnea, which is defined as a reduction in air flow of between 4% and 50% of baseline air flow for at least 10 seconds, with at least a 4% reduction in SpO2 and/or an EEG arousal of at least 3 seconds.
REF: pg. 446
OBJ: 10
20. If, upon assessment, the Mallampati score is class 4, the patient would be at high risk for
which type of SDB? a. CSA b. OSA c. Mixed sleep apnea (MSA) d. SIDS ANS: B
Scoring of the Mallampati assessment from direct visualization is a follows: Class 1: full visibility of tonsils, uvula, and soft palate; class 2: visibility of hard and soft palate and upper portion of tonsils and uvula; class 3: soft and hard palate and base of the uvula are visible; class 4: only hard palate is visible (see Figure 19-10). REF: pg. 447
OBJ: 10
21. Which of the following are considered symptoms of obstructive sleep apnea (OSA)?
1. Morning headaches 2. Memory loss 3. Hyperactive behavior 4. Difficulty swallowing a. 1, 2, 3 b. 2, 4 c. 1, 3 d. 1, 2, 3, 4 ANS: A
Symptoms and signs of OSA in adults may include loud snoring, periods of apnea (as reported by sleep partners), chronic morning tiredness regardless of the length of sleep, EDS, morning headaches, recent weight gain, limited attention, memory loss, changes in judgment or personality, lethargy or sluggishness, hyperactive behavior, the need to take naps, depression, slowed responses, frequent falling asleep while driving, high blood pressure, automatic behavior (performing actions by rote), and swelling of the feet or legs. REF: pg. 448
OBJ: 7
22. Cheyne-Stokes breathing is associated most closely with which type of SDB? a. CSA b. OSA c. Mixed sleep apnea (MSA) d. Sudden infant death syndrome (SIDS) ANS: A
The most common sign of CSA is the presence of Cheyne-Stokes respirations. REF: pg. 449
OBJ: 7
23. With what pattern of SDB is SIDS thought to be associated? a. CSA b. OSA
c. MSA d. RERA ANS: A
SIDS is thought by some experts to be associated with CSA, but no clinical evidence that the two are linked has been presented to date. REF: pg. 450
OBJ: 11
24. What is the peak age of incidence for SIDS? a. Birth to 1 month b. 2 to 4 months c. 4 to 8 months d. 8 months to 1 year ANS: B
Although the cause is unknown, most reported cases of SIDS occur in the first 6 months of life, with a peak incidence reported at between 2 and 4 months of age. REF: pg. 450
OBJ: 11
Chapter 20: Home Care Patient Assessment Test Bank MULTIPLE CHOICE 1. For which of the following conditions (which is the third most common cause of death in the
United States) are both mortality rates and prevalence rates increasing? a. Chronic obstructive pulmonary disease (COPD) b. Asthma c. Pneumonia d. Heart failure ANS: A
COPD is a common cause of death in the United States. REF: pg. 455
OBJ: 1
2. In which of the following skills is it most important for a home care respiratory therapist (RT)
to be proficient? a. Decision making. b. Time management. c. Patient assessment. d. Communication and teaching skills. ANS: C
Patient assessment is the key to success for an RT who is providing home care. REF: pg. 455
OBJ: 4
3. Which of the following is not true of respiratory home care? a. It reduces the number of future hospitalizations. b. It provides a higher quality of life for the patient. c. It decreases overall expenditures for medical care. d. It decreases a patient’s ability to independently manage his or her care. ANS: D
Respiratory home care has been shown to reduce the number of future hospitalizations, reduce the costs of medical care, help a patient become more independent, and enhance a patient’s quality of life. REF: pg. 455
OBJ: 1
4. Which of the following conditions is least likely to be seen and treated by an RT in the home
care setting? a. COPD b. Pulmonary fibrosis c. Sleep-disordered breathing d. Burns ANS: D
Burn victims are not likely to be treated at home.
REF: pg. 455
OBJ: 3
5. What is the most common respiratory care modality applied in the home care setting? a. Intermittent positive pressure breathing (IPPB) b. Oxygen therapy c. Nebulizer therapy d. Continuous positive airway pressure (CPAP) ANS: B
Oxygen therapy is the most common respiratory care modality used in the home. REF: pg. 455
OBJ: 3
6. How many patients are expected to need home care in the United States by the year 2016? a. 5 million b. 12 million c. 25 million d. 50 million ANS: B
About 12 million patients are expected to need home care in the United States by the year 2016. REF: pg. 455
OBJ: 1
7. The primary tools used to conduct a home care patient assessment include all of the following
except a: a. stethoscope. b. pulse oximeter. c. peak flowmeter. d. bronchoscope. ANS: D
The bronchoscope is not used in the home care setting. REF: pg. 455
OBJ: 5
8. What is the name of the primary organization that accredits home care companies? a. Health Care Committee b. The Joint Commission c. Medical Examiner's Office d. Committee for Health Care ANS: B
The Joint Commission accredits most home care companies. REF: pg. 457
OBJ: 2
9. What is the primary role of an RT in home care? a. Communicate with other team members b. Do the paper work associated with home care
c. Provide education and equipment setup d. Give treatments as needed ANS: C
The primary role of the home care RT is to teach the patient and set up needed equipment. REF: pg. 457
OBJ: 4
10. Which of the following staff members is least likely to be a member of the home health care
team? a. Occupational therapist b. Physical therapist c. Social worker d. Radiation technologist ANS: D
Radiation technology is not needed in the home care setting. REF: pg. 457
OBJ: 4
11. Which of the following is a true statement related to reimbursement for RT services in the
home care setting? a. Only oxygen therapy orders allow the RT to be paid. b. The HME cannot bill for RT hours. c. All RT time spent in the home can be billed. d. Only time spent setting up equipment can be reimbursed. ANS: B
Current reimbursement laws do not allow HMEs to bill for the time an RT spends in the home teaching and treating the patient. REF: pg. 457
OBJ: 2
12. Which of the following should be identified by an RT during the initial home care interview
of the patient and caregivers? a. Patient smoking history b. Current list of medications c. Pulmonary risk factors d. All of the above ANS: D
The home care RT will need to interview the patient to identify numerous issues. The patient’s smoking history, current list of medications, and pulmonary risk factors such as animals in the home are just a few of the issues that must be identified. REF: pg. 460
OBJ: 6 | 7
13. Which of the following should be done only with a physician’s order? a. Chest auscultation b. Pulse oximetry c. Check of blood pressure d. Check of body temperature
ANS: B
Pulse oximetry should be done only after a physician order has been written. REF: pg. 460
OBJ: 8
14. Your home care patient has a chronic neuromuscular disease, and ventilation is a concern.
Which of the following tests would be most helpful in evaluating this? a. Chest auscultation b. Pulse oximetry c. ETCO2 d. Peak flow ANS: C
End-tidal carbon dioxide (ETCO2) levels reflect the degree of effective ventilation. REF: pg. 456
OBJ: 6 | 7
15. Which of the following may cause cognitive impairment of the home care patient and may
impede education efforts by an RT? a. Acute hypoxemia b. Stroke c. Certain medications d. All of the above ANS: D
Acute hypoxemia, stroke, and the use of certain medications such as sleeping pills have been known to impair cognitive function in patients. REF: pg. 461
OBJ: 7
16. What should the home care RT do if a patient cannot demonstrate proper use of his
metered-dose inhaler (MDI)? a. Contact the physician. b. Switch the patient to a small-volume nebulizer. c. Discontinue the use of the MDI. d. Call the patient’s nurse. ANS: A
Contacting the patient’s physician is a good idea if the patient cannot use the MDI correctly. REF: pg. 461
OBJ: 8
17. What is the role of the RT in evaluating psychosocial issues with the home care patient? a. The home care RT should not get involved in psychosocial issues. b. The home care RT should observe and document any psychosocial issues. c. Any such issues that are observed should be communicated to the physician. d. Both b and c are correct. ANS: D
The home care RT should observe and document any psychosocial issues that are present, such as anxiety or depression. These observations must be communicated to the patient’s attending physician.
REF: pg. 461
OBJ: 4 | 6
18. What is a common reason why patients with COPD may suffer acute dyspnea during meal
preparation? a. They have bronchospasm from the odors. b. They remove their oxygen cannula during cooking. c. They become nervous about gaining weight. d. They are afraid of losing their balance. ANS: B
Patients who cook are often afraid that their oxygen may explode around heat, so they often remove the oxygen cannula. REF: pg. 462
OBJ: 8
19. Which of the following should the RT assess during environmental evaluation of the home
care patient’s living place? a. The presence of smoke detectors b. An emergency exit route c. Sources for triggers of bronchospasm d. All of the above ANS: D
The home care RT does an environmental assessment of the patient’s living place to evaluate the safety and capabilities of the home. REF: pg. 463
OBJ: T8ESTBANKSELLER.COM
20. A visit by a home care RT is required for a patient who is using any of the following types of
equipment except: a. An oximeter. b. A bland aerosol therapy equipment. c. A pneumatic nebulizer. d. Oxygen therapy equipment. ANS: C
Patients who are using a pneumatic nebulizer do not have to be visited by the RT. REF: pg. 463
OBJ: 4 | 9
21. After teaching the patient to perform a specific skill for his or her home care, what should the
RT do? a. Document the demonstration. b. Repeat the demonstration. c. Ask for a return demonstration. d. Ask the patient if he or she understands. ANS: C
Asking the patient for a return demonstration is an excellent way to make sure that the patient understands the procedure.
REF: pg. 465
OBJ: 11
22. What may cause pedal edema in the home care patient? a. Fluid overload b. Liver failure c. Pneumonia d. Use of diuretics ANS: A
If the patient has too much fluid in the cardiovascular system, he or she often develops pedal edema. Pedal edema also is seen in patients with heart failure. REF: Table 20-2, pg. 466
OBJ: 3 | 10
23. The home care RT notices that the patient is producing more sputum than usual. What is the
most likely cause? a. Improved vital capacity b. An allergic reaction c. A respiratory infection d. Heart failure ANS: C
An increase in sputum production most often is a sign of airway infection. REF: Table 20-2, pg. 466
OBJ: 10
24. If the home care patient is suffering from a poor appetite, what could be the cause? a. Depression b. Dyspnea c. Infection d. All of the above ANS: D
A poor appetite can be caused by many conditions such as depression, shortness of breath, and infection. REF: Table 20-2, pg. 466
OBJ: 3 | 10
25. Which of the following tools should be provided by the healthcare team for a patient to
effectively manage his or her own care? a. Equipment and training in how to use it b. A knowledge base about disease and care needs c. The ability to monitor oneself and knowledge about when to notify members of the team d. All of the above ANS: D
The ultimate goal of respiratory home care is to provide a patient with the tools to manage his or her own care. These tools include a knowledge base about the general situation and the equipment and procedures to be used, and capability in self-monitoring. REF: pg. 466
OBJ: 12
26. The RT who assesses the patient to qualify him or her for home oxygen therapy should do all
of the following except: a. Be aware of the guidelines and criteria for qualifying the patient for oxygen therapy. b. Communicate the process for qualifying for oxygen to the patient. c. Be directly involved in assessing the patient to qualify the patient for oxygen therapy. d. Be aware of the physical and function limitations the patient has and assess oxygen needs with activity. ANS: C
The RT should be aware of the guidelines pertaining to qualifying a patient for home oxygen therapy reimbursement under Centers for Medicare and Medicaid Services (CMS) and should never be directly involved in the assessment process if affiliated with the home oxygen company. REF: pg. 466
OBJ: 9
Chapter 21: Documentation Test Bank MULTIPLE CHOICE 1. The patient’s medical record is a/an: a. financial document. b. legal document. c. educational tool. d. all of the above. ANS: D
The reasons for creating a record of the patient’s interactions with any healthcare organization (HCO) include the following: • To create a legal record of care and service provided; • To collect evidence in support of the patient’s problems and needs (when the clinical facts about the patient’s condition are collected, the correct diagnosis can be confirmed and the patient’s clinical progress can be better documented); • To provide communication between members of the healthcare team; • To support appropriate reimbursement; • To support the operation of the HCO and its allocation of internal resources and to provide documentation of compliance with The Joint Commission and regulatory standards of care; and • To prepare an educational tool. REF: pg. 470
OBJ: T1ESTBANKSELLER.COM
2. What is the primary goal of The Joint Commission? a. Monitor financial reimbursement of hospitals b. Review healthcare organizations to improve the quality of healthcare and patient
safety c. Provide healthcare workers with a safe work environment d. Monitor the ethical practice of medicine at healthcare organizations ANS: B
The Joint Commission performs an on-site survey of HCOs so as to assess the quality of patient care and to improve patient safety. REF: pg. 471
OBJ: 2
3. Which of the following organizations influences what needs to be documented in a patient’s
medical record? a. The Joint Commission b. Center for Medicare and Medicaid Services (CMS) c. Financial intermediaries d. All of the above ANS: A
The Joint Commission surveyors review patient records for documentation of high-quality patient care.
REF: pg. 471
OBJ: 2
4. Which of the following definitions is consistent with negligence? a. Failure to document a procedure performed on a patient b. Failure to explain to a patient the purpose of a therapy c. Failure to obtain a license to practice despite good clinical performance d. Failure to use a reasonable amount of care that results in injury or damage to
another ANS: D
Negligence is defined as an instance of failure to use a reasonable amount of care (ordinary prudence) that results in injury or damage to another. REF: pg. 472
OBJ: 3
5. Which of the following conditions is not required for the legal definition of negligence? a. The defendant owed a duty of care to the plaintiff. b. The defendant breached that duty. c. The plaintiff suffered a legally recognizable injury. d. The defendant’s breach of duty of care did not cause the plaintiff’s injury. ANS: D
Generally, the legal definition of negligence requires the presence of the following four conditions: • The defendant owed a duty of care to the plaintiff; • The defendant breached that duty; • The plaintiff suffered aTleEgSalTlyBrAecNoK gnSiE zaLbL leEinRju.rC y;OaM nd • The defendant’s breach of duty of care caused the plaintiff’s injury. REF: pg. 472
OBJ: 3
6. Which of the following outlines the professional standards for respiratory therapists (RTs)?
1. American Association for Respiratory Care (AARC) clinical practice guidelines 2. Respiratory care practice act and regulations 3. Place of employment 4. The Joint Commission a. 1, 2, and 4 b. 1, 2, and 3 c. 1, 2, 3, and 4 d. 2 and 4 ANS: C
The scope of your duty to a patient is outlined by your professional standard (e.g., the AARC clinical practice guidelines, your state’s respiratory care practice act and regulations, and The Joint Commission standards). Your scope of practice is defined further, or is limited, by your job description at your place of employment. REF: pg. 472
OBJ: 3
7. The absence of information or the lack of documented recognition of specific problems could
result in which one of the following situations? a. Malpractice b. Reduction in salary for an RT c. Reduction in workload d. Probation status for the clinician at fault ANS: A
The absence of information or the lack of documented recognition of specific problems could constitute malpractice. REF: pg. 472
OBJ: 3
8. Which of the following sections of the patient assessment or procedures should be charted
immediately? a. Date and time of test or treatment b. Vital signs c. Result of or response to treatment, including adverse reactions d. Drugs and their dosages ANS: B
Vital signs and parameters should be charted immediately. Late entries of clinical notes or observations should be marked clearly as late entries and should show the time entered and the time or period covered in the note. REF: pg. 473
OBJ: 4
tN coKnS siE steLnLt E wR ith.tChO eM definition of the SOAP charting 9. Which of the following woTrdEsSisTnBoA method? a. Subjective b. Objective c. Assessment d. Physical examination ANS: D
One of the most frequently used methods of documenting patient assessments is the SOAP (subjective, objective, assessment, and plan) charting method. REF: pg. 474
OBJ: 5
10. All of the following are examples of “objective” data, except: a. laboratory results. b. observation of a patient’s sleep apnea. c. the patient’s report of the amount of sputum that he or she produces daily. d. the physician’s interpretation of the patient’s electrocardiogram (ECG). ANS: C
Subjective information is what the patient can tell you about how he or she feels. REF: pg. 474
OBJ: 5
11. According to experts, obtaining a good
from a patient can give you a reasonable chance of correctly identifying a patient’s problem before you do a single test. a. arterial blood gas (ABG) results b. chief complaint c. medical history d. appearance ANS: C
According to experts, obtaining a good medical history from a patient can give you a reasonable chance of correctly identifying a patient’s problem before you do a single test. REF: pg. 474
OBJ: 5
12. Which of the following data do not constitute part of the objective information section in the
SOAP charting method? a. Vital signs b. Review of systems c. Review of clinical laboratory data d. Review of pulmonary function test results ANS: B
The O in SOAP stands for “objective.” This is everything you see, hear, feel, smell, and learn from tests and procedures. This section of the data collection can include the following: • Vital signs; • Physical examination information about the head and neck, abdomen, and extremities, to look for physical evidence of a respiratory abnormality; • Physical examination information about the thorax (heart and lungs) obtained by inspection, palpation, percT usE siS onT,BaA ndNaKuS scEuL ltaLtiE onR. (sC eeOCMhapter 5); • Review of clinical laboratory studies (see Chapter 7); • Review of arterial blood gases (see Chapter 8); • Review of pulmonary function tests (see Chapter 9); • Review of radiologic procedures, such as chest radiographs, computed tomograms, and magnetic resonance images (see Chapter 10); • Review of electrocardiograms (ECGs) (see Chapter 11); • Review of intensive care unit (ICU) hemodynamic data and cardiac outputs (see Chapters 15 and 16); and • Review of respiratory mechanics monitoring (see Chapter 14). REF: pg. 474
OBJ: 5
13. What does the letter I stand for in the APIE method of documentation? a. Implementation b. Impact c. Inconsistencies d. Initiative ANS: A
Primary goals of the assessment, plan, implementation, and evaluation (APIE) method are to condense data collection statements and to emphasize evaluation of the effectiveness of the interventions.
REF: pg. 477
OBJ: 6
14. Which of the following methods of documentation is probably best for a clinician who is
pressed for time? a. SOAP b. APIE c. PIP d. SBAR ANS: C
The PIP method works well for the experienced clinician who is pressed for time, wants to meet charting requirements, and needs to be brief. REF: pg. 477
OBJ: 6
15. Which of the following charting methods has been promoted with implementation of rapid
response teams (RRTs)? a. PIP b. SOAP c. SBAR d. APIE ANS: C
The SBAR method has proven highly successful in critical situations and has been promoted with the implementation of RRTs. It is most successful in documenting summary information, including fewer than five key points. It is not as effective for broad communication and context. REF: pg. 477
OBJ: 7
16. The major purpose of the electronic medical record (EMR) includes which of the following? a. Increase efficiencies in the healthcare system b. Improve the quality of patient care c. Increase patient safety d. All of the above ANS: D
The EMR stands to increase efficiencies in the healthcare system and improve the quality of care and patient safety. REF: pg. 473
OBJ: 8
17. All of the following are advantages of the EMR except: a. lack of standardization among systems. b. increased storage capacity. c. information is concurrently available even at remote sites. d. increased accuracy. ANS: A
The advantages of the EMR include the following: • Legibility;
• Increased storage capacity for longer periods of time; • Accessibility from remote sites; • Information that is concurrently available; • Built-in "alert" systems for critical tests and values; • Customized views for various users; • Increased management monitoring capabilities; and • Increased accuracy. The disadvantages of the EMR include the following: • High start-up and maintenance costs; • Significant learning curve for staff; • Confidentiality and security issues; and • Lack of standardization among systems. REF: pgs. 473-474
OBJ: 9